高校生のための数学の質問スレPART321

このエントリーをはてなブックマークに追加
1132人目の素数さん
前スレ
高校生のための数学の質問スレPART320
http://uni.2ch.net/test/read.cgi/math/1324126835/

【質問者必読!】
まず>>1-3をよく読んでね

数学@2ch掲示板用 掲示板での数学記号の書き方例と一般的な記号の使用例
http://mathmathmath.dotera.net/

・まずは教科書、参考書、web検索などで調べるようにしましょう。(特に基本的な公式など)
・問題の写し間違いには気をつけましょう。
・長い分母分子を含む分数はきちんと括弧でくくりましょう。
  (× x+1/x+2 ;  ○((x+1)/(x+2)) )
・丸文字、顔文字、その他は環境やブラウザによりうまく表示できない場合があります。
 どうしても画像を貼る場合はPCから直接見られるところに見やすい画像を貼ってください。
 ピクトはPCから見られないことがあるので避けてください。
・質問者は名前を騙られたくない場合、トリップを付けましょう。 (トリップの付け方は 名前(N)に 俺!#oretrip ←適当なトリ)
・質問者は回答者がわかるように問題を書くようにしましょう。でないと放置されることがあります。
  (変に省略するより全文書いた方がいい、また説明なく習慣的でない記号を使わないように)
・質問者は何が分からないのか、どこまで考えたのかを明記しましょう。それがない場合、放置されることがあります。
  (特に、自分でやってみたのにあわないので教えてほしい、みたいなときは必ず書くように)
・970くらいになったら次スレを立ててください。
2132人目の素数さん:2011/12/27(火) 23:27:30.00
基本的な記号の使い方は以下を参照してください。その他については>>1のサイトで。
■ 足し算/引き算/掛け算/割り算(加減乗除)
 a+b → a 足す b   (足し算)     a-b → a 引く b    (引き算)
 a*b → a 掛ける b  (掛け算)     a/b → a 割る b    (割り算)
■ 累乗 ^
 a^b     a の b乗
 a^(b+1)  a の b+1乗
 a^b + 1  (a の b乗) 足す 1
■ 括弧の使用
 a/(b + c) と a/b + c
 a/(b*c)  と a/b*c
 はそれぞれ、違う意味です。括弧を多用して、キチンと区別をつけてください。
■ 数列
 a[n] or a_(n)     → 数列aの第n項目
 a[n+1] = a[n] + 3  → 等差数列の一例
 Σ[k=1,n]a_(k)     → 数列の和
■ 積分 ( "∫"は「せきぶん」「いんてぐらる」「きごう」などで変換せよ(環境によって異なる).)
 ∫[0,1] x^2 dx = (x^3)/3|_[x=0,1]
 ∫[0,x] sin(t) dt
■ 三角関数
 (sin(x))^2 + (cos(x))^2 = 1
 cos(2x) = (cos(x))^2 - (sin(x))^2
■ ベクトル
 AB↑ a↑
 ベクトル:V=[V[1],V[2],...], |V>, V↑, vector(V)
 (混同しない場合はスカラーと同じ記号でいい.通常は縦ベクトルとして扱う.)
■行列
 (全成分表示):M=[[M[1,1],M[2,1],...],[M[1,2],M[2,2],...],...], I=[[1,0,0,...],[0,1,0,...],...]
 (行(または列ごと)に表示する. 例)M=[[1,-1],[3,2]])
■順列・組合せ
 P[n,k]=nPk, C[n.k]=nCk, H[n,k]=nHk
3132人目の素数さん:2011/12/27(火) 23:27:41.43
主な公式と記載例

(a±b)^2=a^2±2ab+b^2
(a±b)^3=a^3±3a^2b+3ab^2±b^3
a^3±b^3=(a±b)(a^2干ab+b^2)

√a*√b=√(ab)、√a/√b=√(a/b)、 √(a^2b)=a√b [a > 0、b > 0]
√((a+b)±2√(ab))=√a±√b [a > b > 0]

ax^2+bx+c=a(x-α)(x-β)=0 [a≠0、α+β=-b/a、αβ=c/a]
(α,β)=(-b±√(b^2-4ac))/2a  [2次方程式の解の公式]

a/sin(A)=b/sin(B)=c/sin(C)=2R [正弦定理]
a^2=b^2+c^2-2bccos(A)      [余弦定理]

sin(a±b)=sin(a)cos(b)±cos(a)sin(b)  [加法定理]
cos(a±b)=cos(a)cos(b)干sin(a)sin(b)

log_{a}(xy)=log_{a}(x)+log_{a}(y)
log_{a}(x/y)=log_{a}(x)-log_{a}(y)
log_{a}(x^n)=n(log_{a}(x))
log_{a}(x)=(log_{b}(x))/(log_{b}(a))  [底の変換定理]

f'(x)=lim_[h→0] (f(x+h)-f(x))/h  [微分の定義]
(f±g)'=f'±g'、(fg)'=f'g+fg'、(f/g)'=(f'g-fg')/(g^2) [和差積商の微分]
4132人目の素数さん:2011/12/28(水) 14:04:18.13
f(x)が微分可能なとき、 f(x)^n を定義に従って微分せよ。
全く方針がわかりません。
5132人目の素数さん:2011/12/28(水) 14:22:20.32
>>4
f(x+h) = f(x) + (f(x+h)-f(x))
f(x+h)^n = f(x)^n + n f(x)^{n-1} (f(x+h)-f(x)) + R(x,h)
ただし、R(x,h) = Σ_[k=2,n] C[n,k] f(x)^{n-k} (f(x+h)-f(x))^k
(f(x+h)^n-f(x)^n)/h = n f(x)^{n-1} (f(x+h)-f(x))/h + R(x,h)/h
あとは自分でやって
6132人目の素数さん:2011/12/28(水) 16:14:51.37
a>0とし、二つの放物線y=(a/2)x^2、y=(-a/2)x^2+a/(1+a)を考える。

二つの放物線の交点のx座標はx=±ア/√(イ+ウ)であり、二つの放物線に囲まれた部分の面積SはS=エ/オ×a/√{(イ+ウ)^カ}となる。

ここで、t=1/√(イ+ウ)とおくと、キ<t<クであり、S=エ/オ×(t-t^ケ)となる。

t=1/√コのとき、Sは最大値をとる。

したがって、a=サのとき面積Sは最大となり、その値はシ/スセ√ソである。

カタカナに数字か文字が入ります。スセは10というように二桁の数字です。

アイウが1,1,aだと出たのですが、エから1/6公式を使っても四角に合いません。

申し訳ないんですが答えはありません。

できれば計算過程があればありがたいです。

すみませんがよろしくお願いします。
7132人目の素数さん:2011/12/28(水) 16:43:07.39
なんとなくだけど、最大値 8/(9*√3) を答えにしていない?
(8/27)*√3 だと答えに収まるし、これを指していると思うが、紛らわしいな
8132人目の素数さん:2011/12/28(水) 17:26:03.50
実際の問題で分母か分子か紛らわしいことなんかありえないだろ
1/6公式の使い方間違えてるだけだよ多分
9132人目の素数さん:2011/12/28(水) 22:07:46.30
1/6公式の使い方、間違っていました。ありがとうございます。

またキクが0,1だと分かったのですが、それからまた進まなくなってしまいました・・・
10132人目の素数さん:2011/12/28(水) 22:25:46.31
ん?ケは素直に解けばいいだけじゃないの?
丁寧に誘導してくれてるわけだし。
11132人目の素数さん:2011/12/29(木) 00:19:07.27
初項a[1]=-5,公差dの等比数列{a[n]}と、初項b[1]=-5、公比r(r≠1)の等比数列{b[n]}がある。
a[3]=アイ+2*d,b[3]=エオr^2
と表される。
a[2]=2*b[2],a[3]=3*b[3] が成り立つとする。

(3)c[1]=0,c[n+1]=c[n]+a[n]/b[n] (n=1,2,3,,,,,,)によって、数列{c[n]}を定める。
このときc[n]はn=ト、ナ のとき最大となり、最大値はニである。

a[n]=-5+(n-1)*(5/3)、b[n]=-5*(1/3)^(n-1)

解答では
「c[n+1]-c[n]=-(n-4)*3^(n-2)
 ゆえに
 n=1,2,3 のとき c[n]<c[n+1]
 n=4 のとき c[n]=c[n+1]
 n=5,6 のとき c[n]>c[n+1]
 すなわち
  c[1]<c[2]<c[3]<c[4]=c[5]
  c[4]=c[5]>c[6]>c[7]・・・・
 であるから、c[n]はn=4,5のとき最大となり、最大値は、a[1]=b[1]、a[2]=2*b[2]、a[3]=3*b[3]より
 c[4]=c[1]+Σ_[k=1,3]a[k]/b[k]=c[1]+a[1]/b[1]+a[2]/b[2]+a[3]/b[3]=0+1+2+3=6   」
となっているのですが、
なぜc[n+1]-c[n]をもってきたのか、また、そこから一体どういう風に解答へ導くのか、教えて下さい。
お願いします。
12132人目の素数さん:2011/12/29(木) 04:04:44.08
>>11
  ・ c[n] についての漸化式は,階差数列がすぐに把握できる形
  ・ 階差数列の符号をもとに増減を判断することができる
ということを理解しよう

    c[n] < c[n+1]  …(あ)
   ⇔ c[n+1] − c[n] > 0 ⇔ a[n]/b[n] > 0
   ⇔ ( 4 − n )* 3^( n − 2 ) > 0  …(い)
この同値関係に着目して増減を捉える
  n = 1 は(い)をみたすから,(あ)で n = 1 とした c[1] < c[2] が成立
  n = 2 は(い)をみたすから,(あ)で n = 2 とした c[2] < c[3] が成立
  …
というふうに読み取っていく

反復試行の最大確率の問題などでも似たような考え方があっただろう
13132人目の素数さん:2011/12/29(木) 10:22:05.72
>>12
階差数列の基本的な部分を忘れてました;
回答ありがとうございます。
14132人目の素数さん:2011/12/29(木) 16:34:49.84
数学cでt検定出てきますか?
もし出てくるならば、教科書を手に入れようかと思ってます。
15132人目の素数さん:2011/12/29(木) 17:41:17.25
稠密性と連続性の区別もつかないアホーバカ袋の
apple^6くん、複素積分なんか自分の脳ミソにも
ないことを質問して、板を乱さないでくれるぅ〜。
ママのおっぱいでも吸ってなさい、一生離れられない
だろうけどね、チュバ厨場からw。
16132人目の素数さん:2011/12/30(金) 01:13:03.85
いきなり申し訳ありません。
cos3θ+cos5θ=0を解け。という問題で、

cos(4θ−θ)+cos(4θ+θ)=0
2cos4θcosθ=0
と変形しました。

自分が求めた解は
cos4θ=0より 4θ=π/2 +nπ ∴θ=π/8 +nπ/4
or
cosθ=0より θ=π/2 +nπ(nは整数)
としたのですが、

解答では、θ=π/8 +nπ/4 or θ=-π/2 +nπ(nは整数)

とここでの後者の解に-の符号が付きます。
自分の解答と模範解答にはどのような差がありますか?
自分の解答にもし不都合があればご指摘ください。
お願いします。
17132人目の素数さん:2011/12/30(金) 01:21:12.16
>>16
cosθ が0になる代表的なθは ±π/2
君の答案では − のほうが抜けている
18132人目の素数さん:2011/12/30(金) 01:23:44.31
>>17
失礼
π/2 +nπ としてあったか
なら君の答案でも問題ないはず
19132人目の素数さん:2011/12/30(金) 01:27:27.70
図形の問題で質問させて頂きます。
合同な三角形を2つ組み合わせた図形の面積を二等分する直線の引き方で、
解答には頂点から平行な直線を引けばいいとありましたが、どうしてそれ
でいいのか理由が分かりません。
よろしければ解説の方お願いします。
問題 題問1の(6)
 ttp://school.js88.com/high_school/h_kakomon_img.asp?id=13520
解答 
 ttp://school.js88.com/high_school/h_kakomon_img.asp?id=13526
20132人目の素数さん:2011/12/30(金) 01:29:19.61
レスありがとうございます。

僕自身でも考えてみたのですが、
腑に落ちる結論が出せませんでしたので質問させていただきました。
21132人目の素数さん:2011/12/30(金) 01:44:17.34
>>19
△BEC と △BEP の面積が等しくなればよい
△BEC を等積変形するので, C を通り BE と平行な直線を描く
22132人目の素数さん:2011/12/30(金) 01:57:26.39
>>21
確かに見れば共通の底辺と高さでした。
解説ありがとうございました。
23:2011/12/30(金) 03:35:27.55
>>16
問題ないと言うより、正解は π/2 +nπ の方を書くのが普通じゃないか?
24132人目の素数さん:2011/12/30(金) 09:27:50.83
平均変化率の問題なんですが

f(b)-f(a)
=(-b^2+2b+3)-(-a^2+2a+3)
=-(b^2-a^2)+2(b-a)
=-(b+a)(b-a)+2(b-a)
=(b-a)(-a-b+2)

で平均変化率の公式を使ってf(b)-f(a)/b-aを使って
-a-b+2になるのは分かるんですが
=-(b+a)(b-a)+2(b-a)が
=(b-a)(-a-b+2)になるのが分かりません。
中学生レベルの問題で申し訳ないのですが一体どんな計算を
してるのか教えてください。
25132人目の素数さん:2011/12/30(金) 10:52:41.03
-(b+a)(b-a)+2(b-a)
={-(b+a)+2}(b-a) [(b-a)でくくった]
=(b-a)(-a-b+2) [掛け算の順番をかえて式を整理]
26132人目の素数さん:2011/12/30(金) 11:03:41.38
バームクーヘン(年輪)積分が使える条件がわかりません
回転体の慨形を書いたときに座標軸を中心に円筒が書ければ使えるのでしょうか?
27132人目の素数さん:2011/12/30(金) 11:33:28.99
「円筒が書ける」の意味が分からんが、求める体積の中にピッタリ円筒面が収まるように積分する。
28132人目の素数さん:2011/12/30(金) 11:59:12.46
3x^100+2x^97+1をx^2+1で割った時の余りを求める問題で、
解をax+bと置いてx=iを代入すると3+2i+1=ai+bになることから
ai+b=2i+4であるから a=2 b=4
したがって求める余りは 2x+4 が答えなんですが、
これってつまり x=i,-i を代入した時に剰余の定理で余りが出るってことだと思うんですが
ここでなんで答えは 2i+4 または -2i+4 じゃダメなのか教えてください
29132人目の素数さん:2011/12/30(金) 13:19:56.61
2^(n-1) を3で割ったあまりをB(n)とするとき、B(2n-1)=1、B(2n)=2となることを示せ。

という問題なんですが、これはある程度書き出して
以上より〜題意のとおりになる。としてよいのでしょうか?

それとも数学的帰納法を使わないといけませんか?
使うとしたらどうすればいいでしょうか?
30132人目の素数さん:2011/12/30(金) 13:50:37.39
ある程度書き出すのは証明になってないからダメ
帰納法を使うならB(1)、B(2)で成り立ちB(2n)、B(2n-1)で成立を仮定して
B(2n+1)、B(2n+2)が成立する事を示せばいい
でも素直に割ったほうが早い
31132人目の素数さん:2011/12/30(金) 13:52:25.03
>>30
素直に割るときはどうやればいいでしょうか?
32132人目の素数さん:2011/12/30(金) 13:58:07.89
mod3
2^(2n-2)=(3-1)^(2n-2)≡(-1)^(2n-2)=1
2^(2n-1)=(3-1)^(2n-1)≡(-1)^(2n-1)=-1
33132人目の素数さん:2011/12/30(金) 14:18:09.06
>>32
ちょっと考えてたんですけどわかんなかったです・・
34132人目の素数さん:2011/12/30(金) 14:24:19.54
A,Bは自然数とする
AとBの最大公約数がGならばA+BとABの最大公約数はGである

これの逆を示すことはできるのでしょうか?
もし出来るのであれば過程を添えて教えていただけると助かります
35132人目の素数さん:2011/12/30(金) 14:30:18.44
反例 A=2 B=2
36132人目の素数さん:2011/12/30(金) 14:39:28.14
すみません、A≠Bが抜けていました;
37132人目の素数さん:2011/12/30(金) 14:41:29.94
(3-1)^n=Σ[k=0~n]nCk*3^k*(-1)^(n-k)
3で括れないのはnC0*(-1)^nだけだから余りは(-1)^nと言うわけ
38132人目の素数さん:2011/12/30(金) 14:43:11.28
A=3、B=6
39132人目の素数さん:2011/12/30(金) 14:50:59.20
バギャヤロー!
4034:2011/12/30(金) 14:53:24.11
>>38ありがとうございます
そもそももとの命題が成り立っていなかったようです。。失礼しました
41132人目の素数さん:2011/12/30(金) 16:52:12.75
>>28
君の言う「剰余の定理」というのは、1次式で割ったときの余りについてのものでは?
1次式で割るから余りは定数となるわけだけど
今は2次式で割るから余りは1次式となるわけだ

君が自分で書いたように、2i+4は余りではなく、ai+bの値に等しい

余談だが、今は実数係数の多項式を考えているので、余りも実数係数(つまり、a, bは実数)
42132人目の素数さん:2011/12/30(金) 19:37:05.19
相異なる実数の間には少なくとも一つ有理数が存在する事の証明を
高校範囲でするとしたらどいうなるでしょうか?
43132人目の素数さん:2011/12/30(金) 20:08:36.72
小数展開して
1.4142…と1.4143…となったとすれば
1.41425を考えればいい

もちろん小数展開の可能性を前提にしているので、循環論法であり、証明にはなっていないが
44KingMathematician ◆LoZDre77j4i1 :2011/12/30(金) 20:22:50.39
実数は整数を無限に一列に並べてそのすきまを線で引いた上の点で表現しているようだから,
相異なる実数は十分大きい整数をかけると距離が1より大きくなり間に整数があることがわかる.
実際かなりごまかしている.
45132人目の素数さん:2011/12/30(金) 20:26:50.93
100mg/dlって、パーセンテージで表すと何%になりますか?
46132人目の素数さん:2011/12/30(金) 20:28:44.51
てきとうに30%ぐらいにしておけば?
47KingMathematician ◆LoZDre77j4i1 :2011/12/30(金) 20:49:45.71
%=1/100.
4846:2011/12/30(金) 21:48:57.77
mを定数とし、f(x)=x^2+m+3、g(x)=-mx とする。
x≧0 で 常に f(x)>g(x)となるためのmの値の範囲を求めよ。

F(x)=f(x)-g(x) とおいて[F(x)の最小値]>0 が条件かな、と思ったんですが答えはm>-2 でした。
他の条件がどうにも分かりません。

よろしくお願いします

49132人目の素数さん:2011/12/30(金) 21:50:18.27
名前消すの忘れてました。>>48
>>46とは無関係です 失礼しました
50132人目の素数さん:2011/12/30(金) 21:56:22.95
>>48
x≧0 で
x≧0 で
x≧0 で
51132人目の素数さん:2011/12/30(金) 21:57:00.59
>>48
[x≧0の条件のもとでのF(x)の最小値]>0 でOK
52KingMathematician ◆LoZDre77j4i1 :2011/12/30(金) 21:58:33.55
Re:>>48 f(x)-g(x)の最小値がわかるならそれが0より大きいことだけでよい.
53132人目の素数さん:2011/12/30(金) 22:02:04.92
注意すべきは軸がx≧0の範囲にあるかどうか。
その範囲に軸があるなら、最小値の候補は軸のところか端点のx=0の2つ
その範囲に軸がないなら、最小値の候補は端点のx=0の1つのみ。
54132人目の素数さん:2011/12/30(金) 22:44:15.05
テスト
55132人目の素数さん:2011/12/30(金) 22:56:10.61
-m/2<0と-m/2≧0で場合分けしたらうまいこといきました。
ありがとうございました。
56132人目の素数さん:2011/12/30(金) 22:59:38.64
p,qが実数のとき、点(p+q,p−q)の通過する領域って求まりますか。
点(p+q,pq)だったら、p,qをtについての2次方程式の解とみなして
判別式に持ち込むのですが、点(p+q,p−q)の通過する領域は図示まで
もっていけません。何か方法はありますか。
x=
57132人目の素数さん:2011/12/30(金) 23:14:27.96
>>56
xy平面全部
58132人目の素数さん:2011/12/30(金) 23:27:19.91
>>57 そうなる理由を分かりやすく説明するのであれば
何て説明すればよいですか。
59132人目の素数さん:2011/12/30(金) 23:31:28.08
lim_[n→∞](Σ_[k=1,n](1/k)/ln(n))
60132人目の素数さん:2011/12/30(金) 23:42:48.30
>>56
俺は>>57ではないが…

qを定数とすると、(x,y)の軌跡は直線y=x-2q
ここで、qは実数全体より、-2qも実数全体
よってqを動かすと直線y=x-2qはxy平面の全ての点を通過する
61132人目の素数さん:2011/12/31(土) 00:56:23.35
>>56
例として出した解けると思っている問題では
何故2次方程式の実根条件にもっていくと解けるのかを理解してないようだ。
62132人目の素数さん:2011/12/31(土) 01:01:43.69
>>56
集合{(X,Y)|或る実数p,qがあって、X=p+q、Y=p-q} を A とおく。
平面上の任意の点を(x,y)として、 p=(x+y)/2, q=(x-y)/2とおくと
x=p+q、y=p-qであるから (x,y)∈Aである。
よって、Aは全平面。
63132人目の素数さん:2011/12/31(土) 01:39:48.19
任意の点pと原点を結ぶ線分がx軸となす角をαとしたとき、x 軸ついて対称移動する1次変換を表す行列は
I=[[1,0,],[0,-1]]となりますが、これを元の点が-2αだけ回転する回転変換で表そうとすると成分I(1,1)と成分I(2,2)がどちらもcosなので
先に書いたものと異なってしまいます。どこがおかしいのか教えてください。
64132人目の素数さん:2011/12/31(土) 01:43:22.73
対象移動と回転はさすがに違うだおr
65132人目の素数さん:2011/12/31(土) 01:58:05.28
そうですか。x軸に関する対称変換と回転変換の合成変換=直線に関する対称変換の証明問題があったのでよく分からなくなってしまいました。ありがとうございました。
66132人目の素数さん:2011/12/31(土) 02:14:46.83
三角形の重心ってまさかあの重心ですか?
67132人目の素数さん:2011/12/31(土) 02:18:30.97
大きな声じゃあ言えませんが、そうです、
まさかのあの重心ですよ。
このことは秘密ですからね、あまり言いふらさないでくださいね。
68132人目の素数さん:2011/12/31(土) 02:20:48.84
ポリンキー ポリンキー 三角形の秘密はね
69KingMathematician ◆LoZDre77j4i1 :2011/12/31(土) 05:04:25.88
重心は位置vector三つの和の1/3のことか.
位置vectorを三角形盤面上で積分したものを三角形の面積で割ったもののことか.
70132人目の素数さん:2011/12/31(土) 06:08:19.83
(1+√5):x=x:(1+√5−x)

の場合のxの長さを求めたい場合、どうしたらいいでしょうか。
宜しくお願いします。
71132人目の素数さん:2011/12/31(土) 06:58:45.37
外項の積と内項の積は等しい
比の問題の基本。それさえ知ってれば義務教育レベル。
72132人目の素数さん:2011/12/31(土) 07:50:05.94
途中の計算が解りません。
お願いします。
73132人目の素数さん:2011/12/31(土) 07:51:47.74
計算するだけやん。
手を動かせよ。
7470:2011/12/31(土) 07:59:07.43
解説を見ると、1+√5=aと置いて、x^2+ax−a^2=0となっています。
そこまでは解ります。
しかしその次にx=−1±√5/2*aとなっています。
この展開が解りません。
よろしくお願いします。
75132人目の素数さん:2011/12/31(土) 08:04:50.88
手を動かせよ、としか言いようがない…
76132人目の素数さん:2011/12/31(土) 08:06:16.83
解の公式。頑張れ。
77132人目の素数さん:2011/12/31(土) 08:16:46.79
Aを2次正方行列とし、対角和をt、行列式をdとします。
このとき A^2 - tA + dE = O が成り立ちます。
x^2 - tx + d = 0 ・・・(*)とします。

質問1  (*)が実数解をもたないとき、A-kE (kは実数定数)は逆行列をもつといえるでしょうか。
質問2  (*)が実数解α,βをもつとき、A-kEが逆行列をもたないのはk=α,βのときのみでしょうか。
78132人目の素数さん:2011/12/31(土) 08:19:56.66
>>75
式が変形しているということは何かをしたからですよね。
その過程を教えてください。
よろしくお願いします。
79132人目の素数さん:2011/12/31(土) 08:34:02.26
>>76
ありがとうございます。
解の公式に当てはめたら、−a±√(a^2−4a^2)/2となりました。
ここからどうしたらいいでしょうか。
80132人目の素数さん:2011/12/31(土) 08:49:38.81
計算間違いを探すと良い
81132人目の素数さん:2011/12/31(土) 08:51:01.37
代入して計算しろ。
勝手に置いたaのままではダメなことくらいわかれ。
82132人目の素数さん:2011/12/31(土) 09:07:36.81
>>80-81
ありがとうございます。
−a±√(5a^2)/2→−a±√5/2*aでしょうか。
しかしこのaに1+√5を当てはめても−(1+√5)±√5/2*aとなります。
解説にはx=−1±√5/2*aとありますが、それなら上の−√5はどこへ行ったんでしょうか。
83132人目の素数さん:2011/12/31(土) 09:08:34.23
>>77
複素数λに対し、 A-λE が逆行列を持たないこととλが(*)の解であることは同値
よって質問1も質問2も成り立つ
84132人目の素数さん:2011/12/31(土) 09:10:43.02
やっぱり括弧ルアーか
85132人目の素数さん:2011/12/31(土) 11:51:13.89
>>70
直感を使う。
直感によりxは黄金比になる。
86132人目の素数さん:2011/12/31(土) 12:07:45.16
その発想はなかった
87132人目の素数さん:2011/12/31(土) 17:49:40.53
A=BEのとき
A=CEとなる逆行列が存在しないことを証明せよ。
88KingMathematician ◆LoZDre77j4i1 :2011/12/31(土) 18:27:24.41
スレッド閲覧者に新たな試練が現れた.
89132人目の素数さん:2011/12/31(土) 19:19:02.20
うーんこれは難問だなぁ
90132人目の素数さん:2011/12/31(土) 20:16:02.64
ABC予想を彷彿させられる
91132人目の素数さん:2011/12/31(土) 21:26:28.61
逆2乗則の中心力によるポテンシャル内での1質点の運動で保存する量が
角運動量の3成分とエネルギーの計4っておかしくないですか?
残り2つはどうなったのでしょうか?
92132人目の素数さん:2011/12/31(土) 21:28:25.26
神様に聞けよ……
93132人目の素数さん:2011/12/31(土) 21:31:03.86
>>87
何の逆行列だよあほかす
94132人目の素数さん:2011/12/31(土) 21:53:35.70
物理板に行け
95132人目の素数さん:2011/12/31(土) 22:04:21.42
>>94
数学板で聞けって言われたもんで
96132人目の素数さん:2011/12/31(土) 22:09:31.16
んじゃ、哲学板で聞け
97132人目の素数さん:2011/12/31(土) 22:11:23.99
>>95
じゃ、死ねって言われたら死ぬのか?
98132人目の素数さん:2011/12/31(土) 22:35:00.05
>>91
なんなら相手してやるけど、残り 2つって何だ?
99132人目の素数さん:2011/12/31(土) 22:35:04.73
>>91
実はもうひとつ保存するベクトルがある
角運動量と直角なので独立にとれるのは2成分
それで全部
具体的に自分で求めるのは厳しいが
F↑=-kr↑/r^2として
A↑=p↑×L↑-mkr↑/rが保存することを確かめてみるといい
ハミルトン-ヤコビ方程式を学ぶと自力で保存する量を見つけやすくなるかもしれない
100132人目の素数さん:2011/12/31(土) 22:37:43.78
物理板レベル低すぎwwwwwwwwwwwwwwwwwwwwwwww
101132人目の素数さん:2011/12/31(土) 22:45:40.32
これって、中心力場では物体は平面運動をするということじゃ?
102132人目の素数さん:2011/12/31(土) 22:46:11.11
あいつらトンデモをいじり倒すぐらいしかできないやつの集まりだもんなwwwwwwwww
103132人目の素数さん:2011/12/31(土) 22:49:14.38
>>101
それは角運動量が保存することからわかる
104132人目の素数さん:2011/12/31(土) 22:49:36.27
>>99
ありがとうございます
105132人目の素数さん:2011/12/31(土) 22:50:06.56
>>98
おいwwwwwwwwwwwどうしたwwwwwwwwwwwwwwwwww
106ココ電球 _/::o-ν ◆tIS/.aX84. :2011/12/31(土) 23:24:25.21
下げでいやなことを書いてくるのは荒らしなんだから気にしなくていいのに
107132人目の素数さん:2011/12/31(土) 23:45:54.12
ここは物理板じゃないよ?
108猫好き:2011/12/31(土) 23:47:52.71
>>106
荒し頑張れw
109132人目の素数さん:2012/01/01(日) 00:02:49.39
a、bを定数とする。整式P(x)をx^2+x+aで割ったときの余りが3x+2、
整式(x+b)P(x)をx^2+x+aで割ったときの余りが
2x+8であるとき、a、bの値を求めよ。

商をQ(x)とおいたり、x=-bのときを考えたけど
よく分かりません。 答えはa=-2、b=1です。
よろしくお願いします。
110132人目の素数さん:2012/01/01(日) 00:07:41.87
あけおめことよろ
111132人目の素数さん:2012/01/01(日) 00:08:26.50
余りが0となる場合のxの値と因数定理かな
112猫好き:2012/01/01(日) 00:20:24.72
P(x)をx^2+x+aで割ったときの余りが3x+2 から、(x+b)P(x)を x^2+x+aで割ったときの
余りは(3b-1)x+(-3a+2b)
これと2x+8と比べておしまい。
113132人目の素数さん:2012/01/01(日) 00:35:47.05
>>112
>(x+b)P(x)を x^2+x+aで割ったときの
>余りは(3b-1)x+(-3a+2b)

3bx+2bはb(3x+2)っぽいんで何となく分かるんですが
-x-3aって何なんですか?

114132人目の素数さん:2012/01/01(日) 00:43:17.51
(x+b)P(x)を x^2+x+aで割ったときの余り
=(x+b)(3x+2)を x^2+x+aで割ったときの余り…(!)
なのはおk?
わからなければ、剰余した時の式を書いてみよう。
あとは、実際に(!)の割り算をするだけ。
115132人目の素数さん:2012/01/01(日) 03:21:04.58
>>70>>82
分かる方おられましたら教えてください。
116132人目の素数さん:2012/01/01(日) 03:33:05.52
>>115
二次方程式にしてそれを解く。
117132人目の素数さん:2012/01/01(日) 04:01:24.39
他者だけど解いてみた
x=[1+√5±{√(2)+√(10)}]/2
118132人目の素数さん:2012/01/01(日) 05:45:38.01
(-a±√5a)/2
-a(1±√5)/2
-(1+√5)(1±√5)/2
119132人目の素数さん:2012/01/01(日) 06:01:50.20
>>116-118
解けました。
ありがとうございます。
120132人目の素数さん:2012/01/01(日) 13:34:06.46
直線y=-1/2x+a上を自由に動く点Pがある

このとき原点O C(0,2)と定めて
OP+CPの最小値を求めよ

という問題なんですが

直角になるときというのはわかるんですが
答えを出しても

√(□a^2-○a+△)となっており答えが違います

どなたか教えてください
121132人目の素数さん:2012/01/01(日) 13:35:08.16
書き忘れました
a>2です
122132人目の素数さん:2012/01/01(日) 14:43:44.62
>>直角になるときというのはわかるんですが
この直感(?)が間違い。
直線に関し、(0,2)を対称移動した点をQとすると、
OP+CP=OP+PQ
Qが上のような地点にあると、OP,PQは、一直線に並ぶので、その長さが、OP+CPの最小値
123132人目の素数さん:2012/01/01(日) 15:44:10.69
理解できました

ありがとうございます

全く別のことを考えてました
124132人目の素数さん:2012/01/01(日) 16:37:52.63
中三の俺からみたら暗号にしかみえない
来年こんなむずそうなのやんのか
125132人目の素数さん:2012/01/01(日) 16:39:13.55
>>124
別にやらなくてもいーんだぜ?
義務教育じゃないし
126132人目の素数さん:2012/01/01(日) 16:49:12.00
>>125
たぶんあんたよりは勉強はできるから安心汁
127132人目の素数さん:2012/01/01(日) 17:43:05.28
小学生のとき中学生の数学は難しく見ただろ
128132人目の素数さん:2012/01/01(日) 18:12:21.30
いくら努力しようが凡人は天才には敵いませんよ

二次方程式の連立方程式は、
「お互いの式が等しい値をとる時の値をもとめるわけで、たまたま共通解
があれば共通解になる」
ということでいいのでしょうか?
129132人目の素数さん:2012/01/01(日) 18:30:24.39
式が未知数のことなら、二次は関係なくそう
130132人目の素数さん:2012/01/01(日) 19:05:02.49
導関数の問題なんですが
f(x)=-2x^2+4x+1の答えはf'(x)=-4x+4なのですが

どう計算してもf'(x)=-4x^2+4x+6になります。解法を教えてください。
131132人目の素数さん:2012/01/01(日) 19:07:41.86
導関数の公式を教科書で確認しよう
教科書は学校に置き放しかもしれないけど
132132人目の素数さん:2012/01/01(日) 19:11:56.89
>>129
返答感謝します。
未知数の事意外には何があるのでしょうか?
代数で2個のグラフとの共通点の探し出しと私は考えます
133132人目の素数さん:2012/01/01(日) 19:18:02.91
別に3個でも、もっとたくさんでも構わないよ
134あのこうちやんは始皇帝だった:2012/01/01(日) 19:20:10.23

 中学生や高校生の、クソガキは、こんな、ゴミ板に、来てはいけません!
 
 将来、ニートや無職になりますよ!!!!!!!!
135132人目の素数さん:2012/01/01(日) 19:20:54.24
>>131
公式確認してもう一度解きましたが出来ません。
教えてください。
136132人目の素数さん:2012/01/01(日) 19:21:12.34
2個に帰着できると言えなくもないか
137132人目の素数さん:2012/01/01(日) 19:47:42.70
>>135
ちゃんと理解できているかちょっと確認したい
   ( x^2 )’= □
   ( x )’= □
   ( 定数 )’= □
   { a * f(x) }’= □  ただし, a は定数である
   { f(x) + g( x ) }’= □ + □
□に入る式などを答えよ
138132人目の素数さん:2012/01/01(日) 19:59:38.39
139132人目の素数さん:2012/01/01(日) 23:14:46.07
fn(x)=x^n(1-x)^n nは正の整数
fn+2(x)の第2次導関数f"n+2(x)=Anfn+1(x)+Bnfn(x)が成り立つとき、
定数An,Bnをnで表せ。

fn+1(x)=x^(n+1)(1-x)^(n+1)
f'n+1(x)=(n+1)fn(x)(1-2x)
f"n+2(x)=(n+2)[(n+1)fn(x)(1-2x)^2-2fn+1(x)]
    =(n+2)[(n+1)fn(x)(1-4x+4x^2)-2fn+1(x)]

ここからいろいろ試したがどう変形していいのかわからず、
解答をみると
f"n+2(x)=(n+2)[(n+1)fn(x){1-4x(1-x)}-2fn+1(x)]
=〜
となっていました
どのようにすればこのような発想ができるようになりますか?
140132人目の素数さん:2012/01/01(日) 23:20:02.90
あなた f"n+2(x)=(n+2)[(n+1)fn(x)(1-4x+4x^2)-2fn+1(x)]
回答  f"n+2(x)=(n+2)[(n+1)fn(x){1-4x(1-x)}-2fn+1(x)]

(1-4x+4x^2) をくくって 1-4x(1-x) になっているだけ?
141132人目の素数さん:2012/01/02(月) 01:03:56.02
数学オリンピックに出ることになってたんですけど、今から無駄な抵抗するとしたら何をしたらいいでしょうか
142132人目の素数さん:2012/01/02(月) 01:07:54.62
映画『サマーウォーズ』の主人公は数オリ予選落ちしたものの
軍事基地のセキュリティ暗号を暗算(!)で解いてしまったんだぜ
143132人目の素数さん:2012/01/02(月) 01:08:56.18
2ax^-2x+4a-1=0が-1/3≦x≦2の範囲に少なくとも一つの解をもつときのaの値の範囲を求めよ、
という問題ですが、包絡線の考え方ではどうすれば良いですか。x^の係数が整数なら、例えば
「2x^-ax+a-1=0が-1≦x≦1の範囲に少なくとも1つの実数解をもつようなaの値の範囲を求めよ」という問題なら、
a-1をbと置き、2x^-ax+b=0として、xを定数と見、ab平面で考える事で簡単に答えが出ます。
けれど、x^の係数が文字になると分からなくなりました。
144132人目の素数さん:2012/01/02(月) 01:22:57.88
>>141
まず服を脱ぎます
145132人目の素数さん:2012/01/02(月) 01:24:12.87
>>141
右手で方程式を解き続け
左手でポテチを取るふりをしながら
146132人目の素数さん:2012/01/02(月) 02:05:22.88
キュウリを
147132人目の素数さん:2012/01/02(月) 02:22:50.51
>>143
a = 0 のときは条件をみたす解をもたないから,
与式を a で割って適当に置き換えればよい
しかし,本問では与式を a について整理して,
分数関数の増減を調べたほうがはるかに楽
個人的な趣味の問題でもあるが
148132人目の素数さん:2012/01/02(月) 04:49:09.64
x<2a+5を満たすxのうちで最大の整数が6であるとき、定数aの値の範囲をもとめよ。
問題の要旨がわかりません。
どういう意味ですか?
149132人目の素数さん:2012/01/02(月) 06:16:49.66
>>148
a に具体的な数値を入れてみて様子をつかむことを強くお勧めします
150132人目の素数さん:2012/01/02(月) 09:35:35.80
>>140
fn(x)=x^n(1-x)^n nは正の整数
fn+2(x)の第2次導関数f"n+2(x)=Anfn+1(x)+Bnfn(x)が成り立つとき、
定数An,Bnをnで表せ。

fn+1(x)=x^(n+1)(1-x)^(n+1)
f'n+1(x)=(n+1)fn(x)(1-2x)
f"n+2(x)=(n+2)[(n+1)fn(x)(1-2x)^2-2fn+1(x)]
    =(n+2)[(n+1)fn(x)(1-4x+4x^2)-2fn+1(x)]

ここからいろいろ試したがどう変形していいのかわからず、
解答をみると
f"n+2(x)=(n+2)[(n+1)fn(x){1-4x(1-x)}-2fn+1(x)] ・・・*
=(n+2)[(n+1)fn(x)-4(n+1)x(1-x)fn(x)-2fn+1(x)]
=(n+2)[(n+1)fn(x)-4(n+1)fn+1(x)-2fn+1(x)]
=-2(2n+3)(n+2)fn+1(x)+(n+2)(n+1)fn(x)
よってAn=-2(2n+3)(n+2),Bn=(n+2)(n+1)
となっており、*の式変形ができませんでした
どのようにすればこのような発想ができるようになりますか?
151132人目の素数さん:2012/01/02(月) 09:48:21.01
  ∧l二|ヘ
  (・ω・ )  おいらをどこかのスレに送るんよ
 ./ ̄ ̄ ̄ハ    お別れの時にはお土産を持たせるんよ
 |  福  | |
 |  袋  | |,,,....
   ̄ ̄ ̄ ̄

現在の所持品:日高・米・ロールケーキ・ごんじり・ポチ袋・数I教科書
152132人目の素数さん:2012/01/02(月) 10:01:44.79
>>147 ありがとうございます。答えが出せました。
しかし、場合わけをし、軸、f()の正負を用いて解くよりかは楽だとおもいますが、言われたとおり少し大変だと思いました。
分数関数の増減での方法の概要を教えていただけますか。因みに高1ですが、数Tは終わっています。数Uの途中ですが、包絡線は自学で理解しました。
153132人目の素数さん:2012/01/02(月) 10:46:52.45
>>143
> 「2x^-ax+a-1=0が-1≦x≦1の範囲に少なくとも1つの実数解をもつようなaの値の範囲を求めよ」という問題なら、
> a-1をbと置き、2x^-ax+b=0として、xを定数と見、ab平面で考える事で簡単に答えが出ます。
簡単にでるというこれはどうやんの?
154132人目の素数さん:2012/01/02(月) 12:18:14.42
青チャート1Aの問題が解答をみてもわからないので教えてください

p79 6の(2)の問題で
不等式 q(x+2)+p(x-1)>0 を満たすxの範囲が x<1/2 であるとき
不等式 p(x+2)+q(x-1)<0 を満たすxの範囲を求めよ。
ただし、pとqは実数の定数とする

という問題なのですが
まずp,qは定数なので 2つの不等式のp,qは共通であり
"不等式 q(x+2)+p(x-1)>0 を満たすxの範囲が x<1/2" との問題文から
この不等式をxについて解こうとしました。

q(x+2)+p(x-1)>0 から (p+q)x>q-2p
すなわち x>2q-p/p-q

としていって問題を解こうとしていたのですが

<解答> 
q(x+2)+p(x-1)>0 から (p+q)x>q-2p 
この不等式を満たすxの範囲が x<1/2 であるから
p+q<0・・・@ -2q+p/p-q=1/2・・・A

となっていて
(p+q)x>q-2p の範囲と x<1/2 の範囲が同じにであるとして
解答では解き進めているのですが

x<1/2 と q(x+2)+p(x-1)>0 から求めた範囲が
別々にならない というのがよくわかりません。
155132人目の素数さん:2012/01/02(月) 12:30:45.70
諸々の誤植はともかく、そもそも両者が同じ範囲になるようにする問題じゃないの?
156132人目の素数さん:2012/01/02(月) 12:46:58.83
ある物質を水で溶かした1%,5%,10%の水溶液がある。これら2種または3種の水溶液を混ぜ合わせて、7.3%の水溶液を100g作る場合、1%の水溶液は何gまで使用することが可能か。また、10%の水溶液にはどのような制限があるか。

三種類の水溶液があるので、かんがえかたがわかりません。
ご教授ください。
157KingMathematician ◆LoZDre77j4i1 :2012/01/02(月) 12:54:58.27
Re:>>156 方程式と不等式を作ればわかるかもしれない.そこから始めよう.
158132人目の素数さん:2012/01/02(月) 13:04:09.13
>>153
ちょっと楽にするため問題を変えます。-1<x<1に異なる二つの実数解をもつときにします。
変形してb=ax-2x^とする。xを動かした時の直線群は常にb=1/8a^に接する。この接点のa座標はa=4xなので、ここで-1<x<1より-4<a<4において放物線に接線が引けるa.bの範囲を考えればいい。そして、b=a-1がこの範囲に含まれているところを見つけるというか、求めればいい。
グラフを描いたら凄くわかりやすい。
今紙が手元にないから、もしかしたら変なことを書いたかもしれないけど、流れはこんな感じです。
159132人目の素数さん:2012/01/02(月) 13:12:44.14
>>157
まず、2種類または3種類というのがなんか難しく感じるのですが、問題から、1%の水溶液は必ず使うことがわかります。
10%の水溶液も必ず使うと思うのですが、5%の水溶液はどう判断したらいいですか?
160132人目の素数さん:2012/01/02(月) 13:32:06.94
>1%の水溶液は必ず使うことがわかります。
これはウソ、あまりたくさん使えないことは分かるが

>10%の水溶液も必ず使うと思うのですが
これは正しい

順に x[g], y[g], z[g] 使ったとすると
x+y+z=100, x>=0, y>=0, z>=0
水溶液中の物資量についての方程式がひとつ出来るので、あとは
xの上限、zの取りうる値の範囲を求めれば良い
161132人目の素数さん:2012/01/02(月) 14:01:30.85
行列の問題でA^n=0となる整数nが存在するとき、A^2=0であることを示せ
A^n=0の両辺右からA^-1をかける続けることにより、E=0これは矛盾する
よって題意は成り立つ
としたのですが、この証明は合っていますか?
162132人目の素数さん:2012/01/02(月) 14:04:47.69
全然合っていません、見事な0点
が、軌道修正は可能でしょう
163132人目の素数さん:2012/01/02(月) 14:07:40.31
>>160
1%を最大限に使える組み合わせが1%+10%のときで、このとき10%の水溶液の使用量は70g,
10%の水溶液をできる限り使わない組み合わせが5%+10%のときで、このとき10%の水溶液の使用量は46g
よって10%の水溶液を使用する制限は46g以上70g以下と考えたのですが間違いですか?
164132人目の素数さん:2012/01/02(月) 14:13:50.33
>>163
計算は知らんけど、考え方はOK
種類が増えると本当に困りそうだけどw
165132人目の素数さん:2012/01/02(月) 14:15:38.72
0°≦θ≦180°のとき、y=sin^2θ+cosθの最大値と最小値を求めなさい。また、そのときのθの値を求めなさい。
y=-cos^2θ+cosθ+1までしました。
ここからどうしていいかわかりません。
166132人目の素数さん:2012/01/02(月) 14:45:17.95
ab+4a+3bの因数分解の仕方を教えてください。
167132人目の素数さん:2012/01/02(月) 14:46:00.46
きゃいのきょうしき
168132人目の素数さん:2012/01/02(月) 14:57:43.54
154です
>>155
誤植なのですか・・・ わかりました ありがとうございました。
169132人目の素数さん:2012/01/02(月) 15:19:09.64
>>162
なぜ違うのでしょうか?
170132人目の素数さん:2012/01/02(月) 15:31:57.98
>>169
とりあえずツッこむと、
・A^(-1)は存在するのか?(実は存在しないが)
・途中でA^2=0になりそうなのはスルーか?
・そもそも、何と何が矛盾なのか?
171132人目の素数さん:2012/01/02(月) 16:33:50.48
>>152
2ax^2 − 2x + 4a − 1 = 0 より
   2a = ( 2x + 1 )/( x^2 + 2 )
右辺を f( x ) とおいて微分して増減を調べる( 数Vの範囲 )

ちなみに,>>143 のもう一つの問題
   「 2x^2 − ax + a − 1 = 0 が −1 ≦ x ≦ 1 の範囲に
   少なくとも1つの実数解をもつようなaの値の範囲を求めよ」
は,
   2x^2 = a( x − 1 ) + 1
などと 「 定数 a を分離 」 して視覚化するほうが多分楽

包絡線で解の配置を捉える解法は,今出ている参考書では多分
『数学ショートプログラム』にしか載っていない
著者の栗田先生もほかのところではこの解法を紹介していないようだし
つまり,あまりメジャーにはならなかったということだ

もちろん,包絡線が強力に役立つ問題もある(通過領域を捉える問題など)
適用すべきところを間違えないようにしよう
まぁ,趣味の問題なんだけど
172132人目の素数さん:2012/01/02(月) 17:13:03.27
>>158
簡単に出るというほどのことでもないような。
つまり、楽にするための制限を付け加えるのでは、最初に聞いた意図をはずされてしまっ感が。
173132人目の素数さん:2012/01/02(月) 18:12:53.95
>>165
cosθ = x とでも置き換えれば, x の2次関数
x の変域に注意して最大最小を捉えればよい

>>166
因数分解できません
整数問題などでよく使う,
   ab+4a+3b = ( a+3 )( b+4 ) − 12
のような変形のことをいっているのか?
上の変形がすぐに見えないのなら
   1つの文字について整理する
という原則に従えば手掛かりが得られる
174166:2012/01/02(月) 18:23:05.57
>>173
レスありがとうございます。
一つの文字について整理する、というのは
たとえばaについて整理すると

ab+4a+3b =(b+4)a+3b

となりますが、ここからどうすれば

( a+3 )( b+4 ) − 12 という答えが得られるのですか?
175 【東電 68.4 %】 :2012/01/02(月) 18:27:16.31
中三で知ってたら楽とか憶えたほうがいい公式ってありますか?
176132人目の素数さん:2012/01/02(月) 18:27:17.80
(b+4)a+3b
b+4でくくることを考えて次のように変形する
(b+4)a+3(b+4)-12
=(a+3)(b+4)-12
177166:2012/01/02(月) 19:27:55.38
>>176
おー。
ありがとうごぜえますだ。
178132人目の素数さん:2012/01/02(月) 20:08:44.18
xy座標上で(0,0) (a,b) (c,d) の三点を結んで出来る三角形の面積は1/2*|ad-bc|
179132人目の素数さん:2012/01/02(月) 20:10:37.58
長さa,b,cの三角形の面積はs=(a+b+c)/2で、S=√{s(s-a)(s-b)(s-c)}
180132人目の素数さん:2012/01/02(月) 20:15:52.71
相似、合同の証明はじつは省略しても結果さえわかる様に書けば点がもらえる
181132人目の素数さん:2012/01/02(月) 20:32:14.01
>>171
文字定数分離のやり方を忘れていました。
1つのやり方に固執してしまわないようにしないといけないですね。
微分はさすがにできないので、ごく簡単な問題→f(),軸,判別式。文字定数を分離して一次以下→文字定数分離。複雑な問題→f(),軸・包絡線。
という感じでいいですか。
182132人目の素数さん:2012/01/02(月) 20:55:05.05
>>181
個人的には,解の配置に包絡線は勧めにくい
包絡線の解法が理解できるくらい理解力があるなら
微分法の単純計算など容易にマスターできるだろう
文英堂『合格る計算』などで練習することを勧める
接線の方程式辺りまで理解すれば,包絡線の解法にも役に立つし
183132人目の素数さん:2012/01/02(月) 21:36:12.06
0°≦θ≦180°のとき、次の不等式を満たすθの範囲を求めよ。
tanθ≧-1
2つわからないことがあります。
tanθは、単位円で表すときなぜx=1上のy座標で表すのですか?
またそうだとして、この問題はy座標の-1≦の範囲を求めるのに、なぜ0°≦θ<90°,135°≦180°なのですか?0°≦θ<90°,90°<θ≦180°だと思ったのですが。http://beebee2see.appspot.com/i/azuY0YS7BQw.jpg
184132人目の素数さん:2012/01/02(月) 22:01:05.83
>>183
tan は 「 傾き ( = y/x ) 」 を表す
前半は, x を1にすると都合がよいというだけの話
後半も,傾きと見れば角の範囲が 0°≦ θ < 90°, 135°≦ 180° となることが納得できよう
185132人目の素数さん:2012/01/02(月) 22:08:02.04
>>183
>tanθは、単位円で表すときなぜx=1上のy座標で表すのですか?
正弦、余弦を考える時に単位円も用いるのは半径1で楽だから
同様にtanθ=y/xだから│x│=1だと楽
>y座標の-1≦の範囲を求めるのに、なぜ0°≦θ<90°,135°≦180°なのですか?
90°<θ≦180°のときx=-1であることに注意して考えよう
186132人目の素数さん:2012/01/02(月) 22:08:28.76
>>183
なぜ、0°≦θ<90°,135°≦θ≦180°と表すのかの間違いです。90°<θ<135°は違うんですか?
187132人目の素数さん:2012/01/02(月) 22:17:17.57
>>184>>185
一つめはわかりました。
分母を1にしたほうが簡単だからってことですね。
2つめは、x軸の正のほうから135°の傾きですよね?
なんで0°≦θ<90°,90°<θ≦135°じゃないんですか?
188132人目の素数さん:2012/01/02(月) 22:25:22.06
>>182
微分も予習も兼ねて少し勉強してみることにします。数Vの範囲と書いておられましたが、一対一対応(数U)に載っている微分法とは別物ですか?
接線の式は、f(x)=ax^の(t,f(t))における接線は、f(x)-l(x)=a(x-t)^っていうのをとくやつであってますか?
189 【東電 77.8 %】 :2012/01/02(月) 22:27:10.73
>>179
ヘロンの公式は習いました
このまえ塾でメネラウス習ったんだけどあれって需要ある?
方べき屈指すれば高校受検くらいはなんとかなるきがする
190132人目の素数さん:2012/01/02(月) 22:28:42.08

 下の投票所入口↓から(ちょっと遠回りしますが)

 http://engawa.2ch.net/test/read.cgi/sports/1325039544/3

 宮間あやちゃん(女子サッカー)へ皆さんの清き一票をお願いします。

191132人目の素数さん:2012/01/02(月) 22:30:13.87
>>187
tanθ=y/xよりy/x≧-1
90°<θ≦180°のときx=-1であるから
-y≧-1
よってy≦1
これを満たす範囲は90°<θ≦135°
192132人目の素数さん:2012/01/02(月) 22:33:57.19
>>191
これを満たす範囲は90°<θ≦135°
これは135°≦θ≦180°のミス
193132人目の素数さん:2012/01/02(月) 22:36:31.54
>>188
その接線の式は2次関数限定かな?

数Uでは扱う関数が整関数に限定されている
接線の公式自体は数Uも数Vも同じ形
理系志望なら少し背伸びして数Vの内容を眺めてみるのも悪くなかろう
194132人目の素数さん:2012/01/02(月) 23:08:09.64
>>193
二次関数以外はまだしていないです。
文系で経済志望なので、数Vは必要ないですが数学には強くなっておきたい、という感じです。
解の配置は、場合分けが「〜の範囲に少なくとも1つの解を・・・」問題はf(),軸とかのやり方は計算が多く、ミスしやすかったので他の解き方を探していたら、ショートプログラムの包絡線と出会いました。
「少なくとも」の時は結構使える気がしますが、微分は解の配置問題にオールマイティに対応できますか。
195132人目の素数さん:2012/01/02(月) 23:20:02.05
>>194
やはり問題ごとに臨機応変に対応するべきであるが,
x^2 の係数が文字のときは微分は強力であるとは言える
経済学部なら数V程度の微積は使うだろうから
ちょっと眺めて無理がなければかじってみればいいだろう
数学ばかりやるわけにもいかないだろうし…
196132人目の素数さん:2012/01/02(月) 23:34:43.40
>>195
いろんな問題の最適な解き方で解けるよう、練習しないといけないですね。
いろいろ教えていただき、ありがとうございました。また何かあればよろしくお願いします。
197132人目の素数さん:2012/01/03(火) 00:00:42.77
>>191
ありがとうございます。
198132人目の素数さん:2012/01/03(火) 00:27:22.24
このスレにも天才はいるのですか
199132人目の素数さん:2012/01/03(火) 00:28:11.91
>>198
200132人目の素数さん:2012/01/03(火) 01:00:15.87
等式4x+2y+z=15を満たす正の約数x,y,zの組4(x,y,z)をすべてもとめよ。
どうもとめたらいいですか?
201132人目の素数さん:2012/01/03(火) 01:02:38.21
ちょっと問題文がいみわからないです
202132人目の素数さん:2012/01/03(火) 01:03:35.55
>>200
4ってのは誤字です。
203132人目の素数さん:2012/01/03(火) 01:04:45.49
なるほど4は誤字か…
204132人目の素数さん:2012/01/03(火) 01:06:39.87
>>203
その辺は善処してください。
205132人目の素数さん:2012/01/03(火) 01:07:49.86
約数ってなんですか
206132人目の素数さん:2012/01/03(火) 01:10:00.99
>>205
ほんとごめんなさい。整数の間違いです。
207132人目の素数さん:2012/01/03(火) 01:11:02.92
>>200
xで範囲を絞るのが定石
208132人目の素数さん:2012/01/03(火) 01:13:09.26
>>206
x,y,z≧1
仮にx≧y≧zとしたら与式の範囲は?
209132人目の素数さん:2012/01/03(火) 01:14:24.08
>>208
アホ
210132人目の素数さん:2012/01/03(火) 01:15:42.31
>>207
なぜそうしたほうがいいと思ったのか知りたいです。
わからない問題が出て来たら思考が止まってしまうんです。
211132人目の素数さん:2012/01/03(火) 01:17:50.49
どうせ志望校では整数問題でないからやらんでいいよ
212132人目の素数さん:2012/01/03(火) 01:18:36.80
係数の大きいものほど取りうる値の範囲が狭いから
213132人目の素数さん:2012/01/03(火) 01:21:07.78
じゃあ両辺に大きな数字をかけて係数を大きくしたほうがいいんですか
214132人目の素数さん:2012/01/03(火) 01:21:50.62
>>213
偽者
215132人目の素数さん:2012/01/03(火) 01:28:22.45
>>212
ありがとうございます
とりあえず悩んでみます。
話変わりますが、黄チャートを制覇できたら大学的にどの程度のレベルですか?文系理系でわかれるとは思いますが。
216132人目の素数さん:2012/01/03(火) 01:29:23.46
>>215
京大レベル
217132人目の素数さん:2012/01/03(火) 01:31:03.32
>>216
偽者
218132人目の素数さん:2012/01/03(火) 01:31:29.41
>>216
mjd?
ここにいる人たち旧帝大レベルの人だらけなのかな。
219132人目の素数さん:2012/01/03(火) 01:33:08.07
>>218
高校生と駅弁で落ちこぼれた大学生
220132人目の素数さん:2012/01/03(火) 01:33:24.91
たまに中学生
221132人目の素数さん:2012/01/03(火) 01:34:47.96
最大公約数は、数学好き。
222132人目の素数さん:2012/01/03(火) 01:38:29.08
中学生に教わる数学はむずかしいか
223132人目の素数さん:2012/01/03(火) 03:20:18.38
>>200
偶数か奇数かで分けた方が良い。
224132人目の素数さん:2012/01/03(火) 13:07:53.88
えっ?
225132人目の素数さん:2012/01/03(火) 13:12:32.81
x^4+x^2-2ax-a^2+1 は因数分解できますか? どうやったらいいんですか?
226 【東電 66.9 %】 :2012/01/03(火) 13:16:50.65
だれかメネラウスの定理証明して
227132人目の素数さん:2012/01/03(火) 13:18:07.94
228132人目の素数さん:2012/01/03(火) 13:21:20.09
>>225
aについて整理
229 【東電 66.9 %】 :2012/01/03(火) 13:29:55.30
>>227
理解できました。
塾だと式しか教えてもらえなかったので
ありです
230132人目の素数さん:2012/01/03(火) 13:49:07.57
>>229
そんな塾さっさとやめちまいな
231132人目の素数さん:2012/01/03(火) 13:53:16.49
>>228 aについて整理した後に途方にくれます。
232132人目の素数さん:2012/01/03(火) 13:55:10.71
>>231
式を書け
233132人目の素数さん:2012/01/03(火) 14:44:50.46
-a(a+2x)+x^4+x^2+1 この後何をどうすりゃいいのか全然わかりません
でしたが いろいろ調べたら後ろは(x^2+x+1)(x^2-x+1)になるのはわかりました
これからどういじるのかさっぱり見当もつきません
234132人目の素数さん:2012/01/03(火) 14:51:24.24
>>233
aはどこいった
ケツの穴にでも隠したのか?
235132人目の素数さん:2012/01/03(火) 14:51:58.22
>>233
> -a(a+2x)+x^4+x^2+1
aの降巾の順に整理しろといっているのだよ。
aの二次式になり平方完成できる。
236132人目の素数さん:2012/01/03(火) 14:54:10.04
>>233
aについて整理というのはそういうことじゃない。
-a(a+2x)なんていうふうに一部だけ因数分解しちゃダメ(ダメじゃない場合もなくはないかも知れないが)。
aについて次数の高い順に整理する。
237132人目の素数さん:2012/01/03(火) 14:57:16.79
>>235
> aの二次式になり平方完成できる。
定数項は平方完成できる。
238132人目の素数さん:2012/01/03(火) 15:01:47.94
>>237
訂正ばっかり
とにかく平方完成する。
239132人目の素数さん:2012/01/03(火) 15:48:37.63
>>233
とりま、x^4+x^2+1の因数分解から説明。
x^4+x^2+1
=x^4+x^2+1+x^2-x^2 (x^2を足して引いた)
=(x^4+2x^2+1)-x^2 (後ろの-x^2だけ放っておく)
=(t^2+2t+1)-x^2 (x^2=tとした)
=(t+1)^2-x^2
=A^2-x^2 (t+1=Aとした)
=(A+x)(A-x)
=(t+1+x)(t+1-x)
=(x^2+x+1)(x^2-x+1)
次に、本題。
x^4+x^2-2ax-a^2+1
=-a^2-2xa+x^4-x^2+1
=-a^2-2xa+(x^2+x+1)(x^2-x+1)
ここで「たすきがけ」。
-(x^2+x+1)+(x^2-x+1)=-2xより、
(a+x^2+x+1)(-a+x^2-x+1)
=(x^2+x+a+1)(x^2-x-a+1)
240132人目の素数さん:2012/01/03(火) 15:54:41.13
あーあ
241132人目の素数さん:2012/01/03(火) 16:09:02.24
x^4+x^2-2ax-a^2+1=
(x+1/2(√(-4a-3)+1))(x-1/2(√(-4a-3)-1)))(x-1/2(1-√(4a-3)))(x-1/2(√(4a-3)+1)))
で間違いない
242132人目の素数さん:2012/01/03(火) 16:10:31.25
あほで間違いない
243132人目の素数さん:2012/01/03(火) 16:16:33.77
複素数体は代数的閉体で間違いない
244132人目の素数さん:2012/01/03(火) 16:24:54.64
代数学の基本定理より複素数体は代数的閉体で1次多項式の積に分解で間違いない
245132人目の素数さん:2012/01/03(火) 16:36:47.37
数学者もオナニーするの?
246132人目の素数さん:2012/01/03(火) 16:37:01.00
ひまか(課長風に)
247132人目の素数さん:2012/01/03(火) 18:54:21.46
数学者はオナヌーしません
例外的に各月の素数日にだけオナヌーするのがいたが自害したよ
-a^2-2ax+x^4+x^2+1
=-(a+x)^2+x^4+2x^2+1
=-(a+x)^2+(x^2+1)^2
=(x^2+1+a+x)(x^2+1-a-x)
=(x^2+x+a+1)(x^2-x-a-1)
249132人目の素数さん:2012/01/03(火) 20:58:53.33
因数分解が人生に関係あるのかよ
250132人目の素数さん:2012/01/03(火) 21:00:30.96
人生の選択には関係あるな
251132人目の素数さん:2012/01/03(火) 21:04:55.62
選択されるよ
252132人目の素数さん:2012/01/03(火) 21:50:37.19
癖で10000 の0000をつなげてσσσσって書いたり、10000を10,000って書くんだけど、後で気づいて書き直したり面倒。
ぶっちゃけ、不正解にしないだろうと思うが、どうだろうか?
253132人目の素数さん:2012/01/03(火) 21:51:17.78
三角形OABの辺OAを2:3に内分する点をC、辺OBの中点をD、辺ABを1:2に内分する点をEとし、線分BC,DEの交点をPとする。
以下
【ベクトル記号任意省略】

(1)OPをOA,OBで表せ。
このとき、s:(1-s)などのように足して1になるように置きますが、これは何故でしょう。
このように置くと解けますが。

(2)OPの延長と辺ABとの交点をQとするとき、OQをOA,OBで表せ。
(1)よりOP=2/9OA+4/9OB
解答としては
O,P,Qが一直線上の点より
OQ=kOP(kは定数)
=2/9kOA+4/9kOB
また、点Qは辺AB上の点であるから
2/9k+4/9k=1 ←ここも分かりません。
と、kを求めて...となります。

しかしここでチェバの定理から
AQ:QB=m:(1-m)とおいて、(←論理不理解であるが)計算すれば、
m=3/5と出ました。
つまり、
AQ:QB=3/5:2/5
ここからベクトルのたすきがけをやってもOQの値は合いませんでした。
これは何故でしょう。

2点疑問でした。
254132人目の素数さん:2012/01/03(火) 21:56:36.86
注意はするが不正解にはしない。
255132人目の素数さん:2012/01/03(火) 21:57:20.89
質問文が3行以上だと萎える
256132人目の素数さん:2012/01/03(火) 21:59:37.83
>>253
教科書か問題集で既出のはず
257132人目の素数さん:2012/01/03(火) 22:23:26.01
2次関数f(x)=-x^2+2xのa≦x≦a+2における最大値、最小値はaの関数であり、これをそれぞれF(a)、G(a)と表す。この関数F(a)、G(a)のグラフを書け。
意味がわかりません。aの関数が最大値、最小値とはどういう意味ですか?
258132人目の素数さん:2012/01/03(火) 22:25:53.40
平方完成してxの範囲を動かしてみれば自ずと答えは見えてくるかと
259132人目の素数さん:2012/01/03(火) 22:27:13.99
>>253
> このとき、s:(1-s)などのように足して1になるように置きますが
内分の式わかる?
260132人目の素数さん:2012/01/03(火) 22:34:56.21
>>253
この手のベクトルの公式導出の際にもっとも基本となるのは
    A , B , P が同一直線上にある
  ⇔ AP↑ = tAB↑ …(あ) ( t は実数 )
である
(あ)を位置ベクトルで書き換えて整理すれば,
   ・ t : ( 1 − t ) と比を設定する
   ・ 係数の和が1になる
理由がわかるだろう

次に,本問の構図は,
   チェバの定理が使える構図ではない
チェバの定理では各頂点から線が出ているが
本問は頂点でない E から線が出ている
261132人目の素数さん:2012/01/03(火) 22:37:53.04
>>260
追記
なお,線分 OE を描けば,メネラウスの定理を複数回使うことで解決する
262132人目の素数さん:2012/01/03(火) 22:39:04.53
お詳しいですね
263132人目の素数さん:2012/01/03(火) 22:45:09.45
>>260
天才
264132人目の素数さん:2012/01/03(火) 22:56:54.97
こんな所で質問するのもあほらしいかもしれませんが質問させてください

ロト6の確立なのですが、wikiには43C6で計算しているみたいです
43*42*41*40*39*38ではダメな理由を教えて頂けませんか?
265132人目の素数さん:2012/01/03(火) 22:57:29.38
東大生ですか?
266132人目の素数さん:2012/01/03(火) 22:59:01.78
>>264
選ぶんだから
267132人目の素数さん:2012/01/03(火) 22:59:03.29
>>264
ロト6は順番関係ないから。
268132人目の素数さん:2012/01/03(火) 23:00:58.25
>>257
「 a の関数になる」という表現がしっくりしないなら,とりあえず
「 a の式になる」と思っておけばよい
( ただし,本問では軸での値は a を含まない式になるが…)

2次関数の最大最小では,普通は「区間と軸の位置関係で場合分け」を行う
しかし,2次関数の場合,最大値,最小値をとる x の「候補」は
   ・軸(ただし,軸が区間内にあるときだけ有効)
   ・両端点
の3者だけである
これらを
   全部図示してしまってから比較する
と,場合分けで頭を使わずに済む
本問はこうして得られた図がそのまま解答に直結するので,なおお得である
269132人目の素数さん:2012/01/03(火) 23:02:32.65
>>260
この解答おすすめ
270132人目の素数さん:2012/01/03(火) 23:14:58.06
>>266
>>267
言われたこと考えてようやく分かりました、ありがとうございました


もう一つ質問なんですけど、例えば

 ・5択の当たりくじ(もちろんあたりは1つ)を13回連続でハズレを引き続ける確立

は(4/5)^13でいいですか?
271132人目の素数さん:2012/01/03(火) 23:16:47.41
>>260
偏差値80ですか?
272132人目の素数さん:2012/01/03(火) 23:17:55.56
>>260
IQ200ですか?
273132人目の素数さん:2012/01/03(火) 23:22:47.37
>>260
ハーバードの推薦もとれるのですか?
274132人目の素数さん:2012/01/03(火) 23:39:20.75
世界はなぜ平和にならないの?
275132人目の素数さん:2012/01/03(火) 23:40:26.20
ベクトル分かりました。
>>260さんを始めとして、付き合って下さった方々本当にありがとうございました。
276132人目の素数さん:2012/01/03(火) 23:42:12.42
三角形OABの
辺OAを2:3に内分する点をC、C=2A/3
辺OBの中点をD、D=B/2
辺ABを1:2に内分する点をEとし、E=A+AB/2
線分BC,BC=C-B=2A/3-B
DEの DE=E-D=A+AB/2-B/2=A/2
交点をPとする。P=B+BCt=D+DEs=B+t(2A/3-B)=B/2+(A/2)s
B(1-1/2-t)=A(s/2-2t/3)=0
t=1/2,s=2/3,P=B/2+A/3

以下
【ベクトル記号任意省略】

(1)OPをOA,OBで表せ。

(2)OPの延長と OPk
辺ABとの交点を Ah+B(1-h)
Qとするとき、Q=OPk=Ah+B(1-h)=(B/2+A/3)k=2A/5+3B/5
h=k/3,(1-h)=k/2
h=1-k/2=k/3=2/5
k=1/(1/3+1/2)=6/5

OQを
OA,OBで表せ。
277132人目の素数さん:2012/01/03(火) 23:42:58.92
260が解答します
278132人目の素数さん:2012/01/03(火) 23:44:23.66
俺達の>>260
279132人目の素数さん:2012/01/03(火) 23:45:48.94
>>276
質問の構成パクってるwwwww
280132人目の素数さん:2012/01/03(火) 23:48:00.32
260まだー
281132人目の素数さん:2012/01/04(水) 00:40:45.12
>>258
軸を考慮して場合わけするだけってことですか?
282132人目の素数さん:2012/01/04(水) 01:05:19.93
n/4 *(9n-57) が100を越えるときの最小の自然数nの値を求めよ。

二次不等式を解こうとするとものすごく時間かかってしまいます。
判別式が膨大な数になり、それを整数で近似するのも時間がかかります。
n(9n-57)/4 >100 を解け ではなく、満たす最小の自然数nを求めさせているあたりから、
もっと速い解法があるとおもうのですが。
283132人目の素数さん:2012/01/04(水) 01:07:53.05
{x!*(10-x)!}/{(x-1)!*(11-x)!}

こういう階乗だらけの式をきれいにしたいのですが、数学Aの教科書には使えそうな公式がありません。
何かよい解き方ありませんか?
284132人目の素数さん:2012/01/04(水) 01:10:48.22
n/4 *(9n-57)

n/(4 *(9n-57) )
(n/4) *(9n-57)
のどっちだよ

それと二次不等式でヴァカみたいに時間がかかるのはオマエのやり方が悪いだけかもしれないから、
二次不等式で解いた過程も書け
285132人目の素数さん:2012/01/04(水) 01:11:49.48
>>283
引くこと覚えろks
286132人目の素数さん:2012/01/04(水) 01:12:18.02
>>283
自分で計算できるだろカス
287132人目の素数さん:2012/01/04(水) 01:12:53.60
x! = x*(x-1)!
(11-x)! = (11-x)*(10-x)!
公式というより、定義にしたがって考えるといろいろ見えてくるのでは?
288132人目の素数さん:2012/01/04(水) 01:16:16.02
>>284
わかりやすくスペースを空けたつもりでした
n(9n-57)/4 >100
9n^2 -57n -400 >0

x<(19-√1961)/6 (19-√1961)/6 <x
よって最大の自然数11
289132人目の素数さん:2012/01/04(水) 01:16:59.75
>>284

下の文で書き換えてるだろ
290288:2012/01/04(水) 01:17:20.53
×最大の
○最小の
291132人目の素数さん:2012/01/04(水) 01:20:08.72
>>288
わかりやすいってお前はアホか
括弧一組も入れて説明できないから
こんなのも分からなくなるんだ

お前一人がオナニー自己満で分かってても他にゃわからんよ
292132人目の素数さん:2012/01/04(水) 01:21:39.57
>>284
二次不等式って書いてあるのに
分母に括弧がかかるわけがないだろwwww
293132人目の素数さん:2012/01/04(水) 01:23:27.78
>>292
お前もkuzuか?
問題に二次不等式って書いてないだろ。
解くヤツが勝手に二次不等式とか思いこんでるだけかもしれんだろ?
294132人目の素数さん:2012/01/04(水) 01:27:00.45
>>282
あるけど教えない
295132人目の素数さん:2012/01/04(水) 01:29:32.31
>>293
ゴチャゴチャ言ってるヒマがあればさっさと答えてやれよ
296132人目の素数さん:2012/01/04(水) 01:33:06.31
>>295
ゴチャゴチャてwww
括弧の場所はっきりさせるのはごちゃごちゃじゃねーだろ
意志疎通の基本だろ
お前は基本を捨てろっていうんか?
お前は基本すらできてない小学生レヴェルなのかよ
297132人目の素数さん:2012/01/04(水) 01:33:43.31
>>296
常識で判断しろよ
298132人目の素数さん:2012/01/04(水) 01:34:18.10
>>293
常識的に考えろよ。
見落としてたからって屁理屈いいすぎw


>>282
(n/4)(9n-57)=100
n(9n-57)=400 
nが増えるに従ってn(9n-57)はどんどん大きな値をとることが予測できる。
400になるのはそんな遠い話ではない。
9n-57>0となるのはnが7以上のとき。
8のとき 8*15 =120
9のとき 9*24=216
10のとき 10*33=330
11のとき 11*42=462

299288:2012/01/04(水) 01:35:20.07
>>298
ありがとうございます 参考にします。
300132人目の素数さん:2012/01/04(水) 01:35:52.25
AB=AC=1,∠A=90°である直角三角形ABCの辺AB,BC上にそれぞれ点P,Qをとり、AP=BQとする。2点P,Q間の距離が最小となるときのAPの長さを求めよ。
どう手をつけていいかわかりません。
どういう方針で解けばいいでしょうか?
301132人目の素数さん:2012/01/04(水) 01:37:03.00
>>297
>>298
常識ジョーシキって連呼ロボttおかよwww
見落としてたのはこっちじゃなくて質問者の方だろ?
そこにカッコつけろって言ってやったのになんで質問者のクズカスからすいませんの一言もねーんダよ
そんなんだからゴミの大量生産がとまんねーーーーーんだよ
お前らもその産廃のお仲間か?
302288:2012/01/04(水) 01:37:53.18
>>301
でお前なにしにきたの?
303132人目の素数さん:2012/01/04(水) 01:39:24.42
>>301
すいませーんw
304288:2012/01/04(水) 01:39:51.04
私は>>302ではないのであしからず。
305132人目の素数さん:2012/01/04(水) 01:40:09.15
>>287さん
ありがとうございました。ばっちり理解できました。
306288:2012/01/04(水) 01:40:37.88
>>303でもないです
307132人目の素数さん:2012/01/04(水) 03:15:26.05
重複組み合わせのnHrの説明で、「重複を許してr個取る組合せの数」とありますが、どういうことですか?
袋の中に、赤、白、黒の玉があってそれぞれ6個の全部で24個のとき、6個取り出すってのがそうだと思うのですが、同じ色を取っても1つとカウントするのが重複組み合わせで、24個の玉が全部違う種類の場合が24C6ってことですか?
308132人目の素数さん:2012/01/04(水) 08:00:13.79
>>300
求めるものを文字で置いて式を立てる
まずは座標設定でも余弦定理でもいいので PQ^2 を式にしよう
309132人目の素数さん:2012/01/04(水) 08:42:51.73
>>307
認識に誤りがある
次の例を考えよう
袋の中に,赤,白,黒の玉がそれぞれ十分にたくさんあるとき,
この中から色が重複することも許容して4個の玉を取り出すときに使うのが
重複組合せで,この例では
   赤赤赤赤,赤赤赤白,赤赤赤黒,…,黒黒黒黒
の15通りとなる
3種類の玉から4個を取り出すので, 3H4 と表す
ところで,この例では
   4個の○と 2 ( = 3-1 )本の仕切り棒を並べる
モデルに読み換えることができる
赤|白|黒 と対応させて,例えば
   ○○|○|○
と並んだときは,「赤2個,白1個,黒1個」と読む
この並べ方の総数は,同じものを含む順列( 或いは普通の組合せ )で立式できる
俺は H は使わずにこう考えることが多い
310132人目の素数さん:2012/01/04(水) 09:12:33.55
>>307
n種類から重複を許してr個とる組み合わせ。
3種類(ABC)から重複を許して2個とるのは、AA、AB、AC、BB、BC、CCの6通り。

> 同じ色を取っても1つとカウントするのが重複組み合わせで、24個の玉が全部違う種類の場合が24C6
後者はもちろんそれで合っているが、前者はいまいち意味が分からない。
> 赤、白、黒の玉があってそれぞれ6個の全部で24個のとき、6個取り出すってのがそうだと思うのですが
たしかにそれは重複組み合わせだが、24H6ではなくて3H6だということはわかってる?
6個ずつ計24個である必要は全くなく、100個ずつ計300個から6個とる場合でも
赤6個、白15個、黒100個計121個から6個とる場合でも同じ。
311132人目の素数さん:2012/01/04(水) 09:42:23.58
>>143の問題なのですが、>>171で教えていただいたように分数関数で考えてみました。
2a=(2x+1)/(x^+2).
x≠-1/2のとき、(与式)=1/{(x^+2)/(2x+1)}
分母はP(x^,2x^)とA(-1,-2)を通る直線の傾きを表し、Pは-1/3≦x≦2の範囲で動く。
ここまではわかったのですが、その次からどうすればいいのかがわからないです。
312171:2012/01/04(水) 10:13:01.84
>>311
P( 2x , x^2 ) でいいんだよな?
   X = 2x , Y = x^2
とおいて軌跡を考えれば, P の存在範囲がわかる
よって, AP の傾きのとり得る値の範囲も少し考えればわかる
だからそのやり方でもできるかもしれないが,あまり楽ではなさそう

   f( x ) = ( 2x + 1 ) / ( x^2 + 2 )
とおくと,数Vの微分法の公式で導関数を求めると
   f’( x ) = { −2( x + 2 )( x − 1 ) } / { ( x^2 + 2 )^2 }
となる
これをもとに -1/3 ≦ x ≦ 2 における f( x ) の増減を捉えて考えろ,というのが
>>171 での発言の内容である
313132人目の素数さん:2012/01/04(水) 10:36:36.39
>>311 の発想を生かすなら,( 2x + 1 ) / ( x^2 + 2 ) を
   B( -2 , -1 ) と Q( x^2 , 2x ) を通る直線の傾き
と見るとよい 本問についてはこの解法は成立し得る
>>312 で言ったようにして Q の存在範囲を捉えれば,
Q は横向きの放物線上にあることがわかるので…
314132人目の素数さん:2012/01/04(水) 10:38:07.52
上手く説明出来ないんだけど鈍角の三角比って実際は鈍角じゃないんじゃないかと疑問です
鈍角の三角形ってブーメランやハンガーみたいな形の三角形の事でしょう?
単位円を出して拡張って言ってるけどやってる事は180度-鈍角の余りの鋭角の三角比の様な…
それが何故鈍角の三角比を求めた事になるのかわかりません
315132人目の素数さん:2012/01/04(水) 10:43:54.82
>>312
返答ありがとうございます。
微分はまだ自学しだしたところで、書籍が数Uまでしかないので分数関数での導関数の求め方がわからないです。
なので、とりあえず今でもできそうなやり方を考えました。
整関数での微分なら基本的なことであればわかるのですが。
もう少しやり方を具体的に教えていただけますか。
316132人目の素数さん:2012/01/04(水) 11:08:21.88
>>315
>>311 の立式では傾きの範囲を求めたあとその逆数を考えることになりひと手間余計だから
>>313 のように見ることにすると, BQ の傾きの範囲が 2a の範囲になる
既に相当進んだことをやっているので,
軌跡の求め方がわからないということはないだろうが,
もし忘れているのなら数Uの本に出ているはずなので確認せよ
出てくる軌跡は放物線だから,接するときは重解条件で捉えることができる
(微分法でもできるが,ここではわざわざ微分法を使うまでもなかろう)
これ以上は答えを言うのと大して変わらないのでここまでにしておく
もう少し考えてみたまえ
317132人目の素数さん:2012/01/04(水) 11:16:38.93
>>315
分数関数の導関数公式を聞いているのか?
   { f/g }’ = { f’・g − f・g’ } / ( g )^2
318132人目の素数さん:2012/01/04(水) 11:20:03.69
>>314
それを鈍角の三角比と呼ぶことに決めたから。
三角比という名称を使うことに違和感があるってことなんだろうけど、
ごもっともとしか言いようがない。
でも、誰かが決めちゃってそれでいいことになっちゃったので今さら変える方が混乱する。

君は帯分数を例えば「さんとさんぶんのいち」のように読むように習ったと思うけど、
「さんかさんぶんのいち」と読むように習った者に言わせれば、「さんとさんぶんのいち」には違和感がある。
昭和45年頃だと思うが、今さら変えやがったために混乱した。
読み方だけでも混乱するので、用語自体の変更はもっと混乱するだろう。
319132人目の素数さん:2012/01/04(水) 11:23:38.37
>>314
君の頭が鈍だからだろう、鋭になるよう努力しなさい
320132人目の素数さん:2012/01/04(水) 11:50:07.10
ありがとうございます

>それを鈍角の三角比と呼ぶことに決めたから。
の「それ」とは以下の
>180度-鈍角の余りの鋭角の三角比
の事で良いのでしょうか
321132人目の素数さん:2012/01/04(水) 12:02:02.26
>>320
そだよ。それが正しい定義かどうかは知らんけど。
322132人目の素数さん:2012/01/04(水) 12:04:05.71
>>316
BQの傾きの範囲になるのは理解できるのですが、
「Q(2x,x^2)は、y=1/4x^2の-1/3≦x≦2における範囲を動く」という捉え方で間違いないでしょうか。
Bは固定なので、(-1/3,1/36)と(-1,-2)を通る直線の傾き、と、1/4x^2=p(x+1)-2を整理して、判別式=0となるp(傾き)を求めるという事ですよね。
そうすると、おそらく73/24≦(x^2+2)/(2x+1)≦1/2となり12/73≦a≦1/4となると思います。
しかし、正解は5/12≦a≦3/38となっています。正解f(),軸のやり方で答えを出していますが・・
323132人目の素数さん:2012/01/04(水) 12:49:17.30
>>309>>310
ありがとうございます。
赤、白、黒が6個ずつなので計18個ですね。勘違いしました。
逆に、重複を許さない組み合わせってのは、3種類の計18個の中から6個を取るとなっている時点でありえないということですかね?不安なので僕も○と|で考えるようにします。
324132人目の素数さん:2012/01/04(水) 12:49:26.49
>>322
訂正します
答えは3/38≦a≦1/2です
325132人目の素数さん:2012/01/04(水) 13:13:16.84
>>323
なんか全然分かってない気がする。

> 3種類の計18個の中から6個を取るとなっている時点でありえない
たしかにその通りだ。無理にその問題に答えるなら0通り。

> 不安なので僕も○と|で考えるようにします。
どういうものが重複組み合わせなのかということが理解出来ないと、
○と|で考えればうまくいく問題なのかどうかが分からないと思うのだが。
数学というより国語の問題。
326132人目の素数さん:2012/01/04(水) 13:23:14.33
1から5までの番号のついた球がそれぞれ1つずつあり、これら5つの球をA,B,C,Dのら4つの箱に入れるとき、次のような球の入れ方はそれぞれ何通りあるか。ただし、それぞれの箱には5つまで球をいれることができるものとする。
(1)4つの箱のどれにも球が入るような球のいれ方
(2)少なくとも1つの箱が空であるような球の入れ方
(3)Aの箱とBの箱に同じ個数の球が入るような球のいれ方
ただし、どちらの箱も空の場合は同じ個数とみなす。
(1)から早速つまずいてます。
4!と、残った球をA,B,C,Dに入れる4通りで、4!×4だと思ったのですが、なぜ間違いなんですかね(_ _)
327132人目の素数さん:2012/01/04(水) 13:38:15.06
>>326
最初の4!の球と残りの球を区別してしまっている
328132人目の素数さん:2012/01/04(水) 14:00:27.25
>>326
1番を入れる箱が4通り、2番を入れる箱が3通り……、最後に5番を入れる箱が4通って言う考えなんだろうけど、
それだと例えば1番と2番が同じ箱に入る場合を数えていない。
329132人目の素数さん:2012/01/04(水) 14:10:57.73
座標平面上で3点(0,0)(a,b)(c,d)で表される三角形の面積が1/2(ad+bc)で求まるのはなぜですか?
出来れば証明をいただきたいのですが面倒なら定理の名前や検索ワードだけでもいいのでお願いします
330132人目の素数さん:2012/01/04(水) 14:18:03.38
>座標平面上で3点(0,0)(a,b)(c,d)で表される三角形の面積が1/2(ad+bc)で求まるのは
求まりません
キーワードは「三角形の面積」、「外積」ですぐに見つかるでしょう
331132人目の素数さん:2012/01/04(水) 14:22:34.74
2ax^-2x+4a-1=0が x^-2x=(1-4a)/2a
-1/3≦x≦2の範囲に (3)^-2/3>(1-4a)/2a>2^-4
1/(2(3)^-2/3+4)<a<1/(2^-3+4)
332132人目の素数さん:2012/01/04(水) 14:30:02.21
>>329
S=(1/2)|ad-bc|
座標をとって底辺X高さまたは面積の公式
333132人目の素数さん:2012/01/04(水) 14:32:10.53
すみませんうろおぼえみたいで
ありがとうございます早速やってみます
ありがとうございました
334132人目の素数さん:2012/01/04(水) 14:48:28.58
S=.5axb
335132人目の素数さん:2012/01/04(水) 15:06:56.24
>>322
考え方は大体合っているが, x 座標と y 座標が逆になってない?
Q( x^2 , 2x ) の軌跡を求めたいので
   X = x^2 , Y = 2x
とおく
これで軌跡を求めると,「横向きの」放物線が出てくるはず
Y の範囲は x の範囲から出てくる
336132人目の素数さん:2012/01/04(水) 15:25:29.56
A,B,Cの3組にわける場合と2枚ずつ3組にわける場合の違いがチャートを読んでもわかりません。
さらにわかりやすく解説してもらえないでしょうか(T T)
337132人目の素数さん:2012/01/04(水) 15:52:38.59
>>335
本当だ。知らないうちにひっくり返していました・・
横向きの放物線は初めてなので手こずりましたが、x=1/4y^2という捉え方であっているでしょうか。
そうするとB(-2,-1)とQ{(1/3)^2,(2・1/3)},{2^2,2・2}つまりQ(1/9,2/3),(4,4)の傾きを求めればいいですか。
338132人目の素数さん:2012/01/04(水) 17:41:32.22
三角比を拡張すると三角形は関係なくなるんですか?
339132人目の素数さん:2012/01/04(水) 17:51:46.88
かんけーねーです
円になります
340132人目の素数さん:2012/01/04(水) 17:54:33.08
>>336
それは何を分けてるの?区別できるもの?
区別できないものなら、A,B,Cに分けるときは、組に名前がついてるから全て区別する
2枚ずつに分けるときは、2枚、2枚、2枚で区別できないから3!で割る
341132人目の素数さん:2012/01/04(水) 18:32:42.18
>>339
ありがとうございます
342132人目の素数さん:2012/01/04(水) 19:09:18.30
>337
また間違えた
Q(1/9,-2/3)とB(-2,-1)を通る直線の傾き それとBから横向きの放物線への接線の傾きを出す、ということですか
343132人目の素数さん:2012/01/04(水) 21:09:19.96
|c-1/2|^2 = |c-(1+2i)|^2

の答えが c=4/19 になるみたいなのですが、たどりつけません
解答では

(c-1/2)^2 = (c-1)^2 + (-2)^2

と計算しているのですが、このようになる理由も分かりません

ちなみにcは実数です
よろしくお願いします
344132人目の素数さん:2012/01/04(水) 21:13:29.43
>>343
>このようになる理由も分かりません
|z|で距離、|z|^2で距離のようなもの、

z = a + ib なら距離rは
r = |z| = √(a^2 + b^2)
345132人目の素数さん:2012/01/04(水) 21:13:57.35
>>343
定義を知らなきゃ出来るわけない。
346132人目の素数さん:2012/01/04(水) 21:18:33.52
2等辺三角形の頂点。
347132人目の素数さん:2012/01/04(水) 21:34:04.97
>>344-346
なるほど…。図を書いてみたら理解することが出来ました。
ありがとうございました。
348132人目の素数さん:2012/01/04(水) 21:36:39.16
>>343
> (c-1/2)^2 = (c-1)^2 + (-2)^2
349132人目の素数さん:2012/01/04(水) 21:40:15.45
>>348
?
350132人目の素数さん:2012/01/04(水) 21:44:12.33
>>349
351132人目の素数さん:2012/01/04(水) 21:48:06.79
>>350
!
352132人目の素数さん:2012/01/04(水) 21:56:45.86
>>348
>>350
> |c-1/2|^2 = |c-(1+2i)|^2
> ちなみにcは実数です
353132人目の素数さん:2012/01/04(水) 22:21:12.51
>>342
そういうこと
図から傾きのとり得る値は >>342 の2つの直線の傾きの間になることがわかるので
その両者の傾きを求めればおk
354132人目の素数さん:2012/01/04(水) 23:55:02.12
>>353
できました!場合わけもいらないですし、この解き方はかなり使えそうです。
いろんな考え方が身について良かったと思います。相手をしていただきありがとうございます。
全統模試でもこのような考え方をどんどん使っていけそうです。
ただ、やっていて思ったのですが、このやり方だと直接的に範囲を求めていますがf(),軸とかのやり方は、感覚的にですが間接的な感じがするし、アバウトなのではないかと思います。
たとえば、y=x^2-2mx+2m+3(mは実定数)とするとき、x軸の正の部分とは共有点をもたず2点A(0,3)B(3,3)と共有点を持つようなmの範囲を求めよ、
という問ですが、文字定数を分離して考えると、3/4≦m<3になります。
解答では、2≦m<3ですが、実際にm=3/4を代入すると(判別式)<0となるので、あっていると思います。
どう思われますか?
355132人目の素数さん:2012/01/05(木) 00:57:49.50
>>354
「分数式を傾きと見る」という定石が解の配置に応用できるとは
思っていなかったので,俺自身も勉強になった
ただ,これが一般的に応用可能かどうかは
他の問題で確認しておいたほうがよい
今回は「定点を通過する直線の傾き」と見ることができたから
たまたまうまくいっただけかもしれないので

ところで, >>354 の問題だが,線分 AB と共有点をもつということか?
m = 3/4 として確かめたら,線分 AB にはかすりもしなかったが…
何か勘違いをしていそうなので,正確な問題文と君の解法を提示してほしい
356132人目の素数さん:2012/01/05(木) 01:16:54.89
xの不等式x^2-(a+1)x+a<0を満たす整数xがただ1つだけあるような定数aの値の範囲を求めよ。
xがただひとつだけあるってのはグラフで表すとどういう状態なんですか?
下に凸の放物線てまxがただ1つっていうのが意味わかりません。
357132人目の素数さん:2012/01/05(木) 01:23:02.48
>>356
xは整数です
あまり調子乗らないように
358132人目の素数さん:2012/01/05(木) 01:23:59.76
線分AB(両端も含む)と共有点をもつということです。
何となく解けるんじゃないかなーと思いついた解き方なのですが・・
与式を変形して、2m=(x^2+3)/(x-1) つまり2mはA(1,-3) P(x,x^2)を通る直線の傾きである
ここで、Aと(3,3)を通る直線の傾き、Aから放物線への接線の傾き(等号は成り立たない)を計算したら
答えに結構近い感じのが出たので、あってるのではないか・・という感じです
359132人目の素数さん:2012/01/05(木) 01:25:51.27
>>357
自己解決しました。
x軸で交わる範囲を、整数をただ一つにしろってことですね。そのためにaを定義しなくてはいけない問題ということですね。
360132人目の素数さん:2012/01/05(木) 01:27:18.03
>>355
お前きもい
361132人目の素数さん:2012/01/05(木) 01:30:55.31
>>355
確かに、m=3/4はABと交わらないですね。
m<3が一致したのはは偶然かな。
これは、場合わけでf(),軸、判別式しかないのか・・
362132人目の素数さん:2012/01/05(木) 01:32:17.20
>>354
先ず問題を正確に一文字も変えることなくそのまま書き写せ。
m=0が入ってないのはおかしいだろ。
363132人目の素数さん:2012/01/05(木) 01:32:29.22
√(6+√(6+√(6+•••は3、
√(1+√(1+√(1+•••は黄金比になる事は有名ですが
√(1+√(2+√(3+√(4+•••
みたいな根号の中身が等差数列になるものはどんな値になるんでしょうか
そもそも収束しますか?
364132人目の素数さん:2012/01/05(木) 01:35:59.98
>>363
しない
365132人目の素数さん:2012/01/05(木) 01:38:00.79
>>364
ありがとうございました
366132人目の素数さん:2012/01/05(木) 01:42:08.57
>>358
>>354 の問題では与方程式に 「 y も 」 含まれている
これを無視して変形するのはまずい
ところで,問題文には 「 m は正 」 という条件もあったのでは?

>>360
専ブラで「,」をNGワードに登録せよ
367132人目の素数さん:2012/01/05(木) 01:44:59.90
>>363
収束する。
368132人目の素数さん:2012/01/05(木) 01:45:18.46
>>364
いやするだろ 値はわからんが
369132人目の素数さん:2012/01/05(木) 01:45:42.28
センターの問題です。
2次関数f(x)=x^2+ax+3について、次の条件を満たすような定数aの値の範囲を求めよ。
すべてのxに対し、a≦f(x)が成り立つ。

これはa≦x^2+2ax+3の二次関数の最小値を求めて、もとめたそれが≧0だとする問題ですか?よくわかりません。
370288:2012/01/05(木) 01:48:01.96
f(x)>=a
f(x)-a >=0
x^2 +ax +3 -a >=0
左辺の二次関数の最小値を求め、それが>=0となればよい。
その考えでいいのでは?
371132人目の素数さん:2012/01/05(木) 01:48:13.46
>>367
そうなんですか
具体的に求められませんか?
372132人目の素数さん:2012/01/05(木) 01:52:32.49
>>366
mは実定数という条件だけですが、前の問でx軸が正の部分と共有点をもしないようなmの値の範囲を求めていて、その範囲は-3/2≦m<3となっています。
3/4≦m<3はそれも満たしているので、もう少しで正解にたどり着けたような気がするのですが、どうすれば良かったでしょうか。

373132人目の素数さん:2012/01/05(木) 01:53:08.64
>>370
ありがとうございます。
すごい簡単そうに説明しますね。数学得意そうでうらやましいです
374132人目の素数さん:2012/01/05(木) 01:59:10.65
>>372
問題文を全文、正確に書き写して下さい。
質問する側の解釈、説明は無用です。
375132人目の素数さん:2012/01/05(木) 02:26:48.39
2次関数f(x)=x^2+ax+3について、次の条件を満たすような定数aの値の範囲を求めよ。
-2≦x≦2であるすべてのxに対しf(x)≧aが成り立つ。
>>369の続きなんですが、f(x)≧aを平方完成したら、{x+(a/2)}^2-(a^2/4)-a+3となりました。
軸の-(a/2)の位置で場合わけが必要なわけですが、
[1]-(a/2)<-2のとき不適
[2]-2≦-(a/2)≦2のとき-4≦a≦2
[3]2<-(a/2)のとき-7≦a<-4
となりました。ここまではいいのですが、
[1][2][3]から求める範囲が-7≦a≦2と答えはなってます。
ぼくは共通部分の-6≦a<-4だと思ったのですがどういうことですか?http://beebee2see.appspot.com/i/azuYvPq0BQw.jpg
376132人目の素数さん:2012/01/05(木) 02:32:52.14
なんで共通部分なんだ?
それに-6なんて何所から出てきた?
377132人目の素数さん:2012/01/05(木) 02:35:57.55
>>376
すいません、勘違いでした!
こうですね!http://beebee2see.appspot.com/i/azuYzci8BQw.jpg
378132人目の素数さん:2012/01/05(木) 02:40:38.73
赤ちゃんはどうしたらできるの?
379132人目の素数さん:2012/01/05(木) 02:41:12.96
380132人目の素数さん:2012/01/05(木) 03:20:43.31
f(n)=f(n+f(n+1))
x^2-1=x
x^2-6=x
381132人目の素数さん:2012/01/05(木) 04:17:12.84
大小2個のさいころを投げるとき、次の確率をもとめよ。
(1)異なる目がでる
(2)目の和が6以下になる
(3)目の和が4の倍数になる
(1)30/36
(2)15/36
(3)9/36
看護学校の過去問なんですが、解答がありません。どうでしょうか?
382132人目の素数さん:2012/01/05(木) 04:17:51.97
>>381
約分してません
383132人目の素数さん:2012/01/05(木) 11:52:24.08
白チャートの 反復試行と点の移動 の問題の

1*r+(-1)*(4-r)の1*r+(-1)の意味を教えてください。
384132人目の素数さん:2012/01/05(木) 11:58:19.80
意味がわかりません。
あと接続詞に「の」使いすぎです。文章構造を見なおすようにしましょう。
385132人目の素数さん:2012/01/05(木) 12:08:48.31
ってか、1*r+(-1)*(4-r)から1*r+(-1)だけ抜き出す思考が理解出来ない。
386132人目の素数さん:2012/01/05(木) 12:09:51.04
>>1
> ・質問者は回答者がわかるように問題を書くようにしましょう。でないと放置されることがあります。
>   (変に省略するより全文書いた方がいい、また説明なく習慣的でない記号を使わないように)
> ・質問者は何が分からないのか、どこまで考えたのかを明記しましょう。それがない場合、放置されることがあります。
>   (特に、自分でやってみたのにあわないので教えてほしい、みたいなときは必ず書くように)
387132人目の素数さん:2012/01/05(木) 12:20:22.57
>>383です
原点Oから出発して数直線上を動く点がある。硬貨を投げて、表が出れば
正の方向に1だけ進み、裏が出れば負の方向に1だけ進むものとする。
(1)硬貨を4回投げた時、原点に戻る確率を求めよ。

という問題を解く式で1*r+(-1)*(4-r)の1*r+(-1)の意味が分からないので
分かる方教えてください。
388132人目の素数さん:2012/01/05(木) 12:23:01.32
>>387
なんで全部書かないの?
389132人目の素数さん:2012/01/05(木) 12:25:16.11
まずrの定義がなんなのか分からないよ( ´゚д゚)(゚д゚` )ネー
390132人目の素数さん:2012/01/05(木) 12:29:28.05
>>389
表が出た回数がrです。すいません。
391132人目の素数さん:2012/01/05(木) 12:30:21.36
>>387
rは表が出た回数か?
1*rが表が出たことにより正の方向に進んだ距離。
(-1)*(4-r)の4-rは裏が出た回数で、-1は負の方向に1進むというのを正の方向に-1進むと考えたもの。
(-1)*(4-r)全体で裏が出たことにより正の方向に進んだ距離。
両者を足した1*r+(-1)*(4-r)が硬貨を4回投げて表がr回だったときに点がいる位置。

すでに指摘されているが、なぜ1*r+(-1)*(4-r)から1*r+(-1)を抜き出しているのかわからない。
392132人目の素数さん:2012/01/05(木) 12:31:17.40
4回コインを投げてr回おもてがでたとすると、4−r回裏がでる。
r回表がでた事による、変移は (+1)×r
4−r回裏がでた事による、変移へ (−1)×(4−r)
でトータルの変移は、その和
393132人目の素数さん:2012/01/05(木) 12:31:13.05
>>390
いいから全部書けよ。
推定して解説したけど、不愉快だよ。
394132人目の素数さん:2012/01/05(木) 12:32:44.40
1*r+(-1)*(4-r) は4回投げたときの点の座標を表している
//(4-r)は裏の出た回数
この問いではそれが原点であるので
1*r+(-1)*(4-r) = 0
これを解いてr=2
395132人目の素数さん:2012/01/05(木) 12:37:28.96
>>391
なんでってその部分が分からなかったから
>>393
解説ありがとうwww
396132人目の素数さん:2012/01/05(木) 12:39:26.13
質問を小出しにするカスには答えなくていいのに
甘やかす方が悪いわな
397132人目の素数さん:2012/01/05(木) 12:39:29.62
>>391
>>392
>>394
解決しました。御手数掛けましたが本当ありがとうございました。
398132人目の素数さん:2012/01/05(木) 12:41:59.47
>>396
スレタイ
399132人目の素数さん:2012/01/05(木) 12:43:32.20
>>398
あぁ?それがどうした
400132人目の素数さん:2012/01/05(木) 12:44:07.77
自分の能力をひけらかすスレですから(棒)
401132人目の素数さん:2012/01/05(木) 12:45:14.22
簡単な問題だとどえらい勢いで回答がつくw
402132人目の素数さん:2012/01/05(木) 12:46:23.98
まあ伸びて良かったじゃん。冬休みなのに人少ないし
403132人目の素数さん:2012/01/05(木) 12:47:39.60
解かずに流れでレスするやつもいる
404132人目の素数さん:2012/01/05(木) 12:49:23.30
高校生が集まるスレなんてそんなものよ。気にしちゃダメ
405132人目の素数さん:2012/01/05(木) 15:26:34.02
お○んこ
406132人目の素数さん:2012/01/05(木) 16:01:54.95
次の不等式を解きましょう。
3x^2-4x+2>0

解の公式を使うとルートの中が負になる→実数の解を持たない→x軸と交わらない。
と書いてあるのですが、なぜルートの中が負だとx軸と交わらないのですか?
407132人目の素数さん:2012/01/05(木) 16:07:06.84
実数の解を持たないから
408132人目の素数さん:2012/01/05(木) 16:13:51.83
ルートの中が負ということは、その値は虚数であって実数でないということ。
解の公式で求めた解は
3x^2 -4x +2 =0 の解でしょ?
y=3x^2 -4x +2
y=0 (x軸)
この二つのグラフの交点のx座標が虚数解であるということは
実数の範囲で交点をもたないということ
409132人目の素数さん:2012/01/05(木) 16:14:58.90
ここの人の説明難しすぎて
なに言ってるかわからない
410132人目の素数さん:2012/01/05(木) 16:16:06.30
それは日本語の能力に問題アリだ
411132人目の素数さん:2012/01/05(木) 16:17:39.35
たしかに説明する人の
日本語能力に問題がありそうですね
412132人目の素数さん:2012/01/05(木) 16:48:56.59
>解の公式を使うとルートの中が負になる→実数の解を持たない→x軸と交わらない
平方完成してはいけない、とかの縛りルール?
田舎のバカ教師とかはありがちだよw
413132人目の素数さん:2012/01/05(木) 17:07:53.47
>>406
自分で「→実数の解を持たない→」と書いてるじゃないか

関数f(x)と曲線y=f(x)について
@f(x)の値が0になる
A曲線上の点のy座標が0になる
B曲線がx軸と交わる

この3条件が同じことを言っているのはわかるか?
414132人目の素数さん:2012/01/05(木) 17:07:53.64
平方完成しようが一緒だろ
415132人目の素数さん:2012/01/05(木) 17:23:58.37
平方完成してグラフを描いて確認してもいいけど
判別式を理解しないまま進むことに問題がある
416132人目の素数さん:2012/01/05(木) 18:06:57.52
本人はどこ行った?
417にょろ〜ん♂( 忍法帖【Lv=40,xxxPT】 ):2012/01/05(木) 19:20:14.46
虚数って 変な数 √i は実部があるし虚部もある 
オイラ〜の公式から必ず実部も虚部も持つ なのに虚数と実数と複素数を分けている不思議
418132人目の素数さん:2012/01/05(木) 19:24:36.50
虚数にロマンを持っているんだ
若いってうらやましいw
419132人目の素数さん:2012/01/05(木) 19:40:19.36
虚数にロマンってなんかヒッキーっぽいなあw
420132人目の素数さん:2012/01/05(木) 20:03:11.11
積分で面積が出せることの図形的なイメージが出来ない
421132人目の素数さん:2012/01/05(木) 20:04:06.67
区分求積
422132人目の素数さん:2012/01/05(木) 20:09:38.22
素数は分数で割れませんか
423132人目の素数さん:2012/01/05(木) 20:10:18.87
むしろ面積出すための計算が積分じゃないのか?
そこからいろいろ拡張されてんじゃないの?
424132人目の素数さん:2012/01/05(木) 20:10:24.89
割れます
425132人目の素数さん:2012/01/05(木) 20:10:42.50
>>422
割れないかも知れないと思う理由を聞こうか。
426132人目の素数さん:2012/01/05(木) 20:12:37.98
>>422
> 素数は分数で割れませんか
「割る」の意味は?
427132人目の素数さん:2012/01/05(木) 20:12:46.88
やっぱり割れるんですね。
ありがとうございました。
428132人目の素数さん:2012/01/05(木) 20:13:36.27
>>427
なぜなっとくする?
429132人目の素数さん:2012/01/05(木) 20:31:54.08
(x^2-2x)(x^2-2x-2)-3を因数分解せよ。

(x^2-2x+3)(x^2-2x-1)であってますか?
430132人目の素数さん:2012/01/05(木) 20:33:03.33
>>429
×
431132人目の素数さん:2012/01/05(木) 20:34:34.84
>>430
(x^2-2x+3)(x-2)(x+1)ですか?
432132人目の素数さん:2012/01/05(木) 20:37:03.09
>>431
おしい
433132人目の素数さん:2012/01/05(木) 20:37:21.68
立方体を、辺を切り開いて展開図にするには、7本の辺を切り開くことになりますが、
いま立方体の12本の辺から無作為に異なる7本の辺を選んで切り開くとき、
立方体の展開図ができる確率はいくらでしょうか。
434132人目の素数さん:2012/01/05(木) 20:38:23.41
>>431はミスです
(x-3)(x+1)(x-1)^2ですね!
435132人目の素数さん:2012/01/05(木) 20:39:24.72
>>434
436132人目の素数さん:2012/01/05(木) 21:03:31.26
>>433
立方体の展開図は11通り

では12本の辺を無作為に7本選ぶのは何通り?
437132人目の素数さん:2012/01/05(木) 21:05:50.61
底面の直径と高さが2aの円錐がある。この円錐の頂点と底面の円周に外接する球を考える。
問、球の半径を求めよ

円錐の断面(三角形)を見ると、球が平面で見るとその三角形に対する外接円になる。なので、その外心と三角形の一辺との距離がわかればあとは三平方の定理で求まると考えました。
が、ダメでしたので解法を教えて下さい
438132人目の素数さん:2012/01/05(木) 21:06:00.72
>>436
C[12,7] = 792通りです。
じゃ答は11/792 = 1/72 ってことでそか?
439132人目の素数さん:2012/01/05(木) 21:11:55.02
>>437
それでなんで出来ないんだ?

> 円錐の断面(三角形)を見ると、球が平面で見るとその三角形に対する外接円になる
その外接円が球の大円であることを言わないとダメだとは思うが。
440132人目の素数さん:2012/01/05(木) 21:13:42.01
>>438
11通りって展開図の形で分類した場合じゃないか?
よく考えてないけど。そんなに少ないとは思えないが。
441132人目の素数さん:2012/01/05(木) 21:18:46.09
>>439
外心との距離を文字で置いて求めようとしてもどうでもいいよくわからない値が出て来てしまって・・・
442132人目の素数さん:2012/01/05(木) 21:24:59.96
>>438


11種さがすのは手作業になるけど
ttp://www.shinko-keirin.co.jp/kosu/mathematics/kirinuki/kirinuki24.html
このへんが面白いかもね
443132人目の素数さん:2012/01/05(木) 21:27:57.12
>>441
円錐の頂点をA、三角形のA以外の頂点をB、C、円錐の底面の中心をD、三角形の外心をO、円の半径をrとして、
△BODで三平方の定理。BO=r、BD=a、OD=2a-r。
どういう立式をしたんだ?
444132人目の素数さん:2012/01/05(木) 21:28:06.72
まて、反転と回転で22通りって考え方もあるな
445132人目の素数さん:2012/01/05(木) 21:30:25.06
>>442
○じゃないだろ
446132人目の素数さん:2012/01/05(木) 21:31:05.56
>>444
いや、全然違うだろ。もっとずっと多い。
447444:2012/01/05(木) 21:34:40.21
11通りに回転4方向と裏表反転も含めるのか

11*4*2=88?
448132人目の素数さん:2012/01/05(木) 21:40:20.74
>>443
A、Dは一直線上にあるがOはそこには無いと勝手に勘違いをしてそこから抜け出せなくなっていたみたいです

ありがとうございます
449132人目の素数さん:2012/01/05(木) 21:47:57.33
□■□□  ■■□□  ■■■□□
■■■■  □■■□  □□■■■
□□■□  □□■■

たとえな↑なんか回転は2通りしかないと思うんだけど、その減算はどうするんだ?
450132人目の素数さん:2012/01/05(木) 21:52:12.81
>>433の問題実はすごく難しいんじゃね
451132人目の素数さん:2012/01/05(木) 21:54:27.42
>>449の減算は考えなくていいだろ、立方体の展開図が出来ればいいんだから

むしろ、切り込んだ全てのパターンと展開図にならないパターンの差が展開図の総数って考え方じゃないの?
452132人目の素数さん:2012/01/05(木) 21:59:45.00
aを0<a<x/2を満たす定数とする。
x^2+(y+sina)^2≦1
y≦(tana)x-sina
の表わすxy平面上の図形をDとする。図形Dをx軸の周りに回転させてできる
立体の体積を求めよ。

交点の座標は(cosa,0),(-cosa,-2sina)
y=(tana)x-sinaを回転させた三角錐の体積は8/3π(sina)^2acosa
y=-sina±√(1-x^2)
これを回転させた立体の体積を求めたいのですがどのように
考えればいいのかわからないので教えてください。

453132人目の素数さん:2012/01/05(木) 22:00:25.61
一つだけ教えて下さい
三角関数でsinθ=yになるのは単位円の半径が1だからですか?
sinθ=y/1=y
って事なんでしょうか
454132人目の素数さん:2012/01/05(木) 22:01:15.76
だよ
455132人目の素数さん:2012/01/05(木) 22:03:48.24
>>447
何を言ってるんだ?
456132人目の素数さん:2012/01/05(木) 22:05:42.78
>>449
回転とかじゃないだろ、この問題の場合は。
それぞれの形になる切れ目の入れ方が形ごとに何通りあるのかを考えないとダメなんじゃないか?
457132人目の素数さん:2012/01/05(木) 22:07:23.41
例えば
■□□□
■■■■
■□□□
は24通りあるんじゃないか?
458132人目の素数さん:2012/01/05(木) 22:08:37.61
ここは高校生の質問スレです
459132人目の素数さん:2012/01/05(木) 22:08:59.67
>>454
ありがとうございます
460132人目の素数さん:2012/01/05(木) 22:09:47.69
>>453
質問が意味不明ですよ。
もしかして、x^2+y^2=1のとき、
y=sinθとおける
ってのとぐちゃまぜにしてないかい?
461132人目の素数さん:2012/01/05(木) 22:11:29.12
age
462132人目の素数さん:2012/01/05(木) 22:13:17.52
そもそも11通りが本当に全部のパターンで7回切り込んでいるのかを検証する必要があると思うが
463132人目の素数さん:2012/01/05(木) 22:20:57.54
展開図ができたなら
切断回数は必ず7回でしょ
464132人目の素数さん:2012/01/05(木) 22:26:28.16
>>460
三角関数じゃなくて三角比の拡張のところでした
単位円が導入されてsinθ=y/r(rは円の半径)と定義されるところでr=1ならsinθ=yに単純化されると書いてあるんですが
これは半径が1以外ではあり得ないのか確認したかったんです
465132人目の素数さん:2012/01/05(木) 22:27:53.62
ttp://www.pag1u.net/network/images/tenkaizu.png
この図で考えてみた
回転とか反転に関係なく、1から6までの配置の組み合わせの総和でどう?
466132人目の素数さん:2012/01/05(木) 22:30:43.07
>>452
> aを0<a<x/2を満たす定数とする。
467465:2012/01/05(木) 22:31:49.75
だめだったorz
7920通りになっちまう
ナシにしてくれ
468132人目の素数さん:2012/01/05(木) 22:33:14.57
三角比の問題で
http://Squid.axfc.net/uploader/Img/l/4309208532/v/Img_133093.jpg
なんですが、分からなくて解説を見たら
http://Salmon.axfc.net/uploader/Img/l/6426828943/v/Img_133096.jpg
http://Whitebait.axfc.net/uploader/Img/l/1396162632/v/Img_133097.jpg
となってました
cについて整理すると有りますが途中の式が分かりません
自分がやると(b-a)……になります
よろしくお願いします
469132人目の素数さん:2012/01/05(木) 22:38:14.94
12本中7本選ぶって
12*11*10*9*8*7*6=3991680通りじゃないの?

結果として同じ切り方であっても切り口が違えば違う形とみなすのか、そうでないのかが読み取れない
470132人目の素数さん:2012/01/05(木) 22:41:37.69
>>464
あー、俺の読解能力が足りんかったわw
その通りだよ。
471132人目の素数さん:2012/01/05(木) 22:46:22.93
>>464
ちなみに
y=sinθとおいたとき、
(x,y)は単位円を表すと言った方が分かりやすいかも?
まぁ、どうでもいい話だけど
472132人目の素数さん:2012/01/05(木) 22:48:17.48
訂正
ちなみに
x=cosθ,y=sinθ
とおいたとき、
(x,y)は単位円を表すと言った方が分かりやすいかも?
まぁ、どうでもいい話だけど

473132人目の素数さん:2012/01/05(木) 23:01:27.25
http://detail.chiebukuro.yahoo.co.jp/qa/question_detail/q1346871373
このページの問題についてです。解法が二つ紹介されているのですが、どちらも間違っているような気がします。

1つ目の解法について
偶数人の名前が書かれたカードは最終的には入れ替わらないので、「束がそのまま入れ替わ」るということにはならないですよね。

2つ目の解法について
A,Bを人物の名前として、例えば[AB]をAの名前とBの名前が書いてあるカードとしたとき
左のカードを[A],[B]、右のカードを[AB] とすると
「少なくとも1つの人名pについて, L(p) > R(p) が成り立」つという状況にはならないですよね。


他にこの問題が紹介されているページを見つけられず、正しい解法がわからなくて困っています。
お願いします
474132人目の素数さん:2012/01/05(木) 23:05:50.47
>>468
その参考書は正確にはCで整理するんじゃなくて-Cで整理してる
(b+c)(b^2+c^2-a^2)=(c+a)(c^2+a^2-b^2) ← ここまではいいんだよね

b^3+bc^2-a^2b+b^2c+c^3-a^2c=c^3+a^2c-b^2c+ac^2+a^3-b^2c ← 展開

b^3+bc^2-a^2b+b^2c+c^3-a^2c-c^3-a^2c+b^2c-ac^2-a^3+b^2c=0 ← 移項

c^2(b-a)+2c(b^2-a^2)+(b^3+ab^2-a^2b-a^3)=0 ← cについて整理する

このあと両辺に-1をかけて括弧の中の足し算引き算を反転させている
475132人目の素数さん:2012/01/05(木) 23:08:12.59
Nを1以上の整数とする。長さ4Nのひもがあり、
長さ1で等間隔に4N+1個の目印がついている。
ただし、両端の2つの目印は黒色で他の目印は白色する。
白色の目印から2つ選びこれを2頂点、両端の黒色の目印を重ねあわしてこれを3頂点とする三角形を作る。
このときの三角形ができる確立を求めよ。


この問題において答えは実験から推測できるのですが、解き方がいまいちよくわかりません
476132人目の素数さん:2012/01/05(木) 23:12:14.45
>>474
分かりました
ありがとうございます
477132人目の素数さん:2012/01/05(木) 23:28:22.52
連立方程式で答えがないときどうすればいいの
478132人目の素数さん:2012/01/05(木) 23:29:05.27
それを満たす解は無いのでしょう
479132人目の素数さん:2012/01/05(木) 23:29:09.04
解なし
480132人目の素数さん:2012/01/05(木) 23:39:42.37
>>475
紐の置き方によるんだろうけど、確率を求めるのは難しそう
三角形にならないのは三点が同一直線上にあるとき
紐が交差してもいいなら重なった二点を選ぶ時も三角形にならない
481132人目の素数さん:2012/01/05(木) 23:42:47.02
>>480
多分そういうことではなくて
この問題は三角形の成立条件を絡めた整数問題みたいなものだと思うのですが・・・
482132人目の素数さん:2012/01/05(木) 23:43:10.74
>>480
2点決めたら両端合わせるから同一直線上が三角形できないのじゃなくね?
3辺の長さa,b,cで|a-b|<c<a+bじゃね?
483132人目の素数さん:2012/01/05(木) 23:50:19.41
数直線上に紐置いて 座標設定して
三角形の成立条件絡めたけど
詰まってやめた
484132人目の素数さん:2012/01/05(木) 23:52:45.59
>>475
> このときの三角形ができる確立を求めよ。
確率1で「確立」しない
485132人目の素数さん:2012/01/05(木) 23:57:13.86
>>484
N=2とすると普通に 三辺の長さそれぞれ2、3、3で成立するのですが・・・
486132人目の素数さん:2012/01/05(木) 23:57:36.71
異なる三点が同じ直線上にないときその三点を結べば三角形になるって知ってる?
487132人目の素数さん:2012/01/05(木) 23:59:49.59
>>485
はあ
488132人目の素数さん:2012/01/06(金) 00:02:27.23
>>433
次のように考えたがどうだろう

はさみを1回入れると辺が1本増えることに着目する

はさみを7回入れて6面すべてが繋がっていてかつ
立方体のかどが残るような(つまり平面に展開できない)切り方は存在しない
 ∵ 3枚の連結した面でかどを残すような図形の辺は9本
  これに残りの面を付けると,できる図形の辺は18本
  つまりはさみを6回入れただけの図形になるので

はさみを7回入れた結果,2つ以上の図形に分かれる場合を考える
n個の面が連結してできる図形を「n面」と呼ぶことにする

(1)1面と5面に分かれる:まず1面を切り取り,次に5面に3回はさみを入れる
   6×( C[ 8,3 ] − 4 ) = 312 通り
(2)1面と1面と4面に分かれる:まず2面を切り取り,この2面を2つに分ける
   C[ 6,2 ] = 15 通り
(3)2面と4面に分かれる:まず2面を切り取り,次に4面に1回はさみを入れる
   C[ 6,2 ]× 5 = 75 通り
(4)3面(9本の辺)と3面(10本の辺)に分かれる:
  立方体のかどが残る3面を2つつくり,一方に1回はさみを入れる
   8×3 = 24 通り
計 426 通り

全事象は C[ 12,7 ] = 11・9・8 通り なので,
求める確率は
   1 − 426/( 11・9・8 ) = 61/132
489132人目の素数さん:2012/01/06(金) 00:02:41.62
200以下の自然数のうち、次のような数の関数を求めよ。
4または6の倍数
4で割り切れるが6で割り切れない
それぞれ67,17となりましたが添削お願いします。
490132人目の素数さん:2012/01/06(金) 00:05:45.73
>>489
まず関数じゃないじゃん
491132人目の素数さん:2012/01/06(金) 00:06:15.06
>>489
X
492132人目の素数さん:2012/01/06(金) 00:07:26.81
>>490>>491
ごめんなさい!個数の間違いです!!
ちなみに関数だと答えはなんですか?
493132人目の素数さん:2012/01/06(金) 00:07:48.57
>>492
はぁ?
494132人目の素数さん:2012/01/06(金) 00:08:16.43
>>492
関数の意味を調べておいで
495132人目の素数さん:2012/01/06(金) 00:10:24.07
次の方程式および不等式を解け。ただし0°≦θ≦180°
(1)2cosθ+√3
(2)2sinθ-1≧0
(1)θ=150°(2)30°≦θ≦150°
解くってこういうことですかね?また、あってますか?
496132人目の素数さん:2012/01/06(金) 00:10:54.49
>>489
計算過程は?
497132人目の素数さん:2012/01/06(金) 00:11:46.25
>>495
(1)は=0とか抜けてないか?
498132人目の素数さん:2012/01/06(金) 00:12:19.88
>>497
抜けてました。
499132人目の素数さん:2012/01/06(金) 00:13:22.22
>>475
それ答えある?なんか1/4になったんだけど

つーか誰か>>473頼みます
500132人目の素数さん:2012/01/06(金) 00:13:57.80
>>495
(1)θ=30°は抜けてないか?
501132人目の素数さん:2012/01/06(金) 00:15:10.80
>>496
n{x|xは200以下の4の倍数}+n{x|xは200以下の6の倍数}-n{x|xは200以下の12の倍数}
と、
n{x|xは200以下の4の倍数}-n{x|xは200以下の6の倍数}です。
502132人目の素数さん:2012/01/06(金) 00:15:55.72
>>475
二点をk、lと指定して三角形の成立条件をかく
その存在条件からあるkについてlが何個存在するかわかるので、Σしたら終わり
答えは(N-1)/(4N-1)かな自信ない
503132人目の素数さん:2012/01/06(金) 00:16:34.54
>>500
(1)は2cos+√3=0で、=0が抜けてました。
504499:2012/01/06(金) 00:18:44.08
ミスってた スマソ
505132人目の素数さん:2012/01/06(金) 00:20:42.89
>>502
答えあってます!
今やってみますね
ありがとうございました
506132人目の素数さん:2012/01/06(金) 00:23:05.10
大学は知らんけど、看護専門学校って黄チャートだとかなりオーバーワークでした。
>>489とか>>495とかの問題が普通に出てくるので。でも倍率が10倍とかなんですけど、合格と倍率って関係ないんですか?
507132人目の素数さん:2012/01/06(金) 00:26:58.32
>>473
俺もどっち解法とも間違ってると思うな・・・
508132人目の素数さん:2012/01/06(金) 00:34:49.86
>>501
下側おかしくない?
4の倍数の中には6の倍数が含まれている。
それはつまり12の倍数だから
ひくのは6の倍数ではなくて12の倍数じゃない?
509132人目の素数さん:2012/01/06(金) 00:37:12.96
>>506
すれち
510132人目の素数さん:2012/01/06(金) 00:40:03.65
>>473

そもそもこれって帰納法でやるか?
511132人目の素数さん:2012/01/06(金) 01:14:37.99
この前微分の定義式を習ったのですが、
y=x^2を微分した時に
y'=lim[h→0](x+h)^2-x^2/h
で、展開してhで約分してlim[h→0]2x+hから2xとなっています。
でももしlim[h→0]のときhを0に近いけど0じゃない数というのなら2x+hは2xではないですよね。
lim[h→0]をhをもういいやって0にするなら0除算してることになりますよね。
なんだかよくわからないので教えてください。
512132人目の素数さん:2012/01/06(金) 01:17:39.97
別に2x+h=2xだとは言ってない
513132人目の素数さん:2012/01/06(金) 01:21:22.22
>>512
514132人目の素数さん:2012/01/06(金) 01:22:39.83
>>512
えええ……?
でも数学Uの教科書lim[h→0]2x+h=2xって書いてるんですけど……。
515132人目の素数さん:2012/01/06(金) 01:23:22.12
でじゃわからん
516132人目の素数さん:2012/01/06(金) 01:24:40.05
だから説明しろよ
517132人目の素数さん:2012/01/06(金) 01:25:03.04
>>514
2x+h=2x の前になんかついてるだろ
518132人目の素数さん:2012/01/06(金) 01:25:19.55
あ?
519132人目の素数さん:2012/01/06(金) 01:29:13.03
lim[h→0](2x+h) = 2x+hに対してh→0という操作を施した結果の値 = 2x
なのであって、
2x+h = 2x
ではない
520132人目の素数さん:2012/01/06(金) 01:31:16.49
>>514
だからlim[h→0]2x+h=2x と書いてあるんであって2x+h=2xと書いてあるわけじゃないだろ。
> でももしlim[h→0]のときhを0に近いけど0じゃない数というのなら2x+hは2xではないですよね。
その通り。2x+h=2xなんてことは言ってない。
>lim[h→0]をhをもういいやって0にするなら0除算してることになりますよね。
だからlim[h→0]h=0なんであってh=0じゃない。0除算になんかならない。
521132人目の素数さん:2012/01/06(金) 01:31:22.04
>>512,>>514,>>519
あ、なるほど。
やっとわかりました。ありがとう。
522132人目の素数さん:2012/01/06(金) 01:32:51.02
ちっ
523132人目の素数さん:2012/01/06(金) 01:34:36.38
>>508
ですね。
524521:2012/01/06(金) 01:34:37.23
安価ミスった
>>512,>>517,>>519あと>>520ありがとうございました。
525473:2012/01/06(金) 01:41:24.52
>>510
やらないと思います。投稿者は自分じゃないので

それにしても証明の手がかりすらつかめません。こういう問題に定石ってあるのでしょうか?
526132人目の素数さん:2012/01/06(金) 01:47:41.44
527473:2012/01/06(金) 01:54:57.53
>>526
この問題が気になって夜しか眠れません
528132人目の素数さん:2012/01/06(金) 01:55:37.79
>>433
立方体の7本の辺を切る場合、連結したn個の面とm個の面に分かれる場合をn+mと書く事にすると、可能性は 1+1+4 , 1+5 , 2+4 , 3+3 , 6 の5種類。
それぞれの数は
1+1+4:1面と1面は隣り合う面だから、つなぐ辺の数で12。
1+5:1面を選ぶ数は6で、4辺を切ってるから5面の方は 12−4=8辺から3辺を切る。
   8C3=56 のうち1面を切り取る場合 4 を除いて、6×(56−4)=312。
2+4:2面をつなぐ辺の数で12、6辺を切って2面をつなぐ1辺は切らないから4面の方は 12−6−1=5辺から1辺を切って、12×5=60。
3+3:3面の中心となる頂点の数で8だが2組あるので 8/2=4、6辺を切ってるから残り6辺から1辺を切って、4×6=24。
6:これが展開図の場合で12C7−(12+312+60+24)=792−408=384。
展開図の確率は 384/792=16/33。
529132人目の素数さん:2012/01/06(金) 02:00:20.05
>>527
バカ、大ヒントだよ
名前がまんま数字とかいうマレなアニメの一つ
530132人目の素数さん:2012/01/06(金) 02:15:16.81
>>528
>>488 は2面の数え方を間違えていた… orz
531132人目の素数さん:2012/01/06(金) 05:41:42.83
>>472
ありがとうございます
532132人目の素数さん:2012/01/06(金) 07:09:53.27
aを0<a<x/2を満たす定数とする。
x^2+(y+sina)^2≦1
y≦(tana)x-sina
の表わすxy平面上の図形をDとする。図形Dをx軸の周りに回転させてできる
立体の体積を求めよ。

交点の座標は(cosa,0),(-cosa,-2sina)
y=(tana)x-sinaを回転させた三角錐の体積は8/3π(sina)^2acosa
y=-sina±√(1-x^2)
これを回転させた立体の体積を求めたいのですがどのように
考えればいいのかわからないので教えてください。
533132人目の素数さん:2012/01/06(金) 07:18:50.16
図形Dをx軸の周りに回転

V=piy^2dx
534132人目の素数さん:2012/01/06(金) 08:09:18.69
>>469
なんでやねん
535132人目の素数さん:2012/01/06(金) 08:10:39.01
確認したいんですが
補角公式のsin(180-θ)=sinθって
sin120=sin(180-60)=sin60と書いた時
sin120(与えられた鈍角)=(180-鈍角120に対する補角60)=sin60(120の補角である鋭角)
と言う読み方で良いのでしょうか?
536132人目の素数さん:2012/01/06(金) 08:42:09.14
>>535
ちょっと変な気がする。
そう読むなら、中央は省いてsin120°=sin60°でいいだろ。
その式は、sin120°=sin(180°-180°+120°)=sin(180°-60°)=sin60°だと思う。

ところで、公式ってsin(180-θ)=sinθってなってるの?
単位を省略したら弧度法とみなされるのでsin(180°-θ°)=sinθ°なのかも知れんけど、
θは通常弧度法で用いる文字なので、θ°ってのはおかしいんじゃないか?
だが、sin(180°-θ)=sinθでは度数法と弧度法が混じってることになるし。
537132人目の素数さん:2012/01/06(金) 08:48:18.71
>>535
sin(120°)= sin(180°-60°)= sin(60°)
与えられた鈍角 180-鈍角120に対する補角60 120の補角である鋭角
でいい。
単位円を書いてx軸正の向きから中心角θの半径の端点が(cos(θ),sin(θ))であることを
使って図で覚えるのが普通。
538469:2012/01/06(金) 08:54:48.31
>>534
眠くて何か馬鹿なこと書いてたスマソ
539132人目の素数さん:2012/01/06(金) 09:01:46.56
>>535
そもそもsin120=sin(180-60)=sin60がおかしい気がする(単位がないことは別にして)。
sin120°=sin(180°-120°)=sin60°なんじゃないか?
sin120=sin(180-60)=sin60は間違いではないが、すでに答えがわかっているのを回りくどく書いていることになっていないか?
つまり、意味のない変形をしている気がする。
180-120=60こそがsin120°を鋭角のsinで表す場合の計算のキモ(それほどのものじゃないけど)だと思う。
540132人目の素数さん:2012/01/06(金) 09:54:23.85
>>536>>537>>539
ありがとうございます
それから単位を省いてしまいましたすみません

sin 120° = sin{180゚ − (180°-120°) } = sin 60°

与えられた鈍角120°=sin(180°-鈍角120°に対する補角60)=sin鈍角120°に対する補角60°
120°の補角は60°であると言う式だと整理してみます

公式にはsin(180°-θ)=sinθと乗っています
弧度法はこのあと出てくるのでその時180°がπに変わるみたいです
541132人目の素数さん:2012/01/06(金) 11:12:27.24
√3sinx+cosx=0
左辺を変形すると、2sin(x+π/6)=0

どうして、こうなるかわかりません。
どなたか教えて下さい。
542132人目の素数さん:2012/01/06(金) 11:12:54.59
>>540
> 公式にはsin(180°-θ)=sinθ
なんか変だな。弧度法はまだ習っていないと言うことのようだが、
そうするとθではなくθ°でなければおかしい。
543132人目の素数さん:2012/01/06(金) 11:16:11.17
>>541
加法定理
544132人目の素数さん:2012/01/06(金) 11:24:23.44
>>543
途中式なの書いていただければ幸いなのですが。
545132人目の素数さん:2012/01/06(金) 11:27:08.72
>>544
√3sinx+cosx=0
((√3)/2)sinx+(1/2)cosx=0
546132人目の素数さん:2012/01/06(金) 11:57:14.04
>>545
加法定理はどこで用いるのですか?
547132人目の素数さん:2012/01/06(金) 12:01:35.95
>>546
考える気ないの?
548132人目の素数さん:2012/01/06(金) 12:05:10.85
>>547
自慢しろよ
549132人目の素数さん:2012/01/06(金) 12:07:10.45
???
550132人目の素数さん:2012/01/06(金) 12:24:46.48
>>547
数学が苦手なもので。
551132人目の素数さん:2012/01/06(金) 12:33:15.69
>>547
まあ、加法定理からみちびけるから、もしかしたらそうともいえるのかな?
ふつうは合成とかいうけど
552132人目の素数さん:2012/01/06(金) 12:46:03.79
童貞線があると数学できないってマジ?
553132人目の素数さん:2012/01/06(金) 13:02:03.97
>>551
合成だわ…何で気づかなかったんだ……
ありがとうございますm(_ _)m
554132人目の素数さん:2012/01/06(金) 13:19:18.87
3点(0、0)(1、2)(2、−1)を通る円の方程式を求めよという問題です。
円の方程式の一般形のx^2+y^2+lx+my+n=0の形にそれぞれのxとyを代入する方法で解いてたんですが
最終的にx^2+y^2-3x-y=0となったんだがこれで合ってるのかな?
555132人目の素数さん:2012/01/06(金) 13:25:17.21
>>554
OK
(1、2)(2、−1)が直径であることに気づくと簡単
556132人目の素数さん:2012/01/06(金) 13:55:54.51
未知数は数値の知られていない数
ってことは変数、定数のどちらでもいいってことですか?
557132人目の素数さん:2012/01/06(金) 14:00:54.29
>>556
> 未知数は数値の知られていない数
「未知数をxとおく」と使うだろ、数だな。それをxおいたのが変数。
変数を含まんだろ。
558132人目の素数さん:2012/01/06(金) 14:05:21.10
>>557
日本語で頼むは
559132人目の素数さん:2012/01/06(金) 14:19:19.48
>>558
560132人目の素数さん:2012/01/06(金) 14:22:20.92
>>555
どうもありがとう!
ちなみにこの円の方程式の中心の座標と半径求めたい時はこの式を平方完成して基本形に直せばいいのかな?
それでやってみると中心の座標が分数になったんだけどその分数が中心の座標でOKなのかな?
561132人目の素数さん:2012/01/06(金) 14:23:30.49
>>560
OK
562559:2012/01/06(金) 14:25:11.55
>>558
ご自分の御高説をかたれ
563132人目の素数さん:2012/01/06(金) 14:26:51.57
>>561
サンキューです
564132人目の素数さん:2012/01/06(金) 14:27:43.02
定積分∫[-1→3](2-x^2)dxを計算するとどうなりますか?
565132人目の素数さん:2012/01/06(金) 14:31:18.26
>>564
被積分関数は分数とかルートついてないよな
566132人目の素数さん:2012/01/06(金) 14:31:52.74
>>557
イミフ
567132人目の素数さん:2012/01/06(金) 14:32:55.82
>>564
教科書嫁
568132人目の素数さん:2012/01/06(金) 14:33:00.39
>>566
自分で説明してみろよ
569132人目の素数さん:2012/01/06(金) 14:44:41.73
>>565>>567
いちおう自分で解いてみたら-4/3になったんですが全く自信がないので合ってるか確かめてください
570132人目の素数さん:2012/01/06(金) 14:51:07.46
>>556
定数の場合は未知であっても未知数とは言わず未知の定数と言う。
変数だけど特定の値をとると考えられる場合には未知数という言い方もする。
571132人目の素数さん:2012/01/06(金) 14:51:37.62
>>569
どういう計算をしたのかを書け。
572132人目の素数さん:2012/01/06(金) 14:56:56.94
>>564
あまりにも教科書の例題っぽくて簡単すぎるから
置換積分の質問くらいじゃないと血が騒がない!!!
√のひとつ位ついてなきゃ 聞くほうも答えるほうもアフォフォだろ!!!!

……っていう流れだ

この問題をホントに聞いてるならその単元をもう一度最初からじかにやり直した方がいいレヴェル
573132人目の素数さん:2012/01/06(金) 15:14:18.07
>>571
∫[-1→3](2-x^2)dx
=[2x−1/3x^3][-1→3]
=(6-1/3・27)-(-2+1/3)
=-4/3

となりました
どこかが間違ってますか?
574132人目の素数さん:2012/01/06(金) 15:26:14.42
>>542
θ=60° 等の記法には何の問題もない
575132人目の素数さん:2012/01/06(金) 16:04:22.51
痴漢積分
576132人目の素数さん:2012/01/06(金) 17:07:57.56
はははすごくおもしろいね
577132人目の素数さん:2012/01/06(金) 17:40:43.40
-3ab+18a+b=136を因数分解したいんですけどどうすればいいでしょうか。教科書に出てくる因数分解と全然違ってわかりません。お願いします。ちなみに因数分解したい理由は整数問題の中の一式だからです。
578132人目の素数さん:2012/01/06(金) 17:50:47.33
今さっき、ローソン行ったんだが、デブ女の店員がいたんだよ。
初めて見る店員だから、ふと、名札を見るとなんと『金』。
どう考えてもキムだろ・・・コイツ、とか思いながら、レジに行くんだが、普通に日本語話してるし、妙に愛想が良くてちゃんと仕事してんだよな。
ちょっと、おかしいぞ、これは、と思ってグーグルで調べると、なんと日本人にも「金」という名字があるらしい。キムじゃなくて「こん」と読むそうだ。
579132人目の素数さん:2012/01/06(金) 17:51:09.17
どう因数分解したいの?

-3ab +18a +b -136 =0
-3a(b-6)+b-6 -130=0
(b-6)(1-3a) -130 =0
とか?
580132人目の素数さん:2012/01/06(金) 17:58:49.45
>>579
アホ、それのどこが因数分解か
581132人目の素数さん:2012/01/06(金) 18:04:04.51
>>580
因数分解ではないけどこれで正解だろ
整数問題言うてるし
582132人目の素数さん:2012/01/06(金) 18:27:10.71
だろうな、因数分解できないし。
悪質な問題だ
583132人目の素数さん:2012/01/06(金) 18:30:10.18
>>579
ありがとうございます。
そうです。とりあえずある文字で整理して同じ項を作るように変形というのはなんとなくわかるんですが実際に何かコツみたいなものはあるんでしょうか?
例えば5ab-2a+3b=10も(5a+3)(b-2/5)=44/5のように変形されるんですがこれもたまたま出来ただけのような気がして
584132人目の素数さん:2012/01/06(金) 18:30:22.79
>>577は何とかして欲しければ元の問題を素直に書くべきだろうに。
585132人目の素数さん:2012/01/06(金) 18:32:10.05
すいません。因数分解のようなものですね。正確に言えなくてごめんなさい。
586132人目の素数さん:2012/01/06(金) 18:42:27.93
5ab-2a+3b=10
まずaで括ってみる
a(5b-2)+3b-10=0
5b-2を無理やり作ればよい。bの係数を合わせるようにして、与式は
a(5b-2) +(3/5)(5b-2)+α=0 と表せれる。
(3/5)(5b-2)+α のbの0次の項は
-6/5 +α  これが-10であるから
α=-50/5 +6/5
=-44/5

よって与式は
a(5b-2)+(3/5)(5b-2)-44/5=0
(5b-2)(a+3/5)-44/5 =0     *5をして
(5b-2)(5a+3)=44


587132人目の素数さん:2012/01/06(金) 18:47:55.51
>>579
>>586 さんと同じことだが…
  5ab-2a+3b=10
⇔ a(5b-2)+3b=10   ← a でくくった これで 5b-2 が見えたので…
⇔ a(5b-2)+{(5b-2)+2}*(3/5)=10  ← 5b-2 を無理やり作ってつじつまを合わせる
⇔ (以下略)
588132人目の素数さん:2012/01/06(金) 18:49:50.58
>>587
安価ミスった 
>>579 ではなくて >>583
589132人目の素数さん:2012/01/06(金) 18:52:27.51
5ab-2a+3b-10=0 を見たときに
aで括ればa*(bの1次式) になることは明白で、
3b-10もbの一次式だから、
(aの一次式)(bの一次式)=C
という形になるってことが計算しなくても推測できるね。
この場合 5b-2, 3b-10が 係数の比が一致していないからCが0でないこともわかる
bで括っても同様のことが言える
たくさん問題こなせば定着すると思うよ
590132人目の素数さん:2012/01/06(金) 19:38:07.50
5ab-2a+3b-10
=5{ab-(2/5)a+(3/5)b-2}
この様に、abの係数が1になるように変形
そして、bの係数をaと同じ括弧の中に、aの係数をbと同じ括弧の中に入れるとよい
=5[{a+(3/5)}{b-(2/5)}-2+6/25]
(余計な物((3/5)*(-2/5))を加えてしまったので、それを引く処理も忘れずに)
=(1/5){(5a+3)(5b-2)-50+6}=(1/5){(5a+3)(5b-2)-44}
591132人目の素数さん:2012/01/06(金) 19:53:03.45
ごちゃごちゃうるせえジャップだな!
592132人目の素数さん:2012/01/06(金) 19:59:57.47
誰も>>473を解けないの?どんだけレヴェル低いんだよここは
593473:2012/01/06(金) 20:03:46.82
すみません>>473ですが、学校の先生に聞いてきてだいたい解決しました
ありがとうございました。
594132人目の素数さん:2012/01/06(金) 21:39:14.45
連続するn個の自然数が全て合成数となるような自然数nの最大値はいくらですか?

というより、最大値は存在しますか?
595132人目の素数さん:2012/01/06(金) 21:41:34.77
そんざいしません
596132人目の素数さん:2012/01/06(金) 21:46:08.13
ぼくすごいね、合成数をしってるだ
597132人目の素数さん:2012/01/06(金) 21:50:56.87
>>573があってるのか間違ってるのか迅速に答えて
598132人目の素数さん:2012/01/06(金) 21:53:13.36
This is urgent.
599132人目の素数さん:2012/01/06(金) 22:15:50.85
>>597
数式の書き方からやり直して来い。
>>[2x−1/3x^3][-1→3]この時点で式無茶苦茶。
聞いてる癖して適当に書いてるとしか思えない。

ちなみに-4/3であってる
600132人目の素数さん:2012/01/06(金) 22:24:06.82
>>599
>>[2x−1/3x^3][-1→3]この時点で式無茶苦茶
めちゃくちゃな引用
601132人目の素数さん:2012/01/06(金) 22:30:15.80
1辺の長さが4の正四面体ABCDにおいて、辺BCの中点をMとする。∠AMD=θとして、cosθの値を求めよ。

この問題を解ける猛者はこのスレにいる?
602132人目の素数さん:2012/01/06(金) 22:37:42.10
ちいせい釣り針
603132人目の素数さん:2012/01/06(金) 22:47:37.29
計算したら√3/4になtta
604132人目の素数さん:2012/01/06(金) 23:09:22.97
>>599 に禿同
605132人目の素数さん:2012/01/06(金) 23:14:36.78
x^2 =-1
の解は実数の範囲では存在しませんが、複素数まで範囲を広げると
x=±i
ですよね。

2^x=-1
この解は複素数の範囲まで広げても直感的には存在しないように思ったのですが、
どうなんでしょうか?
606132人目の素数さん:2012/01/06(金) 23:16:47.39
>>605
>この解は複素数の範囲まで広げても直感的には存在しないように思ったのですが、
信じるままにすすみなさい。神はあなたとともにある。
607132人目の素数さん:2012/01/06(金) 23:17:49.31
無数にあるけど、例えば、
x=iπ/log2
かな
608132人目の素数さん:2012/01/06(金) 23:24:13.94
ちゃんと答えとこ。
x=i(2n+1)π/log2
nは任意の整数。

オイラーの公式を使うのだけど、今の高校は複素数平面じゃなくて行列、一次変換をやってるみたいだから習わないと思う。
609132人目の素数さん:2012/01/06(金) 23:27:29.88
三角形ABCにおいて、BC=32, CA=36, AB=25 とする。
この三角形の二辺の上に両端をもつ線分PQによって、この三角形の面積を二等分する。
そのようなPQの長さが最短になる場合のPとQの位置を求めよ。

学校のチャレンジ問題的な感じで出されてましたがさっぱり分かりません
どなたかわかる人いますか?
難関大の過去問らしく、1年生でも解けるとか……
610132人目の素数さん:2012/01/06(金) 23:40:06.98
CからA方向に18√2の点と、CからB方向に16√2の点。
611132人目の素数さん:2012/01/06(金) 23:42:40.16
PがCA上、QがCB上の点とすると、
CP×CQ:CA×CB=1:2
っていうのを使って、まずこの場合のPQの最小値を考える。
次に、PとQが他の辺にある場合も同様に検討する。
612132人目の素数さん:2012/01/06(金) 23:54:11.22
>>609
1975年の東大
613609:2012/01/07(土) 00:10:54.56
と、東大ですか……
でも>>610 さんの様に1つずつやっていけばできなくもないんですね
ありがとうございました
614132人目の素数さん:2012/01/07(土) 00:41:27.33
高校生がいないかもしれない
615132人目の素数さん:2012/01/07(土) 01:36:54.84
等式lim(x→2)(ax^2+bx+6)/(x^2-x-2)が成り立つとき、定数a,bの値を求めよ。

この問題が溶けないので教えていただきたいです。
616132人目の素数さん:2012/01/07(土) 01:41:09.14
とう…しき…?
617132人目の素数さん:2012/01/07(土) 02:15:00.27
なん……だと……?
618132人目の素数さん:2012/01/07(土) 02:24:04.06
>>615
すみません

=1/2

です。
619132人目の素数さん:2012/01/07(土) 02:39:41.93
二つの二次関数の放物線f(x)とg(x)が同じ対称軸を持つってどういう意味ですか?
頂点のx軸ってことですか?
620132人目の素数さん:2012/01/07(土) 03:04:16.22
「二つの二次関数の放物線f(x)とg(x)は頂点のx座標が同じ」と同値ではあるな
621132人目の素数さん:2012/01/07(土) 03:12:36.20
>>620
ありがとうございます。
622132人目の素数さん:2012/01/07(土) 03:18:43.00
同じ対称軸をもつ2つの放物線f(x)=x^2+2x-3, g(x)=-x^2+ax+bがあり、放物線y=g(x)は点(2,-6)を通るという。このとき次の各問いに答えよ。
(1)定数a,bの値を求めよ。
(2)xの範囲が-2≦x≦1のとき、2次関数g(x)の取り得る値の範囲をもとめ、さらに、P(x)=f(x)・g(x)の最大値、最小値をもとめよ。

まず、同じ対称軸をもつの意味なんですが、頂点のx軸が同じって意味でいいんでしょうか?f(x)とg(x)を平方完成してa=-2,b=2と(1)のほうはわかりました。

(2)なんですが、a,bに代入して
g(x)=-x^2-2x+2
=-1(x+1)^2+3 (-2≦x≦1)
∴-1≦g(x)≦3
ここからわかりません。
f(x)=(x+1)^2-4
g(x)=-(x+1)^2+3
P(x)={(x+1)^2-4}×{-(x+1)^2+3}ってことですか?
623132人目の素数さん:2012/01/07(土) 03:50:52.72
円Oの周上に2点A,Bがあり、∠AOB=120°とする、このとき、この円周上に、もう一つの点Cを任意にとる。△が鈍角三角形となる確率を求めよ。

宜しくお願い致します><
624132人目の素数さん:2012/01/07(土) 04:26:54.89
>>622
OK
>>623
円周角についての定理は小学校でやったはずなので思い出すこと
625132人目の素数さん:2012/01/07(土) 04:54:54.51
>>615
自己解決しました

分子→0 と考えて解く方法でいいんですよね?

ちなみに答えはa=9/4, b=-15/2となりました
626132人目の素数さん:2012/01/07(土) 06:34:30.64
4点A(0,0,2),(2,-2,3),(a,1,4),(1,a,1)が同じ平面上にあるように、定数aの値を求めよ.

誰かこの問題を解いてくれませんか?自分でも解いてみたのですが自信が無いので…
627132人目の素数さん:2012/01/07(土) 07:33:28.11
>>626
先ずはお前の解答を見せてミロや
628132人目の素数さん:2012/01/07(土) 08:47:33.52
>>626
a=6bb61e3b7bce0931da574d19d1d82c88
629132人目の素数さん:2012/01/07(土) 09:41:41.42
>>626
a=8f14e45fceea167a5a36dedd4bea2543
630132人目の素数さん:2012/01/07(土) 10:04:07.27
>>626
a=−1,7
631132人目の素数さん:2012/01/07(土) 10:11:03.80
合同な長方形を2個、少しずらして重ねる。対応する辺の交点を順に
A、B、C、Dとする。ACとBDの交点を中心に回転すると2個の長方形は
重なる。              このことを証明せよ。

632132人目の素数さん:2012/01/07(土) 10:58:08.81
>>631
どこまで考えたの?
633132人目の素数さん:2012/01/07(土) 11:40:26.68
4と6の間に実数解があると言われたら4や6もはいりまsか
634132人目の素数さん:2012/01/07(土) 11:43:00.99
>>633
出題者に訊け。
635132人目の素数さん:2012/01/07(土) 11:43:47.17
>>644
前後から判断不能なら、自分の立場を明記すれば良い
636132人目の素数さん:2012/01/07(土) 11:45:42.48
あちゃ、>>633だな
637132人目の素数さん:2012/01/07(土) 11:45:55.23
物と物にはさまれた部分なので入らないと判断するのが適当
638132人目の素数さん:2012/01/07(土) 11:47:36.45
入試問題なら紛れのないように明記されてるから心配いらん。
639132人目の素数さん:2012/01/07(土) 11:49:19.80
どうも
入試問題ではありませんでした
640132人目の素数さん:2012/01/07(土) 14:00:39.17
なんで休日はこうも質問が少ないんだ?
641132人目の素数さん:2012/01/07(土) 14:45:00.51
高2河合全統の過去問の質問です

pは実数の定数とする。
座標平面上に円C:x^2+y^2=1
と点P(p,2)があり、PからCに引いた2本の接線と
直線y=-1との交点をそれぞれQ,Rとする。
PQRの面積をSとする。

(1)p=0のときSを求めよ

(2)p=1のときSを求めよ

(3)S≧9となるときpの値の範囲を求めよ

お手数ですが解説,アドバイスをお願いします。
642132人目の素数さん:2012/01/07(土) 14:55:06.52
みなさんは優生学についてどう思いますか
643132人目の素数さん:2012/01/07(土) 14:58:55.57
>>642
すれち
644132人目の素数さん:2012/01/07(土) 14:59:41.63
>>641
どこまで解いた
645132人目の素数さん:2012/01/07(土) 15:09:04.26
>>644
すいません
(1)と(2)は自己解決しました。
(3)はPQの長さが6以上となるときを求めれば良いんですよね?

接戦を2本求めて
y=-1との交点の座標を求めて
QRの長さを出す手順で間違いないですか?

646132人目の素数さん:2012/01/07(土) 15:54:56.94
sin15°の値が分かりません

sin15°=1-cos30°/2=2-√3/4となるらしいですが1-cos30°/2から2-√3/4に
出来ません。初歩的な計算で申し訳ないのですが教えてください。
647132人目の素数さん:2012/01/07(土) 16:02:03.58
>>646
スレきったカッコルアーでないなら
>>1
648132人目の素数さん:2012/01/07(土) 16:02:20.84
>>641
これどうやって解くのか俺も気になるから誰か頑張ってください。
内接円っていうのも上手く使えるかと思ったけどよくわからんかったなあ
接線出すにも複雑になるし、、論理的には一意的に求まると思うんだけどギブアップ。
649132人目の素数さん:2012/01/07(土) 16:04:50.45
これまた古典ルアー
650132人目の素数さん:2012/01/07(土) 16:14:08.58
>>631
平面の合同変換は回転と平行移動で表されるが、それはある1点を中心とする回転になる。
1点を中心とする直線の回転は、回転前後の直線の交点と回転中心をむすぶ直線に対する反転と同じ。
平行な2直線の回転は同じ直線に対する反転で表され、回転前後の各直線の交点と回転中心は1直線上にある。
2組の平行線があれば、回転前後の各直線の交点をむすぶ直線が2組あり、回転中心はその交点にある。
651132人目の素数さん:2012/01/07(土) 16:17:01.84
>>647
言ってる意味が分からないんですが・・
長い分子分母にはカッコをつけろということですか?
652132人目の素数さん:2012/01/07(土) 16:20:40.34
>>645
解説を探してくるからちょっとまってね
653132人目の素数さん:2012/01/07(土) 16:28:47.20
>>646
cos30°=√3/2
(1-√3/2)/2
分母分子に*2をすると(2-√3)/4
654132人目の素数さん:2012/01/07(土) 16:47:13.83
円周率ってどうやって計算するんですか
655132人目の素数さん:2012/01/07(土) 16:49:39.71
>>654
それは、にゃんこ大先生も悩んだ大問題
656132人目の素数さん:2012/01/07(土) 16:50:19.71
>>654,655
円周を直径で割る。
小学校で習わなかったか?

解析的にやる方法は高校では不要。
657132人目の素数さん:2012/01/07(土) 16:50:20.28
>>654
いい質問ですね
658132人目の素数さん:2012/01/07(土) 17:12:28.65
効いてる効いてるwww
659132人目の素数さん:2012/01/07(土) 17:35:17.79
>>658
薬か
660132人目の素数さん:2012/01/07(土) 17:35:58.32
>>631
意味不明だろ
661132人目の素数さん:2012/01/07(土) 17:37:14.37
>>656
円周は直径×円周率でいいんですか?
662132人目の素数さん:2012/01/07(土) 17:37:43.23
円周率とセックスしたい
663631:2012/01/07(土) 17:38:16.40
>>650 ありがとうございました。
664132人目の素数さん:2012/01/07(土) 17:44:49.09
>>661
3でいいよ
665132人目の素数さん:2012/01/07(土) 17:53:09.83
>>631
> 合同な長方形を2個、少しずらして重ねる。
666無着清興先生:2012/01/07(土) 18:26:39.27
>>661
ちらしを用意して、コンパスで半径5cmの円を書き切り抜く。
円の中心に穴をあけ中心から半径に沿ってたくさん切れ目を入れる。完全に切ってはいけない。
広げて元の円周のところの長さを測る。それを10でわれば円周率である。
667132人目の素数さん:2012/01/07(土) 19:49:52.24
直径2cmの球の体積は、直径2cmの円を底面とする高さ2cmの円錐の体積の何倍か。
2倍ですか?
基本的なことですいません。
668132人目の素数さん:2012/01/07(土) 19:52:44.31
>>667
どこまで計算した?
669132人目の素数さん:2012/01/07(土) 19:54:32.92
4倍
670132人目の素数さん:2012/01/07(土) 19:55:10.69
>>667
(4000π/3):(2000π/3)=球:円錐
です。
671132人目の素数さん:2012/01/07(土) 19:56:14.04
あ、直径が2cmか
なら2倍で合ってる
672132人目の素数さん:2012/01/07(土) 19:56:14.89
2/3倍
673132人目の素数さん:2012/01/07(土) 19:58:32.21
>>672
うっそだ〜
674132人目の素数さん:2012/01/07(土) 19:58:56.72
>>667
円周列をどうやって計算した?
675132人目の素数さん:2012/01/07(土) 20:00:43.62
円周列が円周率の間違いだとしても、何故そんな質問をするのかわからん
676132人目の素数さん:2012/01/07(土) 20:00:50.23
嘘言う奴は帰れ
677132人目の素数さん:2012/01/07(土) 20:01:18.45
帰ります
678132人目の素数さん:2012/01/07(土) 20:01:49.92
Σ[k=1,n]k! を解けって問題を友達から出されたのですが、全くわかりません。
どうやったら解けるでしょうか?
679132人目の素数さん:2012/01/07(土) 20:02:13.21
>>675
そもそもggrks
680132人目の素数さん:2012/01/07(土) 20:03:39.95
>>678
溶けんよ
681132人目の素数さん:2012/01/07(土) 20:06:56.02
682132人目の素数さん:2012/01/07(土) 20:08:19.10
余計わかりにくくしてどうするw
683132人目の素数さん:2012/01/07(土) 20:09:01.62
>>678
積分表示でも使ってやれば
684132人目の素数さん:2012/01/07(土) 20:09:09.67
>>682
haa
685132人目の素数さん:2012/01/07(土) 20:15:20.31
>>678
Σ[k=1,∞](k!)^z
にすると見通しがよくなるぞ
686132人目の素数さん:2012/01/07(土) 20:18:17.37
687132人目の素数さん:2012/01/07(土) 20:19:42.58
688132人目の素数さん:2012/01/07(土) 20:23:04.07
>>687
円周列が>>667と何の関係があるの?
689132人目の素数さん:2012/01/07(土) 20:31:44.03
>>688
いいから自慢しろよ
690132人目の素数さん:2012/01/07(土) 20:33:49.52
みなさんいろいろアドバイスありがとうございます。
ちなみに、まだ解けません・・

>>685
それだと発散するように見えるのですが・・
691678:2012/01/07(土) 20:34:42.22
書き忘れましたが、690は678です
692132人目の素数さん:2012/01/07(土) 20:35:34.20
>>690
絶対数学です、キリ
693132人目の素数さん:2012/01/07(土) 21:52:19.91
>>690
Σ[k=1,n]k・k! じゃなくてそれ?
694132人目の素数さん:2012/01/07(土) 22:34:42.96
1+1=?
695132人目の素数さん:2012/01/07(土) 22:47:13.31
x,yが3x+6, x≧0, y≧0をみたしているとき、次の問いに答えなさい。
(1)xの取り得る値の範囲
(2)xyの最大値と最小値、及びそのときのx,yの値をそれぞれもとめなさい。
(1)x≦2
(2)x=1,y=3のとき最大値3
あってますかね?
696132人目の素数さん:2012/01/07(土) 22:51:29.52
問題を正確に。
697132人目の素数さん:2012/01/07(土) 22:52:22.23
x,yが3x+6, x≧0, y≧0
なんぞこれ
698 ◆m4dNp2lBPQ :2012/01/07(土) 22:55:03.01
>>645
この問題自己解決しました。
協力してくれた方ありがとうございました
699132人目の素数さん:2012/01/07(土) 22:56:45.42
>>695
>>696
x,yが3x+y=6、x≧0、y≧0を満たしているときのまちがいでした。
700132人目の素数さん:2012/01/07(土) 22:57:50.99
>>698
めっちゃ気になります。
具体的な計算はなくても
具体的な方針を教えていただきたいです
701132人目の素数さん:2012/01/07(土) 22:59:10.86
>>699
(1)取りうる値は0<=x<=2 じゃなきゃ減点されると思うよ。
(2)合ってる
702132人目の素数さん:2012/01/07(土) 23:04:59.60
>>701
それ忘れてました。
つまりx=0,y=6のとき最小値0ですね。
703132人目の素数さん:2012/01/07(土) 23:08:55.38
あ、ごめん最小値もあるのか。
>>701は最大値に関しては合ってるということで。

最小値はx=2,y=0のときも0だから
これも書かないと減点なるよ
704132人目の素数さん:2012/01/07(土) 23:13:38.25
>>703
範囲のセンターが軸ですね。
こういう見落としが怖いです。。
ありがとうございました。
705132人目の素数さん:2012/01/07(土) 23:15:05.70
添削お願いします。
放物線y=x^2+3x+aと直線y=x+4について次の問いに答えなさい。
(1)この放物線と直線が共有点をもつとき、定数aの値の範囲を求めなさい。
(2)この放物線と直線の共有点がただ一つであるとき、その経験を共有点の座標を求めなさい。

(1)a≦5
(2)(-1,-1)
よろしくお願いします。
706132人目の素数さん:2012/01/07(土) 23:19:09.34
>>705
(2)は
共有点の座標でいいの?
(-1,3)じゃない?
707132人目の素数さん:2012/01/07(土) 23:22:08.72
>>705
> 705 名前:132人目の素数さん [sage]: 2012/01/07(土) 23:15:05.70
> 添削お願いします。
> 放物線y=x^2+3x+aと直線y=x+4について次の問いに答えなさい。
> (1)この放物線と直線が共有点をもつとき、定数aの値の範囲を求めなさい。
> (2)この放物線と直線の共有点がただ一つであるとき、その経験を共有点の座標を求めなさい。
>
> (1)a≦5
答えのみでは2点
> (2)(-1,-1)
X
708 ◆m4dNp2lBPQ :2012/01/07(土) 23:29:12.86
>>700
(3)はまず1/2r(a+b+c)の公式を使って
S=1/2(PR+PQ+QR)
円の中心をOとすると
Oは原点なので
OP=√p^2+4
PR上にある円との接点をEとすると
PE^2=OP^2-OE^2
=p^2+3
PE=√p^2+3

PQ,QR上にある円との接点をそれぞれF,G
とすると
接点までの接線の長さは等しいから
PE=PF
QF=QG
RG=RE
これより
S=1/2(PR+PQ+QR)=1/2(2QR+2PE)
よってS=QR+PR・・・1式
QRを底辺とすると
S=3/2QR
QR=2/3S
これを1式に代入して
S=3PE
=3√p^2+3
S≧9より
9≦3√p^2+3
これをといて
-√6≦p p≦√6
709 ◆m4dNp2lBPQ :2012/01/07(土) 23:30:09.40
>>708
答え間違えた
p≦-√6 √6≦p
710690:2012/01/07(土) 23:31:57.62
>>693
問題は書いた通りです
1からnまでk!について和を取る、で問題は間違いないです。
711132人目の素数さん:2012/01/07(土) 23:34:17.51
>>706
ただひとつの共有点は頂点ってことですか?
>>707
(1)の方は何が足りないのか教えてくださいf^_^;)
712132人目の素数さん:2012/01/07(土) 23:39:41.00
>>711
答えにいたる説明がない
713132人目の素数さん:2012/01/07(土) 23:43:07.16
>>711
ただひとつの共有点をもつときというのは
y=x^2 +3x +a
y=x+4
この二つの関数がただ一点で交わるということ。
x^2 +3x +a =x+4
これをxについて解くと共有点のx座標が求まる。
共有点がひとつであるから
D=0 よってa=5
x^2 +3x +5 =x +4
x^2 +2x +1=0
(x+1)^2=0
ただひとつの共有点のx座標は-1である。
y=x+4に代入して y=3
714132人目の素数さん:2012/01/07(土) 23:45:34.24
>>710
君のスペックは?
715132人目の素数さん:2012/01/07(土) 23:55:29.11
青チャートの人こないね
716132人目の素数さん:2012/01/08(日) 00:12:29.00
因数分解ができません。
[1] x^2y^2-x^2-y^2+4xy+1
[2] (x+y+z)^3-x^3-y^3-z^3
どうやるんですかね><
717132人目の素数さん:2012/01/08(日) 00:31:30.32
>>716
どこまでやったの
718132人目の素数さん:2012/01/08(日) 00:43:24.03
二次式は一番効率悪くてもたすきがけで解決します
719132人目の素数さん:2012/01/08(日) 00:45:03.61
4次式だよ
720132人目の素数さん:2012/01/08(日) 00:46:15.07
>>716 [2]
因数定理などの応用を考える手がある
与式を x の式と見て f( x ) とおくと,
   f( −y ) = 0
となるから,
  与式は x + y を因数にもつ
ことがわかる
全く同様にして, y + z , z + x も因数になることがわかる
与式は x の2次式(展開すれば x^3 は消える)だから
   与式 = 定数*( x + y )( y + z )( z + x )
となるはずである
あとは, x , y , z に適当な数を代入して,
この定数を求めればよい
721132人目の素数さん:2012/01/08(日) 00:46:17.03
ひとつの文字に注目すれば2次式。
722132人目の素数さん:2012/01/08(日) 02:12:57.93
実数x,yを変数とする関数f(x,y)が連続でなめらかでありlim_[y→∞]f(x,y)=F(x)に収束するとき
lim_[y→∞]∫[a,b]f(x,y)dx=∫[a,b]F(x)dxは成り立ちますか?
反例や証明があれば教えて欲しいです。
723132人目の素数さん:2012/01/08(日) 02:18:21.59
アスコリ・アルツェラの定理?
724132人目の素数さん:2012/01/08(日) 02:22:38.11
線分OPを直径とし円に内接する四角形OAPBについて、OA↑=a↑,OB↑=b↑とすると
(a↑・b↑)/(a↑・a↑)=1/7,(a↑・b↑)/(b↑・b↑)=1/7 となるとき、OP↑をa↑,b↑で表せ。

色々考えたんですがOA⊥AP,OB⊥BPくらいしかわかんなかったです
お願いします
725132人目の素数さん:2012/01/08(日) 02:23:34.52
724問題文間違えました

線分OPを直径とし円に内接する四角形OAPBについて、OA↑=a↑,OB↑=b↑とすると
(a↑・b↑)/(a↑・a↑)=1/4,(a↑・b↑)/(b↑・b↑)=1/7 となるとき、OP↑をa↑,b↑で表せ。
726132人目の素数さん:2012/01/08(日) 02:25:59.12
>>723
ググってみても、理解出来ません
高校生にわかる範囲の説明が欲しいです
727132人目の素数さん:2012/01/08(日) 03:00:14.44
一様収束 積分と極限の交換
728132人目の素数さん:2012/01/08(日) 06:05:47.58
初歩的な問題かもしれませんがよろしくお願いします。m(_ _)m

A(-2,0,0)
B(0,2,0)
C(0,0,2)  
の三点を通る円の中心の座標と円の半径を求めよ。


△ABCが正三角形であることは利用しないで解いてほしいです。
729132人目の素数さん:2012/01/08(日) 06:36:29.85
>>726
背伸びしすぎちゅうこと
730132人目の素数さん:2012/01/08(日) 06:37:02.31
>>728
どこまで解いた
731132人目の素数さん:2012/01/08(日) 07:01:41.02
>>725
OP↑=□a↑+△b↑
732132人目の素数さん:2012/01/08(日) 07:07:33.49
PA=PB=PC
PA=(A-P)(A-P)=AA-2AP+PP
PA=PB AA-2AP+PP=BB-2BP+PP AA-BB=2(A-B)P
PB=PC BB-CC=2(B-C)P
PC=PA CC-AA=2(C-A)P
2MP=N
P=M^N/2
733132人目の素数さん:2012/01/08(日) 07:08:24.51
PP=(M^N/2)(M^N/2)
734132人目の素数さん:2012/01/08(日) 08:14:13.97
p=sa+tb
apa=(p-a)a=pa-aa=pb-bb=0
saa+tab-aa=sab+tbb-bb=0
s+tab/aa=sab/bb+t=1
s+t/4=s/7+t=1
s=(4-1)/(4-1/7)=21/27
t=(7-1)/(7-1/4)=24/27
p=21/27a+24/27b

735132人目の素数さん:2012/01/08(日) 09:47:38.34
数学で重要な概念のうち、高校三年間で学ぶ数学はどれくらいの割合でしょうか?
漠然とした質問ですいませんー。
主観的なものもあると思いますので、みなさんのお考えを教えて下さい。
736132人目の素数さん:2012/01/08(日) 10:01:26.10
数学は進歩しているということを抑えよう。
↓以下高校数学は数学じゃないうんたらかんたら。
737132人目の素数さん:2012/01/08(日) 10:03:59.20
高校数学は数学じゃなくて暗記うんたら
738132人目の素数さん:2012/01/08(日) 10:09:36.71
整式・因数分解に始まり、二次関数・確率ベクトルを学び、微分不定積分で終わる。
只のガキが学ぶとしては、けっこうなもんだと思うけどな。
739132人目の素数さん:2012/01/08(日) 10:28:20.62
高校数学は大学数学の練習問題が解けるための準備だよ
それができてないと留年する
740132人目の素数さん:2012/01/08(日) 10:34:42.08
ユークリッド数学の重要な概念のの基礎的なものは、高校数学でほぼ学べるんじゃない?
大学以降はそれを発展させたものと思う。
トポロジーとか、ひも理論とかは除いて。
741132人目の素数さん:2012/01/08(日) 10:44:22.26
>>722
f(x,y)=x^2 (1−xy)^2…0<x<1/y, f(x,y)=0…other
742132人目の素数さん:2012/01/08(日) 10:59:02.70
ひもは物理
743132人目の素数さん:2012/01/08(日) 11:02:00.68
微積分
線型代数
確率・統計
ユークリッド幾何
初等整数論
744132人目の素数さん:2012/01/08(日) 11:03:11.21
>>743
追加
常微分方程式
745132人目の素数さん:2012/01/08(日) 15:55:03.54
数学科かそれ以外で全く違う。

数学科じゃなかったら高校数学が分かっていれば、大学の数学は簡単に理解できる。
数学科は知らんけど、もうレベルが違いすぎて何言ってるのか・・・
746132人目の素数さん:2012/01/08(日) 15:55:56.59
整数論とか使わんやろ。
どっちかというとベクトル解析のほうが大事。
747132人目の素数さん:2012/01/08(日) 16:16:10.82
放物線y=x^2+9とx軸で囲まれた図形の面積を求めよ。

計算すると36になったんだけどあってるかな?
748132人目の素数さん:2012/01/08(日) 16:16:57.97
>>747
もんだいがおかしい
749132人目の素数さん:2012/01/08(日) 16:19:37.21
>>748
問題間違いだった
y=-x^2+9とx軸で囲まれた図形の面積です
750132人目の素数さん:2012/01/08(日) 16:49:17.40
途中式かいたらどうなんだ?
751132人目の素数さん:2012/01/08(日) 16:52:02.96
どこが分かってないのかが分からない。
答え合わせなら答えを見ろ。
752132人目の素数さん:2012/01/08(日) 17:20:01.92
1/6公式だろ(棒)
753132人目の素数さん:2012/01/08(日) 18:06:45.02
公式ってなんで正しいの
754132人目の素数さん:2012/01/08(日) 18:12:47.08
公式は使用条件を把握して使用しましょう。
755132人目の素数さん:2012/01/08(日) 18:54:37.18
>>753
数式で表された定理だから
http://ja.wikipedia.org/wiki/%E5%85%AC%E5%BC%8F
756132人目の素数さん:2012/01/08(日) 18:59:55.38
>>753
1.国語辞典
計算の一般法則を記号を使って表した式
2.英語のWiki
In mathematics, a formula is an entity constructed using the symbols and formation rules of a given logical language.
757132人目の素数さん:2012/01/08(日) 19:13:33.59
数学の定理は全て正しい。なぜなら正しくないものがあったとしたら仮定を強めて排除するから。

一方、物理の定理は我々の世界を記述しようとするので、それと違うものがあっても、仮定を強めて排除することが出来ないから、それは違ったのだと言われる。ただし、間違っていたのだとしても、無意味だったわけではない。
758132人目の素数さん:2012/01/08(日) 19:14:25.48
>>757
よく勉強してまちゅね、えらいえらい
759132人目の素数さん:2012/01/08(日) 19:16:23.27
760132人目の素数さん:2012/01/08(日) 19:19:26.19
場合の数の問題で
女の隙間に男を挿入するという解法がありました。
とってもえっちな感じがするんですが大丈夫なんでしょうか。
761132人目の素数さん:2012/01/08(日) 19:28:55.05
>>760
停学
762132人目の素数さん:2012/01/08(日) 19:29:39.10
うーん、熱力学の基本定理とかベルヌーイの定理とか色々とあるんじゃないのかな。
だけど分野とかで違いがありそうだから、なんとも言えないなあ。
763132人目の素数さん:2012/01/08(日) 19:37:47.50
>>762
> うーん、熱力学の基本定理とかベルヌーイの定理とか色々とあるんじゃないのかな。
熱力学の法則、ベルヌーイの定理(Bernoulli's principle)
これ以上はスレち
764132人目の素数さん:2012/01/08(日) 19:38:31.98
ごめんなさい
765132人目の素数さん:2012/01/08(日) 20:48:47.15
>>760
女男男女女男女女女男女男男女男女女
766132人目の素数さん:2012/01/08(日) 21:21:52.35
xについての方程式ix^2+(k-i)x+(4-2i)=0   (iは虚数)
が実数解を持つように、実数kの値を定めよ。
また、その時の実数解を求めよ。

という問題で、判別式を用いると係数のiのせいでうまくいきません
教えてください><

767132人目の素数さん:2012/01/08(日) 21:27:26.67
>>766
実数解の判断においては,判別式は実係数のみで使う。
x,kともに実数なので,iについてまとめて,実部と虚部に分離する。
768132人目の素数さん:2012/01/08(日) 21:28:48.76
判別式は本来は重解を持つかどうかを判定するためのもの。
"実数係数"の2次方程式の場合には、実数解を持つかどうか判定するのにも使える。

今の場合、判別式は使えないので、実部=0かつ虚部=0と言い換えてみる。
769132人目の素数さん:2012/01/08(日) 21:30:17.78
>>766
判別式は実係数のときしかつかえません
770132人目の素数さん:2012/01/08(日) 21:32:16.63
>(iは虚数)
771132人目の素数さん:2012/01/08(日) 21:36:37.08
>>766
今日のMIP候補だな
772132人目の素数さん:2012/01/08(日) 21:55:23.72
数TAの問題なんですが

直方体ABCD-EFGHがあり
AB=3√10 AD=3√6 AE=√10である
(1)でAC=12 CH=10 AH=8
cos∠CAH=9/16
△ACHの面積=15√7
を求めました
問題は(2)なんですが
線分AC上にAI=4となる点Iをとり
3点C,H,Iを通る円と線分AHの交点をJとする

AJ=6というのを方べきの定理で求め
四角形CHJIの面積は45√7/4を求めました

最後に
四角錐D-CHIJの体積は直方体ABCD-EFGHの体積の何倍かという問題がわかりません

具体的な数値を求めればいいのですが
高さがどこになるのかもわかりません

誰か教えてもらえませんか
773132人目の素数さん:2012/01/08(日) 21:55:59.51
だが、断る
774132人目の素数さん:2012/01/08(日) 21:59:20.49
>>772
CHJIを含む平面にDから垂線をおろせば高さはわかるんでは?
775132人目の素数さん:2012/01/08(日) 22:04:31.79
>>774
それの求め方がわかりません

申し訳ないです
776132人目の素数さん:2012/01/08(日) 22:12:34.16
>>772
三角錐D-CHAと比べる。
777132人目の素数さん:2012/01/08(日) 22:16:16.49
>>776
どうやって比べるんですか?

三角形は相似ですが高さが異なると思って
778132人目の素数さん:2012/01/08(日) 22:28:09.62
>>772です
わかりました

具体的な高さは出さなくてもできるんですね

ありがとうございました
779132人目の素数さん:2012/01/08(日) 22:29:13.96
テスト
780132人目の素数さん:2012/01/08(日) 22:29:51.65
半径1/2の円Cの周上に3点PQRをとる。
PQ=x QR=y RP=zとし、
F=(x^2+y^2+z^2)/xyz
とおく。

3点PQRをCの周上で動かすとき、Fの最小値を求めよ。

一日中考えてもわかりません!よろしくお願いします!
781132人目の素数さん:2012/01/08(日) 22:31:08.23
>>780
あっちで回答書いているところだったけど、やーめた
782132人目の素数さん:2012/01/08(日) 22:37:06.95
>>781
節操なくてすいません。なんとか教えていただけませんか?
783132人目の素数さん:2012/01/08(日) 22:41:08.08
拙僧は節操なしでござる
784132人目の素数さん:2012/01/08(日) 22:43:00.93
じじいくせいな、仙石60か?
785132人目の素数さん:2012/01/08(日) 22:43:34.96
黙れ小僧!
786132人目の素数さん:2012/01/08(日) 22:55:55.70
イ`
787132人目の素数さん:2012/01/08(日) 23:50:06.97
この問題をグラフに書いてみたんですが、これでいいんですかね?
放物線y=f(x)が直線y=6x-6に接し、その頂点はy=x-7上にある。
http://beebee2see.appspot.com/i/azuYp-DBBQw.jpg
788132人目の素数さん:2012/01/08(日) 23:50:46.39
>>787
問題ではないでした。
789132人目の素数さん:2012/01/09(月) 00:21:28.47
質問が良く分からんが、お絵描きが上手かってこと?
そうだな、まあまあ上手い
上に凸なy=f(x)の絵も考えないと
790132人目の素数さん:2012/01/09(月) 00:22:32.19
>>787
合ってるよ。
791132人目の素数さん:2012/01/09(月) 00:28:50.43
m^2+4n-4(m+1)a=0という式があって、aが全ての実数値をとるときってどういう意味ですか?
m^2+4n-4(m+1)a=0
すべてのaの値に対して成り立つから
{m^2+4n=0
m+1=0
これを解いてm=-1,n=-1/4
と書かれているのですか、「すべてのaの値に対して成り立つから」の意味がわかりませんし、またそうならなぜ下のような連立方程式になるのですか?

と書かれているのですが、
792132人目の素数さん:2012/01/09(月) 00:29:40.62
>>791
最後の行はまちがいです。
793132人目の素数さん:2012/01/09(月) 00:31:19.28
4行目もねw
794132人目の素数さん:2012/01/09(月) 00:35:51.10
>>791
n次多項式(整式)が値0となる点は多くてもn個しかない
m^2+4n-4(m+1)aはaの1次式または(多項式として)0である
全ての実数aに対してm^2+4n-4(m+1)a=0となるから、
m^2+4n-4(m+1)aは多項式として0
つまり係数は全て0
795132人目の素数さん:2012/01/09(月) 00:40:13.55
>>794
多項式として0ってどういうことですか?
796132人目の素数さん:2012/01/09(月) 00:46:43.99
>>795
ax^n + bx^(n-1) + … (a≠0)
はn次の多項式

つまり、係数に0でないものがあれば、その最高次nを「多項式の次数」と定めている
係数が全て0であるとき、「多項式として0」であると言う
797132人目の素数さん:2012/01/09(月) 00:49:27.23
この状況で多項式として0ねぇ…
たまたま間違いじゃないけど
798132人目の素数さん:2012/01/09(月) 00:52:21.12
>>791
問題を正確に書いて
799132人目の素数さん:2012/01/09(月) 00:54:54.17
>>798
aが全ての数(実数)値をとるとき、放物線y=(x+a)^2+aは常に1つの直線y=mx+nと接する。
このときのm,nの値を求めよ。
800132人目の素数さん:2012/01/09(月) 00:59:47.76
>>799
aが全ての実数値をとるということは
a=0
a<0
a>0
これらのときその式を成立させたいんだったらm+1=0としたらaがどんな実数値だろうと成立するだろ。
801132人目の素数さん:2012/01/09(月) 01:08:49.51
>>799
xについての方程式(x+a)^2+a-(mx+n)=0が重解をもつから
その判別式m^2+4n-4(m+1)a=0
放物線y=(x+a)^2+aは(aに関して)常に1つの直線y=mx+nと接するから
判別式=0が(aに関して)常に成り立つ
つまり、aについての方程式m^2+4n-4(m+1)a=0の解は実数全体となる
続きは>>794の通り
802132人目の素数さん:2012/01/09(月) 01:09:58.91
ax^2+4x+a>0が全ての実数xにたいして成立するとき、aの値の範囲をもとめよ。
これは答案はa<2だけでいいんですかね?
それとも0<a<2ですか?
803132人目の素数さん:2012/01/09(月) 01:12:08.00
>>802
a=-100のとき
(-100)x^2+4x+(-100)>0 は全ての実数xにたいして成立するか?
804132人目の素数さん:2012/01/09(月) 01:13:04.68
多項式の0とか言いだす前に、a=0,1辺りで必要条件、逆に…
とか辺りが工房流というか
多項式の0の定義を持ちだすと余計ややこしいw
805132人目の素数さん:2012/01/09(月) 01:16:52.27
>>802
4-a^2>0じゃねえぞ

4-a^2<0
a>±2
下に凸に放物線のときなのでa>0
よってa>2
806132人目の素数さん:2012/01/09(月) 01:18:06.61
>>804
俺も思った
センター数1程度なら>>800の理解で十分
807132人目の素数さん:2012/01/09(月) 01:21:40.36
>>805
> a>±2
減点
808132人目の素数さん:2012/01/09(月) 01:23:16.25
a>±2
なんだこれなめてんのか
809132人目の素数さん:2012/01/09(月) 01:26:01.83
±2> a>±2
うおおおおお
810132人目の素数さん:2012/01/09(月) 01:34:04.95
>>799
別解
与えられた放物線の式はパラメータ a についての2次方程式と見ることができるので,
「 判別式 = 0 」 として得られる図形が包絡線になることが知られている
これを示しながらやるには,例えば次のようにする

   y = ( x + a )^2 + a
  ⇔ a^2 + ( 2x +1 ) a + x^2 - y = 0
  ⇔ { a + x + ( 1/2 ) }^2 - x - y - ( 1/4 ) = 0 .…(あ)  ← a について平方完成した
ここで,直線
   y = -x - ( 1/4 ) …(い)  ←上の平方完成で { } の外にあるものを = 0 とした式である
を考える.
放物線の式と連立して,
   { a + x + ( 1/2 ) }^2 = 0  ←(あ)で { } の外を = 0 としたのだから,こうなるのは当たり前
つまり,放物線(あ)と直線(い)は,各 a ごとに
   x = - a - ( 1/2 ) において接する
ことがわかる.
よって,この直線が求める直線である.
811132人目の素数さん:2012/01/09(月) 01:51:06.43
sinA:sinB:sinC=√7:√3:1のとき、
(1)正弦定理を利用してa:b:c=√7:√3:1を証明せよ。
(2)余弦定理を用いて、この三角形の最大角の大きさを求めなさい。
(1)ですが、
a/2R:b/2R:c/2R=√7/2R:√3/2R:1/2R
よってa:b:c=√7:√3:1
でいいでしょうか?
また、(2)は150°でいいでしょうか?
812132人目の素数さん:2012/01/09(月) 01:56:35.62
>>811
何が不安なんだ?
813132人目の素数さん:2012/01/09(月) 01:59:17.02
>>812
過去問なのですが、解答がないんです。
814132人目の素数さん:2012/01/09(月) 02:04:35.00
>>811
正弦定理使うとき証明は R の説明をちゃんとつけとけよ
減点されちゃうぞ
815132人目の素数さん:2012/01/09(月) 02:10:17.29
>>814
sinA=a/2R(以下略)なので、ですか?
それともR>0のときということでしたら、どうやってR>0は証明するんですか?
816132人目の素数さん:2012/01/09(月) 02:11:58.55
なかなか良い質問だw
817132人目の素数さん:2012/01/09(月) 02:12:19.36
三角形の外接円の半径をRとする。ということでしょう。
半径をRとする。と言ったということはR>0であることも明示したことになる。
818132人目の素数さん:2012/01/09(月) 02:14:49.66
>>815
そんなに難しー話じゃないよwww
その R は三角形ABCの外接円の半径だってことを書いとけよってこと

ややこしいこと書いちゃってごめんね
819132人目の素数さん:2012/01/09(月) 02:17:14.16
>>817
>>818
ありがとうございました。
820132人目の素数さん:2012/01/09(月) 02:25:13.49
x+y+z=3, xy+yz+zx=2, xyz=3のとき、
x^2+y^2+z^2, 1/x+1/y+1/zをもとめよ。

因数分解の公式でこんなのがあったような気がするのですが、それを知らないとしたらどうやって解けばいいですか?
821132人目の素数さん:2012/01/09(月) 02:27:22.40
(x+y+z)^2を展開してみる。

通分してみる。
822132人目の素数さん:2012/01/09(月) 02:28:08.09
>>821
そういう問題なんですかね?
分数の方は検討もつかないんですか。
823132人目の素数さん:2012/01/09(月) 02:29:42.74
そういう問題だと思うよ。
x^2+y^2+z^2=(x+y+z)^2 -2(xy+yz+zx)=9-4=5

1/x +1/y +1/z =(xy+yz+zx)/xyz=2/3
824132人目の素数さん:2012/01/09(月) 02:34:48.16
>>823
簡単な質問に付き合ってくれてありがとうございました。
825132人目の素数さん:2012/01/09(月) 02:45:06.84
http://s1.gazo.cc/up/s1_10916.jpg
上図で、
AB=BCの時、すぐに角CDB=角ADBがいえるらしいのですが、
なぜでしょう?
確かに証明はできますが、なぜすぐ言えるのか。
826132人目の素数さん:2012/01/09(月) 02:45:31.47
ようは、普通はどんなふうにして(もっとも最短に)
角度が等しいことを言うのでしょうか?
827132人目の素数さん:2012/01/09(月) 02:54:21.14
相似とかの証明だったら、∠***=∠***(円周角)
ぐらいはつけておいたほうがいいかもね。
828132人目の素数さん:2012/01/09(月) 03:06:31.43
>>823
違う
829132人目の素数さん:2012/01/09(月) 03:10:04.68
>>825
違う
830132人目の素数さん:2012/01/09(月) 03:14:24.28
>>829
何が違うのかわからない
831132人目の素数さん:2012/01/09(月) 03:35:34.03
>>825
俺の数学偏差値40だから信じないで欲しい。
AB/sin∠ADB=2R=BC/sin∠CDB (正弦定理)
AB=BCより、sin∠ADB=sin∠CDB.
∴∠ADB=∠CDB
832132人目の素数さん:2012/01/09(月) 03:42:28.95
>>825
俺の数学偏差値30だから信じて欲しい。
∠ADB=∠CDB(円周角)
833132人目の素数さん:2012/01/09(月) 04:23:37.57
>>825
△AOB≡△COB(条件と半径から三辺相等)
よって∠AOB=∠COB
これらは弧AB,弧BCの中心角であるから円周角の∠ADB,∠CDBも等しい
834132人目の素数さん:2012/01/09(月) 08:40:15.32
>>825
そのような図であればAB=BCのとき、弧ABと弧BC(いずれも短い方)は等しい。
従って、それらの円周角も等しい。
1行目で言ったことは一目でわかるのでいちいち言わなくてもいいということなんじゃないか?

> 確かに証明はできますが
どうやって証明したの?
835132人目の素数さん:2012/01/09(月) 10:21:18.37
1.(同じ弧長に対する)中心角は等しい (∵合同)
2.中心角は、円周角の二倍 (∵外角は二つの内角の和)
3.従って(同じ弧長にたいする)円周角は等しい (∵三段論法)
836132人目の素数さん:2012/01/09(月) 11:54:59.83
-(c+d)^3+3ab(c+d)+(c+d)^3-3cd(c+d)-3(a+d)(b+d)(c+d)=3(c+d)(ab-cd-(a+d)(b+d))

(c+d)^3はどこへ行ったのですか?
837132人目の素数さん:2012/01/09(月) 11:59:58.14
>>836
-(c+d)^3はどこへ行ったと思うの?
838132人目の素数さん:2012/01/09(月) 12:00:11.43
みなみへ行きました
839132人目の素数さん:2012/01/09(月) 12:01:40.11
深遠な問題ですな
840132人目の素数さん:2012/01/09(月) 12:12:22.67
@x^2-30x+225=0

Ax^2+14x-49=0

それぞれの方程式を解く場合どうすればいいんですか?
因数分解は両方できないし解の公式でも使うの?
841132人目の素数さん:2012/01/09(月) 12:14:08.99
>>840
> 因数分解は両方できないし解の公式でも使うの?
842132人目の素数さん:2012/01/09(月) 12:40:09.02
>>841
なんだよ
わからないなら安価すんなよ 
気持わりーな
843132人目の素数さん:2012/01/09(月) 12:42:48.01
解の公式を使うことに躊躇いでもあるのか?
844132人目の素数さん:2012/01/09(月) 12:47:52.13
>>842
> 気持わりーな
845132人目の素数さん:2012/01/09(月) 12:48:58.44
>>840
> 因数分解は両方できないし解の公式でも使うの?
忍法帳か
846132人目の素数さん:2012/01/09(月) 12:51:57.36
(1)は因数分解できるように見えるが……
847132人目の素数さん:2012/01/09(月) 13:14:39.84
>>846
@x^2-30x+225=0
これ因数分解できるか?
できるならやってほしい
848132人目の素数さん:2012/01/09(月) 13:19:05.55
釣りみたいなんで、この質問は終了
849132人目の素数さん:2012/01/09(月) 13:19:53.11
つまらん質問すんじゃねーよボケ!
850132人目の素数さん:2012/01/09(月) 13:41:57.41
>>847
15^2、7^2、2×15、2×7
851132人目の素数さん:2012/01/09(月) 13:50:44.58
http://www.j3e.info/ojyuken/math/kyoto/q_jpg/1984_3.jpg
これの(2)をいわゆるファクシミリの原理でやろうとしたら全くうまくいかないのですがどこが悪いでしょうか?

直線PQの式はy=tx-t^2
xを-1<=x<=2で固定しyをtの関数とみるとy=-(t-x/2)^2+x^2/4
ここからがうまくいきません
852132人目の素数さん:2012/01/09(月) 13:59:44.12
>>851
ファクスの電源いれるね
853132人目の素数さん:2012/01/09(月) 14:04:25.56
因数分解はどんな二次方程式もできるだろ
ただ複雑な数字だからそんなことするよりかは二次方程式の基本形を変形して
xについて解いた解の公式という便利な物を使った方が速いってだけだよ

854132人目の素数さん:2012/01/09(月) 14:06:31.64
組み合わせ男6人女4人の全10人から4人選ぶ
これを10C4以外の方法で求めるにはどんなことをすればいいのでしょうか?
855132人目の素数さん:2012/01/09(月) 14:10:15.50
どんな方法で選んでも結果は同じ10C4
856132人目の素数さん:2012/01/09(月) 14:20:43.37
>>850
Aはともかく@はなるほどですね
ありがとうございます
>>848-849みたいな文句ばかりで問題も解けないバカとは大違いです
857132人目の素数さん:2012/01/09(月) 14:20:46.60
また忍法帳か
858132人目の素数さん:2012/01/09(月) 14:22:07.29
>>851
何がどううまくいかないというのか?
859132人目の素数さん:2012/01/09(月) 14:22:11.09
>>856
身の程を知らないバカにばかにされるとやっぱり腹がたつ
860132人目の素数さん:2012/01/09(月) 14:28:40.02
>>859
バカと言われて腹立つ奴は図星の証拠
やっぱりお前はバカだよ
861132人目の素数さん:2012/01/09(月) 14:29:58.21
マジレスすると、2次式の因数分解ができない回答者なんてまずいないから。
皆、アホらしすぎて呆れてただけ。
862132人目の素数さん:2012/01/09(月) 14:31:17.58
>>860
こんにちは、ばか
863132人目の素数さん:2012/01/09(月) 14:33:45.98
>>862 だからおまえがバカw
864132人目の素数さん:2012/01/09(月) 14:34:46.02
質問が終わったらさっさと消えましょう。
あなたがバカかどうかとは関係ありません。
865132人目の素数さん:2012/01/09(月) 14:36:11.66
>>858
tの変域がよくわからずyの最大値最小値が求まりません
866132人目の素数さん:2012/01/09(月) 14:59:14.66
>>856
なんだ、ちゃんと因数分解ができているようじゃねえか。
丸2で引っ掛るとおもったけど、ちゃんと回避している。
867132人目の素数さん:2012/01/09(月) 15:03:11.38
>>861
言い訳見苦しいw
868132人目の素数さん:2012/01/09(月) 15:06:15.97
よかったね初めてレスついて、忍法帖
869132人目の素数さん:2012/01/09(月) 15:08:06.36
>>868
忍法帖とかwww
うけるwww
870132人目の素数さん:2012/01/09(月) 15:09:58.38
>>869
じじいくせーな、仙石60か
871132人目の素数さん:2012/01/09(月) 15:11:42.49
実際中3くらいでも出る問題だぞそれ
それで因数分解できないだの解の公式だの喚いてるのはすごく・・・かわいそうというか・・・
872132人目の素敵さん:2012/01/09(月) 15:17:25.83
冬休み最終日だからかねえ
873132人目の素数さん:2012/01/09(月) 15:25:08.92
>>865
問題文に書いてるじゃん
>tの変域
874132人目の素数さん:2012/01/09(月) 15:30:51.60
>>873
0<x<2のとき実際の最小値はy=x-1なのですがこの考えで行くとmin{0,x-1}となりx=1で入れ替わることになってしまいます
875132人目の素数さん:2012/01/09(月) 15:58:47.11
>>851
直線 x = k と線分 PQ が必ずしも共有点をもつわけではないことに注意
共有点をもつための条件から t の範囲が決まる
876132人目の素数さん:2012/01/09(月) 17:11:38.18
(a+b+c)^3-a^3+b^3+c^3
どうやって因数分解するのでしょうか><
877132人目の素数さん:2012/01/09(月) 17:13:47.55
前2項と後2項をまず、因数分解してみよ。
878132人目の素数さん:2012/01/09(月) 17:46:09.19
一部の正負ミスってる予感
879132人目の素数さん:2012/01/09(月) 17:47:30.80
(1-a)(2-a)(3-a)-2+2(2-a)-2(1-a)=0
(1-a)(2-a)(3-a)=0
の途中式をなるべく詳しく教えてください お願いします。

880132人目の素数さん:2012/01/09(月) 17:51:12.60
>>879
フツーに後半の項をばらせば正負で消える
881132人目の素数さん:2012/01/09(月) 18:04:41.89
>>880
共通項をくくってできませんかね?
882132人目の素数さん:2012/01/09(月) 18:09:16.76
>>881
だから後の3項を計算してみろって。
883132人目の素数さん:2012/01/09(月) 18:10:09.28
(a+b+cw)^3-a^3-b^3-c^3の間違いでした!
どうやるんでしょうか?
884132人目の素数さん:2012/01/09(月) 18:10:41.84
↑(a+b+c)^3
885132人目の素数さん:2012/01/09(月) 18:14:41.39
886132人目の素数さん:2012/01/09(月) 18:21:28.44
716だろ
887132人目の素数さん:2012/01/09(月) 18:28:30.02
行列の問題で
A=(3 1) E=(1 0) O=(0 0)
 (-1 1)  (0 1)  (0 0)
Q1.A-kE=BとしてB^2=O となるときの実数kを求めよ。
これはk=2だと思うのですが、
Q2.Q1で得たkとBを用いると、A=B+kE と表される。これを利用してA^nを求めよ。
が、どうしても分かりません。
だれか教えてください。お願いします。
888132人目の素数さん:2012/01/09(月) 18:31:39.09
>>887
二項定理
889132人目の素数さん:2012/01/09(月) 18:40:55.33
>>888 そんなのあったな… 無事解けました。ありがとうございました。
890132人目の素数さん:2012/01/09(月) 19:01:26.87
>>881
どこに共通項があるん?
891881:2012/01/09(月) 19:13:02.74
計算するのが面倒だったので、
考えた振りをして答をかいてもらうつもりでした。
ごめんなさい。
892132人目の素数さん:2012/01/09(月) 19:39:16.56
死ね
893132人目の素数さん:2012/01/09(月) 19:51:24.34
ここの人はヒント言うだけで計算してくれないよね(´・ω・`)
894132人目の素数さん:2012/01/09(月) 19:52:44.91
足し算掛け算すらできない高校生はまさかおるまい
895132人目の素数さん:2012/01/09(月) 19:54:01.71
>>893
教育的配慮(棒)
896132人目の素数さん:2012/01/09(月) 20:03:28.54
2次方程式x^2+2ax-a+2=0の解が異なる2つの解であり、ともに1より小さい時、定数aの値の範囲を求めよ。
このグラフの条件は
D>0
軸<1
f(1)>0
ですよね?これは間違いですか?http://beebee2see.appspot.com/i/azuYy8XCBQw.jpg
897132人目の素数さん:2012/01/09(月) 20:04:48.38
画像は見れないけどその3つの条件で合ってる
898132人目の素数さん:2012/01/09(月) 20:04:48.59
間違い
899132人目の素数さん:2012/01/09(月) 20:05:49.75
うんだ
900132人目の素数さん:2012/01/09(月) 20:09:34.42
これ誤字多すぎなんですけど(^_^;)
しかも古すぎて虚数とか1Aなのにでてきます。笑
問題は良問多くてチャートより理解できるんですけど。http://beebee2see.appspot.com/i/azuYvbDCBQw.jpg
901132人目の素数さん:2012/01/09(月) 20:11:13.78
ここの回答者っていうか、どこだって質問者も回答者も人それぞれだろ
回答者だってただの自己満で答えてるんだから
立派な回答がもらえるとは限らない
質問者も煽られ耐性ないからってぐちぐち言ってても回答もらえる訳じゃないし
最初からそんな期待しないほうがいい
902132人目の素数さん:2012/01/09(月) 20:17:07.64
>>897
>>898
>>899
国語的な質問なんですけど、これが2つの解の場合D≧0なんですか?
あくまでも二次関数は解は2つあるという意味で。
903132人目の素数さん:2012/01/09(月) 20:19:02.43
>>902
2解ならD≧0
同じ解を2つ持ってるって意味になるね
904132人目の素数さん:2012/01/09(月) 20:19:43.08
ありがとうございます
905132人目の素数さん:2012/01/09(月) 20:36:14.04
二次方程式の1つの解かわ-1と0の間に、他の解が2と3の間にあるグラフについて
-1と0の間にあるの条件がf(-1)・f(0)とあるのですが、かけたらその間に解がある条件になるのはなぜですか?
906132人目の素数さん:2012/01/09(月) 20:39:26.73
>>905
f(-1)・f(0)<0だろ
だまされたと思ってグラフ書いてみろよ
907132人目の素数さん:2012/01/09(月) 20:40:03.06
f(-1)*f(0)<0
f(2)*f(3)<0

だろうか。
f(-1)*f(0)の値が負になるということは、f(-1)かf(0)のどちらかが正で、またどちらかが負であるということ。
-1<x<0の間で正負が変わっていることになる。
ということは-1<x<0の範囲でx軸と交わることになる。
908132人目の素数さん:2012/01/09(月) 20:41:03.10
数学の質問です、いくら考えてもわからないので教えてください…

f(x)=ax^3+bx^2+cx+dがA(α、f(α)),B(β、f(β))で極値をとるとき、
f(α)=f(s)となるs(s≠α),f(β)=f(t)となるt(t≠β)に関して、x軸上でsはα、βを3:1に、tは1:3に外分していることを示せ。
909132人目の素数さん:2012/01/09(月) 20:43:32.08
>>907
なんか数学のおもしろさがわかった気がします
ありがとうございました。
910908:2012/01/09(月) 20:43:54.82
すいません追記です。
f(x)-f(α)=0の解sを求めることは分かったのですが、具体的にどう計算し証明すればいいのか教えてください
911132人目の素数さん:2012/01/09(月) 20:47:31.72
>>908
x軸上でsは...のくだりってどゆこと?
読解能力疎くて申し訳ない
912132人目の素数さん:2012/01/09(月) 20:48:45.34
s,t,α,βの 「x座標」にだけ注目して
外分してるってこと?
913908:2012/01/09(月) 21:03:53.16
>>911
極大値であるf(α)は3次関数上にもう一つ同様の数値が存在します。
そのx座標がsとなっています、極小値であるf(β)とt、βの関係も同様です。
グラフをにしてみると具体的にわかりやすくなると思います。

>>912
そういうことです。

x軸上でsはα、βを3:1に、tは1:3に「それぞれ」外分していることを示せ。
すみません、それぞれが抜けていました。


914132人目の素数さん:2012/01/09(月) 21:08:14.00
>>913
f(x)-f(α)は(α, 0)でx軸に接しているから、(x-α)^2を因子に持つよ

f(x)-f(α)
= a(x^3-α^3) + b(x^2-α^2) + c(x-α)
= (x-α) ( a(x^2+αx+α^2) + b(x+α) +c )
= (x-α) ( a( x^2-α^2 + α(x-α) + 3α^2 ) + b( x-α +2α) + c )
= (x-α) ( (x-α) ( a(x+α+α) + b) + 3aα^2+2bα+c )
= (x-α)^2 ( a(x+α+α) + b )
915908:2012/01/09(月) 21:14:09.46
>>914
質問させてください。
f(x)-f(α)はなぜx軸に接しているといえるのですか?
916914:2012/01/09(月) 21:17:43.43
>>915
直感的には、グラフを書けばすぐわかる
証明するには、>>914 の計算をすることになるだろう
917132人目の素数さん:2012/01/09(月) 21:32:56.95
>>915
A(α、f(α)),B(β、f(β))で極値をとるという条件があるから。
918132人目の素数さん:2012/01/09(月) 21:42:58.63
ちょっと確認させて下さい。
0.9999999…
は1に近づく極限値ではなくて、
1そのものでよろしいですか?
919908:2012/01/09(月) 21:43:20.93
>>914
>>917
なるほど、式は理解できました。

その後の証明はどうすればいいでしょうか・・・?
理解が悪くすいません

920132人目の素数さん:2012/01/09(月) 21:43:23.17
二つの二次関数の放物線f(x)とg(x)が同じ対称軸を持つ
この意味を教えてください。
何の軸に対して対称なんでしょうか?
921914:2012/01/09(月) 21:47:13.61
>>919
>>914 の計算から、a(s+2α)+b=0
s=-2α-b/a
あとは、f'(x)=0の解と係数の関係
922132人目の素数さん:2012/01/09(月) 21:48:04.73
>>918
「1に近づく極限値」という表現は数学にはない
「1に近づく数列」ならよい

1に近づく数列:
0.9 0.99 0.999 0.9999 …
の極限値は0.9999999…(=1)
923132人目の素数さん:2012/01/09(月) 21:48:05.48
>>920
二次関数の放物線はその軸に関して線対称 という意味じゃない?
924132人目の素数さん:2012/01/09(月) 21:49:08.99
>>920
放物線の軸が同じってことじゃないの?
y=x^2はy軸を軸とするだろ?
925132人目の素数さん:2012/01/09(月) 21:53:29.57
>>918
アルキメデスの雉だよ
926132人目の素数さん:2012/01/09(月) 21:53:48.64
>>923
>>924
ややこしい言い回しですよねこれ。
x^2の係数が同じだと思ってました。
頂点の軸が一緒ってことなんですね。_φ(・_・
927132人目の素数さん:2012/01/09(月) 21:54:57.09
>>926
頂点の軸?
君の言葉の方がわけわからんよ。
928132人目の素数さん:2012/01/09(月) 21:56:20.16
ちゃんと言葉の意味を理解してたら普通に分かることだろ
929908:2012/01/09(月) 21:57:39.63
>>921
おおっ、やっと理解できました。
ありがとうございます!
930132人目の素数さん:2012/01/09(月) 21:59:58.38
頂点を通るx軸?
931132人目の素数さん:2012/01/09(月) 22:05:49.30
△ABCと△DACが相似なのはどこで判断できるんですかね?http://beebee2see.appspot.com/i/azuYwZ_ABQw.jpg
932132人目の素数さん:2012/01/09(月) 22:07:32.77
>>931
解決しました
933132人目の素数さん:2012/01/09(月) 22:07:40.85
2角相等
934132人目の素数さん:2012/01/09(月) 22:09:05.83
なんとその差たったの8秒
935132人目の素数さん:2012/01/09(月) 22:09:13.63
>>931
直角三角形で直角でない角がおなじ
936132人目の素数さん:2012/01/09(月) 22:09:42.62
つりか
937132人目の素数さん:2012/01/09(月) 22:26:15.74
938132人目の素数さん:2012/01/09(月) 22:27:01.79
dy/dx
939132人目の素数さん:2012/01/09(月) 22:27:04.75
いやです
940132人目の素数さん:2012/01/09(月) 22:27:29.67
計算くらいは自分でやれ
941132人目の素数さん:2012/01/09(月) 22:34:58.38
お尻の穴=#+#
942132人目の素数さん:2012/01/09(月) 23:05:37.72
とりあえず、自分の考え書いてみ。
間違ってるなら指摘してあげるから。
943132人目の素数さん:2012/01/09(月) 23:23:28.62
二次関数の放物線f(x)とg(x)が同じ対称軸を持つ
f(c-x)=f(c+x) and g(c-x)=g(c+x)
944132人目の素数さん:2012/01/09(月) 23:24:03.95
945132人目の素数さん:2012/01/09(月) 23:59:02.79
946132人目の素数さん:2012/01/10(火) 00:05:57.22
OA=√5、OB=5の三角形OABの辺AB上に2∠AOC=∠BOCとなるように点Cをとると、
△OAC:△OBC=1:4であった。∠AOC=θ,OC=xとするとき、cosθ、xを求めよ。

△OAC:△OBC=(√5/2)xsinθ:(5/2)xsin2θ=1:4
5xsin2θ=(4√5)xsinθ
5sin2θ-4√5 *sinθ=0
sinθ(10cosθ-4√5)=0
cosθ=2√5/5 =2/√5

△OAC:△OBC=AC:CBであるから,AC=yとおくとCB=4yとおける。
△OACに余弦定理を用いて
y^2=5+x^2-(2√5)xcosθ @
△OBCに余弦定理を用いて
16y^2=25+x^2-10xcos2θ A
@×16してAと連立
80+16x^2 -(32√5)xcosθ=x^2+25-10xcos2θ
cos2θ=2cos^2θ -1, cosθ=2/√5 を代入して整理して
15x^2-58x+55=0
(3x-5)(5x-11)=0
x=5/3 ,11/5

cosθが求まっているので、∠AOBは固定されていますよね。OA,OBも与えられているので、
三角形OABは確定されて、三角形OACも確定されると考えて、
xの値はただ一意的に求まると思ったのですが、どちらが不適かわかりません。
947132人目の素数さん:2012/01/10(火) 00:50:38.51
948132人目の素数さん:2012/01/10(火) 00:54:39.43
3Cの微分の計算がおおすぎるお
なんでグラフ描くだけであんなに時間かかるの…
949132人目の素数さん:2012/01/10(火) 01:02:57.54
>>946
その解法だと、点CがABを内分してるか外分してるかが定まってない
950132人目の素数さん:2012/01/10(火) 01:28:33.89
>>949
なるほど。
しかし吟味方法が全く浮かばないのですが・・
951132人目の素数さん:2012/01/10(火) 01:28:57.50
>>946
3θがπ/2より小さいことを確認せよ。
952132人目の素数さん:2012/01/10(火) 01:36:24.07
>>950
外分してる場合と内分してる場合を図にしてみなよ
せっかくだからx=5/3のときと11/5のときで考えてもいい
953132人目の素数さん:2012/01/10(火) 01:42:36.91
>>951
ごめん。間違いだった。
954132人目の素数さん:2012/01/10(火) 01:50:34.93
http://iup.2ch-library.com/i/i0530448-1326127606.png
http://iup.2ch-library.com/i/i0530445-1326127418.png
二通りかいてみたのですが、
どちらも矛盾が生じてしまいました
内分と外分している画像の方(二枚目)では、
角が等しく、その向かいの辺も等しいので、
OC=OEとなってしまいます。(違う値が出たのに)

1枚目が正しい作図ですよね?
しかし吟味に持っていけません。
955132人目の素数さん:2012/01/10(火) 01:51:09.73
訂正: どちらも ではなく 二枚目では でした
956132人目の素数さん:2012/01/10(火) 01:58:21.09
何度も申し訳ありません。
1枚目はhttp://iup.2ch-library.com/i/i0530453-1326128193.png
こちらです

それと>>954ではどうもおかしなこと言っていますね
こんがらがってきたので今一度しっかり整理してきます
957132人目の素数さん:2012/01/10(火) 01:58:21.73
角AOBは鋭角。OA<OBでCが辺AB上にあるなら、OA<OC=x<OBの筈。
しかし、x=5/3、11/5どちらであっても x<√5=OA。おかしい。
958132人目の素数さん:2012/01/10(火) 02:00:01.22
>>957
これも嘘だ。
959132人目の素数さん:2012/01/10(火) 02:21:31.88
>>946
3倍角公式で sin3θを求めて,面積から攻めるのでは駄目なのか?
このほうが紛れがなくてよさそうだが…
960132人目の素数さん:2012/01/10(火) 02:29:37.27
>>959
なるほど!!!
今回の質問に関係ないから省いてたのですが
sin3θの値も求めさせられたんですよね、
面積から攻めろとほのめかしていたのですね・・

cosθ=2/√5 よりsinθ=1/√5
sin3θ=3sinθ-4sin^3θ=3/√5 -4/5√5 =11/5√5
△OAC=(√5/2)xsinθ
△OAC*5=△OAB=(5√5)sin3θ/2
(5√5)xsinθ=11
5x=11
x=11/5 

ああ・・・ありがとうございました皆さん
961132人目の素数さん:2012/01/10(火) 02:37:36.87
ペイントに悪戦苦闘してたら解決してたか遅かった
内分する場合と外分する場合というのはこういうこと
http://iup.2ch-library.com/i/i0530478-1326130388.jpg
962132人目の素数さん:2012/01/10(火) 02:54:56.96
>>961
わざわざありがとうございます
心から感謝しています
C'を直線OC上にとり、角COBの二等分線とBC'の交点をA'とし、
C'A' :BC'=1:4 となるようにC'の位置を定める。
△OACの余弦定理とOCBの余弦定理 で求めたつもりでも、
△OA'C'の余弦定理と△OBC'余弦定理 を使ったかのような計算になってしまう
ということですか?
確かに△OCBに使ったつもりの余弦定理は
>16y^2=25+x^2-10xcos2θで、
OC'=x,A'C'=yとすると △OBC'に余弦定理を用いると
16y^2=25+x^2-10xcos2θ
となり一致しますが、
△OACに使ったつもりの余弦定理は
>y^2=5+x^2-(2√5)xcosθ で、
△OA'C'に使う際、√5の辺に隣していない・・と思いきや
A'C'は偶然にも√5なんですかね。だとすれば
y^2=5+x^2-(2√5)xcosθ が成り立ちますね。
こういうことでしょうか?
963132人目の素数さん:2012/01/10(火) 02:56:22.18
訂正;
>A'C'は偶然にも√5なんですかね。だとすれば
O'C'です。すみません。

964132人目の素数さん:2012/01/10(火) 03:06:42.48
訂正:
O'C'じゃなくてOA'です。ほんとにごめんなさい.
965132人目の素数さん:2012/01/10(火) 03:16:33.27
>>963
OA'は√5だよ
OA=√5としてxを求めたんだから必然
966132人目の素数さん:2012/01/10(火) 03:41:06.03
>>946
x = 5/3 のほうが不適になることは
メネラウスの定理などで比を確認すればわかる
967132人目の素数さん:2012/01/10(火) 03:42:08.13
OA'=√5となるように作図したということですか?
つまり∠BOCの二等分線上にOA'=√5となるようなA'をとり、
BA'とOCの交点をC'とする。と言った形です。
このとき偶然A'C':BC'=1:4となったのでしょうか。

A'は本当にBC'上に存在するのですか?

∠BOCの二等分線上にOA'=√5となるようにとる。
そして、直線OC上に、A'C':BC'=1:4となるような点C'をとる。(C',A',Bは一直線上とは限らない)
こうであれば納得いきます!
968132人目の素数さん:2012/01/10(火) 03:47:28.80
チェバもメネラウスも使えない気がするのですが
補助線などひくのでしょうか。
>>954の2枚目の画像の色がついた線を除いた図形での話ですよね。
969132人目の素数さん:2012/01/10(火) 03:51:41.77
>>967
もともと∠BOCというのは、AC:BC=1:4になるように求めたものだよね
だからAC':BC'=1:4となったのは必然
970132人目の素数さん:2012/01/10(火) 03:55:58.99
AC:BC=1:4 ⇒A'C':BC'=1:4
これが成り立つ理由がわからないです...
971132人目の素数さん:2012/01/10(火) 04:11:34.79
>>968
>>966 は x = 5/3 のときの C を C’とし, AC’と OB の交点を B’として
△ACC’と直線OB’B に対して定理を用いたのであるが
AC : CB = 1:4 を前提に比を考えていた
だから答案には使えないかも 申し訳ない

ただ, AC’: C’B’ = 1:4 , AC : CB を未知として定理を適用すれば,
AC : CB = 125 : 132 となって
C が AB の内分点であることにふさわしくない比の値が出てくる
972132人目の素数さん:2012/01/10(火) 04:16:40.78
>>971 訂正
誤 AC : CB = 125 : 132
正 AB : BC = 125 : 132
973132人目の素数さん:2012/01/10(火) 04:19:35.28
>>971
x=5/3のときを仮定し、矛盾を導くということは
計算が上手くいき、求めたかったOCの値が5/3で正しいと仮定すると、他のところで矛盾が生じるということですよね。
ということはOC=5/3を満たすCはAB上に存在するということを仮定することになりますよね。
仰られているのは「OC=5/3を満たすCが直線ABに関してO側にある」としてしまっている気がするのですが..
974132人目の素数さん:2012/01/10(火) 04:20:45.35
>>970
ごめん、条件が足りてなかった
OA=√5,OB=5,∠BOC=2∠AOC,AC:BC=1:4
これを満たすように∠BOCを求めたのだから、
OA'=√5,∠A'OC'=∠AOCとなるA',C'について
AC':BC'=1:4となるのは必然

今更だけど、AC:BC=1:4になるようにcosθを求めたんだから、
AC=y,BC=4yとして余弦定理から連立するのは全然意味ないよ
cosθを求めるときはOAとOBの比しか考慮されていなくて
連立するときにOA (orOB)の値を入れたから解答が進んだだけのこと
975132人目の素数さん:2012/01/10(火) 04:36:08.53
>>974
なるほど!!
△OACと△OBCは、
2∠AOC=∠BOC、
OA=√5, OB=5
AC:BC=1:4 である。
△OA'C'と△OBC'は
2∠A'OC'=∠BOC'
OA'=√5, OB=5
ならばA'C':BC'=1:4 ってことなんですね。(反復しただけですが)
相似ではないけどなにか似たようなものがあるんですね。

OA,OBの長さを条件として盛り込めばいずれにしても解けるということですね。
余弦定理、面積・・・ぐらいしか私は思いつきませんが..
ありがとうございます。
976132人目の素数さん:2012/01/10(火) 04:39:17.95
ひとつだけ。
B,A',C'は一直線上にあるかどうかはわかりませんよね?
977132人目の素数さん:2012/01/10(火) 04:41:57.06
>>973
∠AOP = θ ,∠AOQ = 3θ ,∠POQ = 2θ となるように点 P , Q をとり,
半直線 OP 上に 2点 C[1] ,C[2] をとる( C[1] が x = 11/5 のほう )
AC[1] ,AC[2] と OQ との交点をそれぞれ B[1] , B[2] とする
B[1] , B[2] のどちらが本当の B であるかを確かめればいい
で, B[2] のほうは >>971 のように不都合が生じる

というつもりだったのだが,やはりまずいかな…
978132人目の素数さん:2012/01/10(火) 04:42:23.13
979132人目の素数さん
>>977
あ、理解できました!
それなら問題ないですね、ありがとうございます。